Downwind faster than the wind: Blackbird sets a record

Education Science
Downwind faster than the wind: Blackbird sets a record

201011041335
Blackbird wind cart. Photo: Steve Morris

Introduction

By Mark Frauenfelder

In 2007, MAKE projects editor Paul Spinrad sent me a link to a YouTube video of a wind-powered cart, made by a Floridian named Jack Goodman, that seemed to be able travel directly downwind faster than the wind. How could a wind-powered cart outrun a tailwind, we wondered? Intrigued, Paul and I asked contributing editor Charles Platt to repeat the experiment and report on the results for MAKE.

Charles built a small cart, but was unable to get it to outrun the wind. Charles wrote in his article (which appeared in MAKE Volume 11 in August 2007), “Perhaps Jack Goodman has some clever explanation for this. Perhaps I didn’t build my version exactly the same way that he built his. Perhaps you should build your own, just to make sure.” In the end, Charles suspected Goodman had played a prank.

As soon as the issue hit the newsstands, our message boards (here, here, here, and here) began to boil with impassioned arguments over the theoretical and practical possibility of travel “DDWFTTW” (Directly Downwind Faster Than The Wind). Several people emailed me, including a fellow named Rick Cavallaro, about errors in MAKE’s wind cart design. I soon came to the conclusion that we had likely misunderstood the design of Jack Goodman’s cart. I told Rick I would like to publish a follow-up article (authored by him) in MAKE, but not until I’d conducted more research, and not until Rick had developed a convincing DDWFTTW demonstration. Rick happily obliged, and in the months to follow, he kept me apprised of his impressive research efforts.

Meanwhile, I began corresponding with wind cart enthusiasts, skeptics, physicists, and Jack Goodman. There was no clear consensus — even some physicists told me DDWFTTW was impossible — but I was starting to side up with the proponents.

Then, in July 2010, Rick informed me that he and his colleagues had taken a downwind cart they’d built onto the El Mirage Dry Lake Bed in southern California and had achieved a speed 2.8 times faster than the wind. They carried out the test in front of authorized representatives of the North American Land Sailing Association (NALSA). It was very convincing, and I asked Rick to write an article about it, which you can read below.

Is DDWFTTW possible? In 2007 I didn’t think so. In 2010, I think so!

—Mark Frauenfelder, editor-in-chief of MAKE

The Little Cart That Did

By Rick Cavallaro

Is it possible to build a wind-powered vehicle that travels directly downwind, faster than the wind, powered only by the wind, continuously?

This is a question I asked myself several years ago. After working out a few vectors, I decided it should work just fine. But since this is pretty clearly counterintuitive, I figured I’d pose the question as a brainteaser on a kitesurfing forum and a
radio-controlled helicopter forum.

To my surprise, very few people believed my claimed answer was correct. They simply didn’t accept my vector analysis.

But more surprises were in store. It turns out that an aeronautical engineer by the name of Andrew Bauer built exactly such a vehicle in the 1960s. There exist a few still photos of Bauer standing next to his cart, but little other evidence. There’s no compelling documentation beyond Bauer’s own claim that he beat the wind speed briefly, and by a modest amount.

The next surprise came when a man by the name of Jack Goodman built a working model of such a cart to settle the now raging debates taking place across the internet. Goodman intended only to prove to his sailing buddies that the feat was possible, but one of those buddies posted Goodman’s proof (in the way of a video) on the internet.

This only fanned the flames. The video went viral. There were claims it was a hoax — that it was being pulled by a string, that it was going downhill, the wind wasn’t directly at its back, the wind wasn’t steady, and on and on. Goodman could have made additional videos addressing these concerns but, as we learned the hard way, doing so would not be trivial, and still few would be convinced.

In 2007, MAKE published an article titled “The Little Cart That Couldn’t” by Charles Platt. This article specifically addressed Goodman’s video, and concluded that Goodman had hoaxed us all.

windCart_2.jpg

Small demonstration wind cart. Photo: Rick Cavallaro

 

Learning the Hard Way

After more than a couple of years debating this on the internet, my old hang gliding buddy, John “JB” Borton, convinced me we would have to build our own cart and address the skeptic’s concerns if we hoped to convince them. And so we did.

To address the issues of whether the road was level, the wind was constant, the cart was going downhill, or being pulled by a string, we decided to perform (and document) carefully controlled tests. If this were an airplane, we’d use a wind tunnel. But since this vehicle is intended to go downwind — rather than into the wind — the appropriate instrument for our tests was a garden-variety treadmill.

Rather than move the air over the road, we’d move the road beneath the air. Galileo, Newton, and Einstein have assured us (and it’s now one of the most basic and accepted principles in physics) that these two situations should be identical from the point of view of the cart.

If our cart could hold its ground on a level, moving treadmill, it would be a demonstration of going directly downwind at exactly wind speed. If it could advance, against the motion of the belt, it would be an ideal demonstration of going directly downwind faster than the wind.

Of course there are those that argue there’s no wind in the room, so the treadmill test is not valid. But it’s really just a matter of looking at it from the cart’s point of view. If you ride your bike downwind at exactly wind speed you won’t feel any wind either.



Video footage of the first run of the two days spent on the lakebed in Ivanpah, NV

So, the little cart did as we predicted. Not only did it hold its own on a level treadmill, but it advanced against the belt consistently. In fact, we inclined the treadmill to its maximum angle, and the cart climbed the treadmill against the motion of the belt.

Still, most of the skeptics remained … skeptical. So we invited them to submit tests they’d like to see. We panned the camera to show there were no fans or strings. We hung streamers over the belt, in front of the cart, and behind. We placed a fan in front of the cart to create a headwind (as requested). But still the doubters (including Platt as well as some noted physicists and aero engineers) remained unconvinced.

Ultimately, we posted a detailed set of build videos (watch part 1 | part 2 | part 3) so that anyone could build their own working model of our cart and prove it to themselves for about $40 in parts. In Texas, group of high school students on a budget followed our plans and improvised them to keep the price below $20! The result? They won the science fair with
their demonstration.

Going Full-Scale

But with the skeptics still unconvinced, and the hang gliding and kitesurfing seasons winding to a close, JB convinced me we’d have to build a full-scale, manned cart that could operate outdoors in “natural” wind and be witnessed by trustworthy experts. And so we did.

With funding from Joby Energy and Google, we proceeded to build a cart that weighs 450 lbs and stands 23′ tall to the top of its 17′ propeller. We approached the North American Land Sailing Association (NALSA) to develop a record category specifically for wind-powered vehicles designed to go directly downwind faster than the wind. We worked with NALSA over the next several months, and ultimately established a world record by going 2.8 times the wind speed directly downwind on the 2nd of July 2, 2010 on the El Mirage Dry Lake Bed in California’s Mojave Desert.

201011041307Blackbird wind cart. Photo: Emilio Castaño Graff

Some skeptics still remain, but then there are still those who believe the Earth is flat and man never stepped foot on the moon. So we’ve decided to be satisfied with the evidence we’ve produced and the minds we’ve changed. And we’d certainly like to thank MAKE magazine for giving us the chance to set the record straight regarding “the little cart that did.”

201011041308

Blackbird wind cart. Photo: Steve Morris

In the spirit of MAKE and the scientific method, we truly hope some of you will follow our build videos and make one of these carts for yourselves. It’s not difficult to do, and if you have a hard time accepting that a wind-powered vehicle can go directly downwind faster than the wind, you owe it to yourself to see it firsthand.

Videos

Video shot by Richard Jenkins from the back of JB’s truck at El Mirage during the NALSA runs

First Ivanpah run

Second Ivanpah run

Last Ivanpah run

Discovery Channel segment on the Blackbird

Official NALSA announcement of world record run

Up next: On Monday, Make: Online will run my piece about wind carts and their proponents and detractors, and on Tuesday, you can read my interview with Jack Goodman, the wind cart designer who captured the attention of so many people. –Mark Frauenfelder

Editor’s note: We gave Charles Platt, the author of the wind cart article in MAKE Volume 11, the opportunity to respond. His response is below. Rick Cavallaro asked to respond to Charle’s statement, which is also below.

I have never denied that a vehicle may be designed that will move into a headwind if the propeller is geared appropriately. What I do not believe is that this vehicle can start from rest with the wind behind it, accelerate until it is moving at the same speed as the wind, and then continue to accelerate faster than the wind, i.e. into a net headwind, without any fluctuations in wind speed, and without any gear-shifting along the way. That is what the original video from Florida purported to demonstrate, and is where all the arguments began. I have always suspected that the Florida video was faked.

I know very little about Rick Cavallaro’s cart, and am not very interested, partly because Rick has been extremely abusive, obnoxious, and condescending to me, and partly because, as I say above, I am quite willing to believe that his vehicle can move into a headwind. Indeed, the very primitive cart that I built for my original MAKE article did succeed in edging forward into a strong blast from a large fan. Again, what I do not believe is that his vehicle or any other can start with a steady wind behind it, accelerate to a speed equal to that of the wind, and then continue to accelerate so that it is moving faster than the wind, in one uninterrupted process.

I have repeated myself in an effort to make this clear. — Charles Platt

Mr. Platt seems to suggest that he refuses to consider real-world evidence because he feels I’ve been “abusive, obnoxious, and condescending” to him. This strikes me as being ironically similar to our differing position on reference frames. From where I stand, Platt established the pattern of abuse and condescension in his emails to me. Perhaps we’re both right (or wrong) on that count.

Unfortunately, Platt wrote an article presenting an unfair attack on an honest man (Jack Goodman), without even contacting Goodman or making any reasonable attempt to reproduce Goodman’s results. To me this seemed unfair. My only hope here is to set the record straight. I can live with the fact that Platt, and many others, may never accept these results that have been ratified by NALSA (a qualified, independent, and disinterested organization). I hope that we can agree to leave it at that, and not attack each other in public any further. — Rick Cavallaro

372 thoughts on “Downwind faster than the wind: Blackbird sets a record

  1. DavidGlover says:

    When will he admit his mistake? He just does not get it.

    1. Charles Platt says:

      I recently learned, from a source whom I trust, that Rick’s vehicle has a ratchet that allows the propeller to spin freely (i.e. not driving the wheels) when the wind blows from behind. The observer told me that there was “no easy way” to disconnect this ratchet, and it was used in every run that he observed.

      This is a violation of the conditions that I set when I first issued a challenge. The challenge was to build a cart that behaves as Goodman’s cart allegedly behaved. It must begin by accelerating in response to a following wind, must reach a speed equal to that of the wind, and must continue to accelerate into the wind–without any kind of gearing or other transmission that changes the mechanical relationship between the propeller to the wheels during this sequence.

      A ratchet allows the propeller to spin freely without exerting a reverse force via the wheels when there is a following wind. It is an obvious way to get around the problem that a propeller which is angled to drive the wheels and move the cart forward into a headwind will have the opposite effect in a tail wind. This is the same old, same old problem which I raised at the very beginning, and which Rick told me was merely a reflection of my stupidity and lack of understanding of aerodynamics.

      I look forward to an admission from Rick that he had to add the ratchet to make the cart work. I also look forward to the $10,000 he pledged, although not with much expectation.

      Lastly I understand that the propeller on the cart has variable pitch. This, too, is a violation of the terms that I set out.

      1. DavidGlover says:

        You completely misunderstand the function and use of the ratchet. This video has no ratchet nor variable pitch: http://www.youtube.com/watch?v=EEuAqq8FINw

        By what you wrote I think you still don’t understand how the vehicle works.

        Then you say: ” It must begin by accelerating in response to a following wind, must reach a speed equal to that of the wind, and must continue to accelerate into the wind–without any kind of gearing or other transmission that changes the mechanical relationship between the propeller to the wheels during this sequence.”

        That is exactly what Blackbird does. It works – I was there at El Mirage. You have a fundamental blind spot to the claim and the reality of what is happening. Some day you will see the error and I hope you blame yourself because many people have tried to explain it to you, but I think you are unwilling to listen.

      2. Rick Cavallaro says:

        >> Violation of the original terms of the challenge

        Wrong again – in so many ways. First of all, you never accepted my challenge, nor did you offer me a challenge. Secondly, our ratchet exists to allow us to push and tow the cart around without the prop spinning. The ratchets were engaged during the runs. Our model carts have no ratchets (because we can pick them up and carry them around) – yet they still work just fine. These ratchets are a convenience for handling the cart and to allow us to brake at the end of the run without putting reverse loads on the transmission. They have nothing to do with the terms of the challenge you had with a different person.

        >> … which Rick told me was merely a reflection of my stupidity and lack of understanding of aerodynamics.

        Perhaps I should post the entire email thread between you and I. I’m pretty sure I never used that word. You’re the one that seems intent on making this abusive – and on getting it wrong at every turn.

        >>I look forward to an admission from Rick that he had to add the ratchet to make the cart work. I also look forward to the $10,000 he pledged, although not with much expectation.

        You simply refuse to read anything at all on this topic – huh? The arrangement between you and I was to be a bet. I offered you $100K against your $10K (as Mark Frauenfelder noted in his article). You refused the bet for reasons that only you know and we can only guess – even though it was 10:1 in your favor.

        You’re darn right that you’ll get no admission from me that I had do use a ratchet – because I did NOT have to use a ratchet. If you even made an honest attempt to understand this thing you would understand that.

        Instead I get exactly what I got throughout our email chain – evidence that you didn’t even read what I wrote. Instead you asked what made me so arrogant as to think I could write an article. What I wrote was an article to correct the article you wrote. And unlike you, I did my homework. What you did was to attack a man that you never even bothered to contact. That’s not journalism!

        >>Lastly I understand that the propeller on the cart has variable pitch. This, too, is a violation of the terms that I set out.

        There are three problems with that:
        1) You did not set out any terms for me. You’re in no position to do so. You refused to take my bet.
        2) Variable pitch is not required – we proved that with the small carts and with the manned cart in Ivanpah.
        3) If you understood this cart you would understand that this doesn’t violate the terms you set out for YOU to give $10K to Peslel. The only terms involving me paying anyone were based on a bet that you refused – and would have lost.

        If you insist on continuing to misrepresent our conversations I will post our entire email exchange here.

        1. Rick Cavallaro says:

          >> If you understood this cart you would understand that this doesn’t violate the terms you set out for YOU to give $10K to Peslel. The only terms involving me paying anyone were based on a bet that you refused – and would have lost.

          @ myself

          I blew it here. Platt offered Peslel $1K if Peslel could demonstrate DDWFTTW under Platt’s terms (which Peslel did).

          The arrangement between Platt and I was to be a 10:1 bet in Platt’s favor – my $100K against his $10K.

          I simply made an error when I spoke of Platt’s arrangement with Peslel being $10K.

      3. theflyingtinman says:

        Even if Rick’s vehicle did need a ratchet to work (which it doesn’t) how would that invalidate it as a DDWFTTW vehicle?

      4. ThinAirDesigns says:

        Yowza Charles — through your lack of research on the vehicle you just continue to dig your hole deeper.

        I can only assume that this lack of research is willful since rather than simply asking us questions about the vehicle (like you should have done with Goodman to start), or read the extensive construction blog (that you are well aware of), you just continue to make stuff up. There are so many things fundamentally wrong in your comments that it’s hard to know where to start.

        Charles:
        >>I recently learned, from a source whom I
        >>trust, that Rick’s vehicle has a ratchet …

        The ratchet in question is no secret — it’s fully documented on the construction blogs, pictures and everything.

        http://www.fasterthanthewind.org/2010_04_01_archive.html
        http://www.fasterthanthewind.org/2010/05/higher-torque-upgrades.html

        Let me point something very important here — the ratchet has absolutely *nothing* to do with normal operation of this craft. The ratchet has *not once* released and gone ‘click, click, click’ during the running of the vehicle. Easily demonstrated physics dictates this and video documentation shows it.

        Here’s the purpose for the ratchet on the prop shaft — if the driver of the Blackbird has to make a panic stop, the spinning mass of the propeller would try to keep it spinning while the brakes (on the drive axle) try to bring it to an immediate stop. For safety reasons, the brakes are much more powerful than the propulsion system (just as on any car). We wanted strong brakes (for obvious reasons), but we don’t want our transmission to snap just because the driver had to slam on the brakes.

        Without the ratchet, the mass of the prop would backdrive the transmission and overload it. With the ratchet, in a panic stop the vehicle can be brought to zero quickly, while propeller can at its own pace, coast to a stop independent of the vehicle movement — just going ‘click, click, click’ until it RPMs are zero.

        As an additional plus, the ratchet allows us to tie the prop down and push the vehicle around by hand (backwards) without the prop spinning and getting in our way or hitting something.

        Again, since under normal operation the torque on the shaft is always in the same direction and the ratchet is FORCED by this torque to remain engaged the entire time during and run.

        We’ve never had to make a panic stop so it’s never even been used for it’s primary purpose.

        Charles:
        >> [this ratchet] allows the propeller to
        >> spin freely (i.e. not driving the wheels)
        >> when the wind blows from behind.

        First, the prop NEVER tries to drive the wheels forward under ANY circumstances other than panic stop, so this parenthetic notion you put forth above (i.e. not driving the wheels) is ALWAYS true — no ratchet needed to influence that.

        Second, since I’m familiar with every single observer who was at every one of our tests, I can assure you that you are misundstanding whatever it was that they told you.

        Ask ANY of those observers this question:

        “Before and during each run that you observed, when the Blackbird was sitting still in the tailwind or running below windspeed (i.e. “wind blowing from behind”):

        A: was the propeller spinning freely as Charles Platt asserts the ratchet allowed?

        or

        B: was the propeller’s rotational speed directly tied to and spinning directly proportional to the rotational speed of the wheels?”

        In other words, “did the prop every spin freely in the early stages of the run, or was the propeller always tied directly to the wheels?

        Their answer to a man will be “B”.

        I also know that each of them is willing to talk publicly about their observations and they know we would encourage them to do so and actually hope that they would.

        In light of that I encourage you to email your observer that question and ask him to respond with “A” or “B”. and post the answer here. Thanks.

        Let’s be clear — I’m not accusing you of lying here, but I am most assuredly stating that just like your continued misunderstand of the fundamental operation of the craft, you have misunderstood the operation of the ratchet. You owe it to yourself, your readers, Pelesl (whom you owe $1,000 to as he fulfilled your original requirements, word for word btw) and especially the observer you have just misrepresented to get and post the observers answer to that above question. What it will show is that again are attempting to defend yourself through lack of knowledge and understanding.

        Additionally, as someone else already posted, there is plenty of video around showing NALSA observed runs (in Ivanpah) where the prop shaft ratchet was NOT installed (it broke just before we left — again documented in the construction blog) and there was NO variable pitch hub in use (not built yet — again documented in the blog) and still we went near 3x the speed of the wind from a standstill. These points themselves demonstrate the fallacy of your position.

        Your lack of knowledge on this topic, and your willingness to make stunningly false assertions based on that lack of knowledge is a serious setback to any credibility you might wish to have as a journalist. You really should step up to the plate, do the sort of research the your Editor-in-Chief did and return wiser and accurate. Your not doing yourself favors at this point.

        JB

      5. ThinAirDesigns says:

        @Charles Platt:
        >>> “I recently learned, from a source whom I trust, that Rick’s vehicle has a ratchet that allows the propeller to spin freely (i.e. not driving the wheels) when the wind blows from behind. The observer told me that there was “no easy way” to disconnect this ratchet, and it was used in every run that he observed.”

        In my earlier post, challenged you to email a specific question to your observer source. The purpose of this clearly worded question (see bottom of post) is for YOU to find out for youself if the assertion you make above is actually true, or if you simply misunderstood what the observer told you.

        http://www.nalsa.org/BlackBirdDDWSR/Observers%20ReportNALSA%20C4BlackbirdJuly2and%2032010.pdf

        You will notice that the official Observers Report from NALSA includes a section/paragraph listing everyone who was at the runs. Attending folk were asked to ‘sign in’ and leave their info in case there was a need to refer to them later (like perhaps someone who wasn’t there later makes claims that aren’t true).

        To put it bluntly, I have no faith that you will actually email that question to your observer source on your own initiative and post the answer here, so in the spirit of Make Magazine (a go out and do it yourself if you want it done publication) I will be performing that service here, in this public forum.

        So here’s how it’s going to work — one by one each of the observers on that list are going to read that question and respond with their A/B answer (and any other comments they may wish to add). I will post their answers here as I receive them. They will all respond and will give me permission to post their answer because that’s the kind of folk that they are – all of them took the time out of their busy lives to travel to a god-forsaken spot in the desert to observe the tests. Most drove for hundreds of miles, some flew for thousands, all on their own dime and they will not shy away from such a challenge I am sure.

        The way for you to look your best would be for you to FOR ONCE, get ahead of the curve on this mistake you’ve made and to return here with a statement saying that it’s a simple mistake that you’ve made in this case and that even if you don’t understand how the ratchet was integrated and used, what you DO know is that contrary to your assertion, the propeller wasn’t “freewheeling” when the vehicle had a following wind but rather was tied directly to the rotation of the wheels.

        Here is a reprint of the relevent question:

        “Before and during each run that you observed, when the Blackbird was sitting still in the tailwind or running below windspeed (i.e. “wind blowing from behind”):

        A: was the propeller spinning freely as Charles Platt asserts the ratchet allowed?

        or

        B: was the propeller’s rotational speed directly tied to and spinning directly proportional to the rotational speed of the wheels?”

        In other words, “did the prop every spin freely in the early stages of the run, or was the propeller always tied directly to the wheels?

  2. DavidGlover says:

    Richard Jenkins shoots unedited video, and read his write up – very funny – RIchard is the world land speed record holder for a land yacht. http://www.fasterthanthewind.org/2010/07/video-from-richard-jenkins-world-land.html

  3. Rick Cavallaro says:

    I find it more than a little ironic that Platt thinks I’ve been abusive, obnoxious, and condescending. I’ll be more than happy to post the email exchanges between Platt and myself so that people can decide form themselves who has been abusive, obnoxious, and condescending.

    I think people will find that Platt’s version of reality in that respect is similar to his unfounded denial of the facts shown clearly in these videos – which he was invited to witness for himself.

    But I guess I can understand his refusal to accept the obvious since he has a $1000 incentive to not understand. Platt offered another reader $1000 of his own money if that reader could demonstrate DDWFTTW. He did. Platt did not pay.

  4. http://david.rysdam.org/blog/ says:

    I see a link but the word “vector” doesn’t appear over there.

    1. http://david.rysdam.org/blog/ says:

      I see now that that link has multiple pages. Page two does mention vectors. There’s even a broken img tag that I guess used to have a drawing. I’d be interested in a page dedicated to the mathematics of this (in pictures and equations, which are harder to argue with than hand-waving explanations).

  5. ThinAirDesigns says:

    Platt:
    >>
    Again, what I do not believe is that his vehicle or any other can start with a steady wind behind it, accelerate to a speed equal to that of the wind, and then continue to accelerate so that it is moving faster than the wind, in one uninterrupted process.
    >>

    And yet that is what exactly what the Blackbird has done scores of times in front of dozens of independent witnessess. This including Officials from NALSA while 18 recording sensors were documenting the easy to achieve feat.

    Charles, when the evidence disproves your theory, it’s time to admit you were wrong and come up with a new theory.

    JB

  6. Rick Cavallaro says:

    >> I see a link but the word “vector” doesn’t appear over there.

    Which link did you follow? I don’t know that my initial vector analysis is on-line, but I’ll be more than happy to post it.

  7. http://david.rysdam.org/blog/ says:

    “I figured I’d pose the question as a brainteaser on a kitesurfing forum and a radio-controlled helicopter forum.

    To my surprise, very few people believed my claimed answer was correct. They simply didn’t accept my vector analysis. ”

    The helicopter thing is a link. Since they “didn’t accept” a vector analysis, I assume one was given in that thread.

    Yes, I’d very much like to see it. For one thing, I can’t follow any of these YT videos. I can’t even tell what is being claimed, let alone what is really happening (i.e. wind directions or speeds, rotation directions or speeds, etc).

  8. ThinAirDesigns says:

    Hi David,

    I’ll let Rick get you the vector analysis you are looking for, but I wanted to responsed to this:

    >> I can’t even tell what is being claimed,

    Here’s the claim:

    “Directly downwind, faster than the wind, powered only by the wind, steady state”.

    JB

  9. NeuroPulse says:

    Has anything new happened since July?

    What prompted this article now?

    1. ThinAirDesigns says:

      Make originally published an article which contained incorrect conclusions. Fortunately, the editor of the magazine has been following the progress of the Blackbird project and decided to to the right thing by publishing an article with made the appropriate corrections.

      Kudos to Mark Frauenfelder and any editorial staff involved.

      JB

    2. Mark Frauenfelder says:

      We are running Rick’s article in MAKE, but since we are a quarterly, we wanted to run it online before the story got too old.

  10. Rhudson says:

    My take on the wind wagon:

    The prop drives the wheels, but the wheels also drive the prop. The wind spins the prop, but the prop also provides lift. Think of it as an autogyro on its side.

    This cart operates in 3 states:

    1. Starting downwind, the wind pushes on the prop (and cart) causing the prop to spin, transmitting power to the wheels and off she goes. The lift from the prop increases and pulls the cart along.

    2. Matching wind speed, the prop freewheels and begins to receive power from the wheels. The lift from the prop is still pulling the cart forward.

    3. Exceeding the speed of the wind, there is now a headwind. The momentum of the cart is causing the prop to continue rotating, causing it to act like an airplane prop and pull the cart along. Friction enters the scene (fah, it was always there!) and eventually slows the cart back below windspeed.

    disclaimer: I am not coming to this as a scientist, but as a sailor and model airplane builder.

    1. ThinAirDesigns says:

      Hi Rhudson:

      >>> 1: . Starting downwind, the wind pushes on the prop (and cart) causing the prop to spin, transmitting power to the wheels and off she goes.”<<< There is never any power transmitted to the wheels from the prop. From the very first moment of movement, the rolling wheels force the propeller to turn. There is even a ratchet on the prop shaft that ensures the prop can never drive the wheels. http://www.fasterthanthewind.org/2010/04/not-this-time.html http://www.fasterthanthewind.org/2010/05/higher-torque-upgrades.html >>> 2. Matching wind speed, the prop freewheels and begins to receive power from the wheels. The lift from the prop is still pulling the cart forward.”<<< As pointed out above (and by Neuropulse), the prop never freewheels, but rather recieves power from the wheels at *all* times and *all* speeds. >>> 3. Exceeding the speed of the wind, there is now a headwind. The momentum of the cart is causing the prop to continue rotating, causing it to act like an airplane prop and pull the cart along. Friction enters the scene (fah, it was always there!) and eventually slows the cart back below windspeed.”<<< The vehicle will continue to travel steady state, well wind speed as long as there is wind and a surface to run on. It does not repeatedly cycle above and then below wind speed, but rather exceeds wind speed and stays there. JB

      1. Rhudson says:

        Well, I never claimed to be a physicist ;) I hope at least one of my points was right though.

        1. ThinAirDesigns says:

          No worries Rhudson — I wasn’t trying to come of a critical of you personally, just trying to set the record straight on how it works.

          JB

          1. spleen says:

            I wonder if acceleration could be increased (or matched) by using the variable pitch on the propeller to drive the vehicle at below wind speeds (that is use the rotational energy of the propeller to drive the wheels). As the car approaches wind speed the blades are flattened acting as a sail (The vehicle is simply being pushed by the wind). Lastly to go beyond wind speed, the pitch is altered to provide the air cushion acceleration (The same method they employ now). I understand that this would not increase there top speed, but it would be interesting to hear if this method would work also.

          2. Rick Cavallaro says:

            That method would absolutely work. We originally ran with a fixed pitch prop, and later went to a variable pitch (which was planned from the start), but for several reasons we opted not to let the prop drive the wheels (we used ratchets to prevent the possibility). This was partly due to the design of our drive-train, and partly to prove to NALSA that we could not make any use of stored momentum or energy to accelerate.

            So we did start out with a flat pitch and increase it as our speed increased, but we never ran a negative pitch.

          3. spleen says:

            Thanks for your reply. I was also wondering about the plausibility of an electrical drive train v the mechanical one you are using. I believe that generators and hub motors can provide around a 95% efficiency of transfer, though I do not have any figures to back this. I guess that weight is critical to you, but it may be a simple way of varying your wheel speed without a mechanical CVT.

          4. Rick Cavallaro says:

            Actually weight isn’t all that critical. It affects our acceleration, but has relatively little impact on our top speed as rolling resistance is a small piece of our overall losses.

            Also, we don’t currently do any gear changing. We have a gear cluster for finding the optimal ratio, but we haven’t changed it since our early experiments. Once you’re in the neighborhood on gearing you can achieve the same effect to first order by adjusting prop pitch.

            For a larger vehicle a generator and electric motor could make sense for a number of reasons. On Blackbird our drive is pretty simple, and I think higher efficiency than we could achieve electrically.

          5. spleen says:

            Thanks for the comments, I think it is a fascinating project and wish you the best.

          6. Rick Cavallaro says:

            And thank you.

    2. ThinAirDesigns says:

      @Rhudson:

      Sorry, but this will be my second attempt at answering your post — my answer just above somehow got delivered mangled and incomplete.

      Here goes round 2:

      “1. Starting downwind, the wind pushes on the prop (and cart) causing the prop to spin, transmitting power to the wheels and off she goes.”

      The prop *never* transmits power to the wheels. Not at any speed nor at any time. In fact, there is a large ratchet on the prop shaft that ensures that this can never happen:

      http://www.fasterthanthewind.org/2010/04/not-this-time.html

      http://www.fasterthanthewind.org/2010/05/higher-torque-upgrades.html

      “2. Matching wind speed, the prop freewheels and begins to receive power from the wheels. The lift from the prop is still pulling the cart forward.”

      The propeller was recieving power from the wheels the entire time and at all speeds — see above.

      “3. Exceeding the speed of the wind, there is now a headwind. The momentum of the cart is causing the prop to continue rotating, causing it to act like an airplane prop and pull the cart along. Friction enters the scene (fah, it was always there!) and eventually slows the cart back below windspeed.”

      The vehicle will continue to travel steady state, well wind speed as long as there is wind and a surface to run on. It does not repeatedly cycle above and then below wind speed, but rather exceeds wind speed and stays there.

      JB

      (Hope the formatting works this time — crossing fingers)

      1. elros says:

        It appears to me that understanding this involves getting past an initial assumption. I had initially assumed that it was wind rotating the propeller that drove the wheels. Once I saw your note correcting this, the entire system makes sense. Initial acceleration depends on the wind pushing the cart in general, not on wind rotating the propeller.

        Would I be correct in concluding that at the point where the speed of the cart equals the wind speed, it is the momentum of the cart spinning the wheels driving the propeller to accelerate the cart? What would then be the theoretical maximum speed for the cart?

        Would this principle work on a windless day, with an external vehicle getting the cart moving in the first place?

        1. theflyingtinman says:

          ?? Would this principle work on a windless day, with an external vehicle getting the cart moving in the first place?

          No. It relies entirely on a difference in speed between the air and the ground to work. If it could work in still air, starting from a push, you would have a form of “perpetual motion”, which we all know is impossible.

        2. Rick Cavallaro says:

          >> Would I be correct in concluding that at the point where the speed of the cart equals the wind speed, it is the momentum of the cart spinning the wheels driving the propeller to accelerate the cart?

          No. We never make use of stored momentum or energy. Even at exactly wind speed the cart could pull a trailer all day long and then accelerate to beyond wind speed.

          >>What would then be the theoretical maximum speed for the cart?

          There is no theoretical maximum speed as a multiple of wind speed. There is however a maximum practical absolute speed. Once your prop tips get into the range of about 200 mph you’re pretty much done because of compressibility effects of the air.

          >>Would this principle work on a windless day

          No

          1. Jon Seymour says:

            Rick,

            Isn’t it more correct that upon reaching steady state, Blackbird does not make use of any stored energy or momentum.

            I would have though that the initial losses incurred by the propeller are “borrowed” from the accumulated kinetic energy of the cart (as accumulated from the initial apparent tail wind) until they are paid back from the net momentum transferred from the airmass by the propeller itself.

            In other words, during the initial “pushed by the wind” phase, the cart travels slightly more slowly with the propeller engaged than it would if it was disengaged.

            jon.

          2. Rick Cavallaro says:

            >> Isn’t it more correct that upon reaching steady state, Blackbird does not make use of any stored energy or momentum.

            Nope. In a stead wind the vehicle would accelerate monotonically until it reached its max speed. At no time would we take use any of the momentum of the cart or spinny bits for any reason. In fact we just keep *adding* momentum until we reach steady-state. But it’s just there – it can’t be avoided, and we don’t make use of it.

            >>
            I would have though that the initial losses incurred by the propeller are “borrowed” from the accumulated kinetic energy of the cart
            << Nope. To borrow from the kinetic energy of the kart the kart would have to slow down - which it doesn't do until I apply the brakes. >>
            In other words, during the initial “pushed by the wind” phase, the cart travels slightly more slowly with the propeller engaged than it would if it was disengaged.
            << That's quite true. That's simply a battle of forces and torques. The wind is trying to push the propeller CCW (as viewed from behind) while it's also trying to push the whole thing downwind (which is trying to turn the prop CW). The greater torque prevails.

          3. Jon Seymour says:

            Ok, so the propeller is engaged from the start of the run?

            Would it be true to say that _if_ the propeller was engaged later in the run, one might expect to see a transfer of kinetic energy from the translational components to the propeller and hence a drop in velocity?

            jon.

          4. Rick Cavallaro says:

            >> Would it be true to say that _if_ the propeller was engaged later in the run, one might expect to see a transfer of kinetic energy from the translational components to the propeller and hence a drop in velocity?

            Sure, if you “dropped the clutch” on the prop very abruptly, it might slow the vehicle down just a bit. That is *if* we had a clutch – which we don’t.

  11. Gareth Branwyn says:

    I just want to remind folks here, as this conversation continues, that we have a “be nice” policy here on Make: Online. You can read the guidelines on the link below.

    As this is a controversial subject and there are strongly opposing views, things can easily get personal and ugly. Please try and keep comments productive and about the experiments themselves and the science involved, not personal. Thanks!

  12. Prang says:

    The treadmill is not a valid test. Put the cart in a wind tunnel since it is wind-powered. Let it run on rollers. It will never exceed the wind speed in the tunnel. I guarantee it!

    1. ThinAirDesigns says:

      >>The treadmill is not a valid test

      The world record wasn’t set on a treadmill — it was set on a perfectly level dry lake bed in a steady wind with 18 recording sensors on vehicle and course placed by an independent testing organization and witnessed by noted sailing contest officials and writers.

    2. NeuroPulse says:

      @Prang

      It is important, crucial, that one understands that the vehicle does not accelerates because the wind spins the prop which spins the wheels. It accelerates because the ground is moving relatively beneath it spinning the wheels which spins the prop which propels the vehicle.

  13. NeuroPulse says:

    @Rhudson

    “1. Starting downwind, the wind pushes on the prop (and cart) causing the prop to spin, transmitting power to the wheels and off she goes.”

    I think it starts moving because the wind is pushing on it as it would anything wheeled, prop or no prop.

    “2. Matching wind speed, the prop freewheels and begins to receive power from the wheels. The lift from the prop is still pulling the cart forward.”

    The energy goes from the wheels to the prop the entire time.

    “3. Exceeding the speed of the wind, there is now a headwind. The momentum of the cart is causing the prop to continue rotating, causing it to act like an airplane prop and pull the cart along. Friction enters the scene (fah, it was always there!) and eventually slows the cart back below windspeed.”

    It does not slow down. It goes about 3 times faster than the wind and continues at that speed.

    1. Rhudson says:

      I’m sure I would have gotten a different impression had I seen it operate in person. I was wrong about the steady state thing.

      I’m still fairly sure that the propeller’s lift enters into the equation.

      1. NeuroPulse says:

        @Rhudson

        “I’m still fairly sure that the propeller’s lift enters into the equation.”

        Yes, the propeller *pulls* the vehicle through the air just as a plane propeller pulls the plane through the air. A plane will move forward regardless of whether or not the plane is going slower or faster than the wind.

        What causes so much confusion is the source of the power is the same thing that is propelling the vehicle, the propeller.

        The propeller is acting both as a sail and a propeller. The wind pushes on the propeller like a sail. This moves the vehicle over the ground turning the wheels. The turning wheels turn the propeller and provide more thrust.

        1. edgertronics says:

          What you describe is infact the basis of many perpetual motion machines.

          The most reasonable cause is the following:

          Like a wind turbine extracts kinetic energy from oncoming wind, the propeller (when the cart is moving faster than the downwind) will act as a turbine and should drive the wheels in order to take the cart forward. This does however require the coefficient of Drag to be less than the total energy gain from the prop,which I can make no conclusion on.

          1. ThinAirDesigns says:

            @edgertronics
            >>> “The most reasonable cause is the following:
            Like a wind turbine extracts kinetic energy from oncoming wind, the propeller (when the cart is moving faster than the downwind) will act as a turbine and should drive the wheels in order to take the cart forward.”

            Edger — *you* may find your described “cause” as the most reasonable, but since the design of the machine makes your theory impossible, there’s not much ‘reasonable’ about it.

            There is a ratchet on the prop shaft which specifically prohibits the propeller from ever being able to drive the wheels forward.

            http://www.fasterthanthewind.org/2010/04/not-this-time.html

            http://www.fasterthanthewind.org/2010/05/higher-torque-upgrades.html

            This ratchet had to be included and functioning (confirmed by NALSA observers) to meet the NALSA rules that no stored momentum from the propeller could be used to drive the vehicle.

  14. clide says:

    Very interesting experiment, but I am even more amazed at how bullheaded people can be. If they can’t accept it with this much evidence then I guess they never will.

    I’ve ran into naysayers like that before (regarding a different subject) and it is terribly frustrating. I offered to do any reasonable demonstration to prove my point, but luckily they were honest with me and flat out told me that they would assume any demonstration on my part was faked. Although I still did an experiment to prove it to myself. I just couldn’t believe how unwilling people were to accept an idea different than their preconceived notions.

    Anyway, great work Rick. You’ve definitely proved your idea to any reasonable person.

  15. edgertronics says:

    While I don’t necessarily believe this to be impossible. One ridiculous factor riles me so.
    Richard, you obviously have a very interesting concept of reference frames. I would suggest that you go back and ask you first science teacher about it as it is really rather tragic. Your treadmill experiment is not only non-sensical but embarassing, the test is only valid if the cart is moving in relation to the air surrounding it. ipso fact if the carts not moving relative to the ground and neither is the air then you’re doing sweet fa, and miraculously you seem to have created a cart that can roll up hill!!!!!!!! If only if only. I would suggest taking the cart to your local university with a wind tunnel and asking the engineering department to test it, for currently you have NO PROOF whatsoever nor any scientific basis for your “invention.” I must also add that your “vector analysis” is also BS

    1. Gareth Branwyn says:

      Please don’t resort to telling people to go back and query their grade school science teacher (or similar insulting comments). That’s really unnecessary to arguing your points or refuting others. Thanks. – The Mgt.

      1. edgertronics says:

        If make is going to be a sensible proponent of citizen science I would suggest that the treadmill test never have been mentioned. It is both the fact that it found publication, AND that it is touted as meaningful that I take issue. The very topic of this is personal as we have one man, who has shown no formal analysis the theory behind the cart, allowed to publish his thoughts on the subject when it appears that his underlying knowledge is flawed. I do not refute the operation of the cart, simply that Richard has offered no solid grounds for why it does, nor a particular disposition for meaningful experimentation.

  16. Greg says:

    I watched the video on youtube with the model/treadmill as well as the video on Vimeo with the fullscale test. If they would include just a few more items it would strengthen the evidence much more….Include a few sensors!

    On the treadmill video if they just had a handheld wind speed meter showing how fast the wind was going and then give us a readout as to what the treadmill is doing…this would solve everything! Of course the cart will go faster than the treadmill if there is a 10mph wind while the treadmill is going 7mph.

    On the fullscale video, how do we know there isn’t a gust of wind pushing the cart to go faster? They obviously invested a lot of money building the cart, trucking it out there and video taping it…why wouldn’t you invest in some simple sensors to be done with it???

    Unfortunately posting videos of something just “working” is not enough. You can search youtube for plenty of videos of perpetual motion machines just “working”. Here is one: http://www.youtube.com/watch?v=epLOEaoPMFU&feature=channel

    I’m still skeptical…hopefully Mythbusters will come to the rescue. They’re always pretty good with setting up sensors and showing some actual numbers.

  17. Rick Cavallaro says:

    We had a total of 18 recording sensors for our record setting run. I believe you can see the data at http://www.NALSA.org

    Regarding the wind over the treadmill, the air in the room was still. We demonstrate this in some of our youtube videos. The wind in that experiment is the relative motion of the treadmill belt at 10 mph below the still air of the room.

    1. edgertronics says:

      There is no relative motion of the cart to the air however. This is the problem, because the cart is not moving in the room, and in still air on flat ground it cannot move unless it is propelled by another means

  18. ThinAirDesigns says:

    @greg:
    >>They obviously invested a lot of money building the cart, trucking it out there and video taping it…why wouldn’t you invest in some simple sensors to be done with it???

    NALSA oversaw the installation and data collection of 18 recording sensors during the record runs. These were installed on the vehicle itself, on chase vehicles and on the desert floor. They used the data from these sensors to determine the outcome of the runs.

    http://picasaweb.google.com/davidhglover/DDWFTTWJuly3RecordDay?feat=flashalbum#5490178307451061170

    http://picasaweb.google.com/davidhglover/DDWFTTWJuly2RecordDay#5489555438988192866

    http://picasaweb.google.com/davidhglover/DDWFTTWJuly2RecordDay#5489555487020943538

    http://picasaweb.google.com/davidhglover/DDWFTTWJuly2RecordDay#5489555272027178082

    Etc.

    How many more sensors might you need?

    JB

    1. Slapyak says:

      The number of sensors are not the problem.

      The lack of mathematical proof or presentation of the true physics is the issue.

      The prop and wind speed are a red herring if I understand correctly.

      A quick gust of wind would be enough to get it moving and then the wind could disappear completely and the craft would continue to accelerate?

    2. Greg says:

      First, the comment about there not being any sensors was in regards to the video as being proof enough that this concept works. The video alone does not show any sensors.

      Now onto the paper posted at NALSA. Here are my comments:

      For any paper of this type to hold any weight, an extensive report on the measurement setup must be given. Location of sensors, brands/models of sensors, accuracy of sensors, method of recording data, method of manipulation/calculations of recorded data. The GPS sensors were mentioned, however any wind sensors present were not described in any real detail. The one plot-Chart 6 shows the true wind speed measured at the chase vehicle. No indication was given as to how these numbers were arrived at.

      Get an outside non-invested party to evaluate this. You should be able to find some grad student up at Berkley who needs funding. Have them write up a real research paper detailing the necessary items for a paper to hold up against scrutiny.

  19. cary says:

    So, to begin with, the propeller acts like a sail, driving the thing forward through the mast. Then, as speed increases, the wheels are driving the propeller, keeping momentum going kind of like a flywheel. The wind continues to act on the propeller, providing more ‘sail’ power through the mast, and the cart keeps building speed to it’s ability to slip through the headwind and other friction factors.

    Is that anywhere close to how it actually works?

  20. Slapyak says:

    The frustrating part of this article, and even the proof, is the lack of real hard physics/math proofs of what is really happening. The trick is all in the gear/screw ratios. As you can assume – minimizing/maximizing friction in all areas is the key to maximizing the gains (and therefore top speed)

    To start(t=0), the wind nudges the cart forward, as any wheeled object. The prop is a non-factor at t=0

    Once underway, (for the purpose of this example I’ll use made up numbers) We have a wheel, with a 1 ft circumference, rotating at 20 rpm, connected to a shaft & prop. The wheel is driving the prop (through friction of the wheel on the ground, driven by the wind behind to over come initial conditions)

    At this point we have a vehicle moving at 20 feet per minute. Driving a prop at 20 RPM.

    Here’s the fun part; each revolution of the prop (it’s an airfoil, like a plane’s wing, and produces forward thrust) produces enough forward thrust to PULL the cart 1.5 ft. There are a lot of things going on, but for the sake of the example, shaft friction, vehicle drag and other items are non factors. Essentially, picture the prop being driven by an engine, we can agree that faster spinning produces faster speeds of travel.

    So the prop is pulling forward at 30 feet per minute, while spinning at 20 RPM (with that connected shaft). There’s some slippage with this, but the net effect is to speed up the whole craft.

    At this point, if there were no drag, only friction to the wheel/ground system; we could accelerate infinitely.

    The only thing stopping infinite acceleration is the drag on the rest of the craft. Initial wind speed plays a key role in that it shifts the equilibrium point of the whole system. A faster initial wind speed, will add more ‘push’ to unity, and less drag afterward. By unity, I mean the point at which the vehicle is moving at the same speed as the wind (and therefore with 0 drag force)

    I hope this makes it a little clearer than the vagueness I’ve seem so far. If it really comes down to it, I could run the actual numbers, but in reality these numbers should suffice.

  21. ThinAirDesigns says:

    @edgertronics:

    >>> “The very topic of this is personal as we have one man, who has shown no formal analysis the theory behind the cart, … ”

    edger, you might want to check around a bit before you make claims that no “formal analysis” has been shown. That assertion is totally and completely false.

    In addition to many, many formal treatments published by Rick, here is a nice set from world renoun MIT professor Mark Drela.

    http://www.boatdesign.net/forums/attachments/propulsion/28167d1231128492-ddwfttw-directly-downwind-faster-than-wind-ddw2.pdf

    http://www.boatdesign.net/forums/attachments/propulsion/28168d1231128492-ddwfttw-directly-downwind-faster-than-wind-ddwe.pdf

    In the past Dr. Drela has been very responsive to interested folks like you who email him with questions. I suggest that you might wish to do so and what you will find is that Drela’s understanding of frames of reference are the same as Rick’s and that Drela will tell you that the treadmill experiment is absolutely valid.

    Give it a shot, but publish his response please — it would be educational for the others here to see whether your understanding is correct or not according to one of the most noted and accomplished aero professors of our time.

    JB

    1. Slapyak says:

      When writing about your own technological feat, for a fairly technical blog (and with the intense debate that has followed this one), perhaps including a link to your research and proof would be a wise idea to avoid some of this.

      If you would at least link to your proofs, there would be half as many comments, and they would be much more toned down.

      1. ThinAirDesigns says:

        @Slapyak:

        Slapyak, we were not asked by Make to submit an article containing comprehensive technical proofs. That is not even the normal content of Make Magazine. We were asked to submit an article telling the story of the Blackbird and link to instructions on how readers could build their own correctly (to make up for Platt’s incorrect instructions/plans).

        I have happily included links (directly above) to extensive proofs from a noted source.

        JB

  22. NeuroPulse says:

    Greetings,

    I thought I understood how this works, and have tried to help explain it. But, now I am confused again.

    When the vehicle is traveling at say 2.8 times wind speed, where does the energy come from to maintain that speed? Energy must be put into the vehicle’s system for it to maintain that speed in the face of air resistance and friction. The energy can not be coming from wind pushing on the vehicle from behind if it is going faster than the wind.

    Thanks

    1. ThinAirDesigns says:

      @NeuroPulse:

      >>> “The energy can not be coming from wind pushing on the vehicle from behind if it is going faster than the wind.”

      I will give you a short thought experiment that will show your above assertion to be flawed:

      Grab an ordinary bicycle. Have someone hold it upright. Place your finger halfway down the upper most spoke on the wheel of the bike (either one). Push forwards with your finger.

      You will find that the bike will move forwards twice as fast as your finger is pushing forwards on the spoke.

      Now to take it a step further, imagine replacing your finger with a sail sticking out from the spoke at that same point. Imagine a mechanism that would retract this sail at the bottom and deploy it at the top. Now equip each spoke with the same.

      It’s quickly apparent that even with the wind coming from behind and the bike going faster than the wind, the sail will be moving slower than the wind and there will still be wind pushing on the deployed sail and spoke, just as your finger pressed on the spoke.

      The above would require a very complicated (and lossy) mechanism to retract and deploy its sails, so I’m not presenting it as anything other than an example showing that energy *can* still be harvested from the wind even when the vehicle is traveling faster than the wind DDW.

      The trick is to use a mechanism that is more efficient in it’s use of the harvested energy than the lossy one described above — the wheel driven propeller is that mechanism.

      JB

      1. B_Petschel says:

        Without sounding crass, you either need to pick up your Nobel Prize or admit that either this is a mistake or fraud.

        Your thought experiment doesn’t work when the bike is traveling 2 times the velocity of the wind the “sail” will have net drag and not thrust. The thrust comes from the relative velocity difference between the sail and the wind.

        This is the same kind of garbage that you hear from people who hook a generator up to a motor and claim to have net energy.

        1. Mark Frauenfelder says:

          We have warned everyone not to use insulting language (“garbage”). In order to keep this discussion focused on the science (and not the personalities involved) I’m going to start deleting comments that use insulting words. This is a second warning. There will not be a third warning.

          1. B_Petschel says:

            Fair enough. I knew I was being crass and I should have shown more restraint.

            It is difficult to talk about a concept that explicitly violates Newton’s Second Law and conservation of energy. Simple mechanical analysis precludes their explanation. Where do we go from there?

            If their explanation holds any water then there is another experiment they could run to validate this concept.

            -In absolutely still wind conditions pull the car up to 5 to 10 mph.

            -The linkage between the wheels and the turbine should get it going just like in the video

            -Cut the line and have the vehicle continue driving for say another couple miles.

            This will not work. If it did, it would cause quite a stir.

            /Their machine looks cool!

          2. Rick Cavallaro says:

            >> It is difficult to talk about a concept that explicitly violates Newton’s Second Law and conservation of energy.

            There is no basis at all for that claim. Please try and explain exactly how you think our vehicle (or explanation) violates this law.

            >> Simple mechanical analysis precludes their explanation.

            Then please give us this simple analysis.

            >>If their explanation holds any water… Cut the line and have the vehicle continue driving for say another couple miles.

            There is nothing about our explanation that suggests that our vehicle will work in zero wind.

            If you want to understand the claims and how the vehicle works, I strongly recommend you join the discussion. Simply trying to ridicule our results and explanation will not get you there.

          3. B_Petschel says:

            I am not ridiculing your design. My first instinct was/is that your explanation violates the aforementioned principles. You clearly built a car that goes really fast with the wind and appeared to do just what you claim. I am not taking that away from you. However, your explanation is still way short of satisfying my questions.

            Maybe this would all be cleared up if you told us how your use of a turbine is different than a sail.

          4. theflyingtinman says:

            >> If their explanation holds any water then there is
            >> another experiment they could run to validate this
            >> concept.
            >>
            >> -In absolutely still wind conditions pull the car up
            >> to 5 to 10 mph.
            >>
            >> -The linkage between the wheels and the turbine should
            >> get it going just like in the video
            >>
            >> -Cut the line and have the vehicle continue driving for
            >> say another couple miles.
            >>
            >>This will not work. If it did, it would cause quite a
            >> stir.

            This assertion is 100% correct .. but irrelevant.
            There is no claim that the DDWFTTW vehicle can move at any speed in still wind. It requires a relative difference in speed between the air and the surface over which it travels to work. Consider this little experiment http://www.youtube.com/watch?v=k-trDF8Yldc which is a direct analog of the DDWFTTW vehicle (in which the ruler is equivalent to the wind and the large rotating wheel is equivalent to the prop.) Your “thought experiment” would suggest that just because the ruler can push the little cart in a particular direction faster than the ruler is moving itself that the cart could somehow be coerced to move under a stationary ruler by starting it with a push – which is obviously impossible. Again: the ruler has to move relative to the surface for the cart to move at all : And in the case of the DDWFTTW vehicle the wind has to be blowing at least strong enough to overcome friction for the vehicle to begin to accelerate to a speed at which the prop (driven by the linkage to the wheels) can drive itself by pushing against the air, faster than the air flowing over it, in exactly the same way as the big wheel of the little cart in the video, driven by its contact with the small wheels) drives itself by pushing the ruler, faster than the ruler moving over it.

        2. ThinAirDesigns says:

          @B_Petschel

          >>> “Your thought experiment doesn’t work when the bike is traveling 2 times the velocity of the wind …”

          Correct … that’s why I never said it *would* work at “two times the velocity of the wind” — I merely said with bike going “faster than the wind”

          >>> “…the “sail” will have net drag and not thrust.”

          Contrary to your assertion, there will be neither drag NOR thrust on the sail with the bike at 2x wind speed.

          With the sail positioned on the spoke 1/2 of the way between the tire and the hub (as prescribed in the thought experiment), the deployed sail will be traveling exactly 1/2 the speed of the bicycle. This means with the bike at 2x, the sail will be traveling exactly the speed of the wind and thus no drag OR thrust.

          If you wish to insert the requirement of “two times the velocity of the wind” into my thought experiment, you are welcome to do so — it merely requires that we adjust the point along the spoke where the sail resides.

          Let’s push with our finger at a point 3/4 of the way down the spoke towards the hub. Now obviously when we push at that location on the spoke, the bike goes 4x the speed of our finger. Now replace your finger with the previously prescribed sail at this same point.

          With the above adjustment, when the bike is going 2x the speed of the wind, the deployed sail is going only 1/2 the speed of the wind and thus can extract energy from the tailwind even with the bike going 2x the speed of the tailwind.

          >>> “The thrust comes from the relative velocity difference between the sail and the wind.”

          Yep — and I just demonstrated in a very simple way that there is still a velocity across the sail of 1/2 the speed of the tailwind.

          QED.

          JB

          1. zik says:

            I don’t think this thought experiment works.

            The point of the wheel in contact with the ground is traveling at speed zero. The hub of the wheel travels at the speed of the cart. The top of the wheel travels at twice the speed of the cart. Any point above the hub travels faster than the cart and any point below the hub goes slower.

            You would need to push on a point below the hub, near the ground, but this would slow the cart down as this force works against the rotational motion of the wheel.

            That said, I don’t need a mathematical proof to see that it works.

          2. ThinAirDesigns says:

            @zik:

            LOL — you are 100% correct. In my haste to write up the experiment I spaced on which side of the hub I needed to be on.

            Move all actions to below the hub but leave all the proportions and directions the same (1/2 the way down the spoke, 3/4 of the way down the spoke etc) and the thought experiment then works.

            Thank for pointing out the brain fart. Things do work better when you get them right. :-)

            JB

          3. chopin says:

            The proposed thought experiment correctly suggests that the “motion of the finger” is related to the motion of the wheel, and, thus, by connection to the spokes (and, presuming friction between wheel and ground), to the motion of the axle. Sounds great.

            Unfortunately, the extrapolation from “motion of the finger” to motion of a sail based on force of wind on sail holds no wind . . .

            Presuming a sail with 100% efficiency, all of the force of the wind is translated to the force on the sail (providing a sail that could do that would earn you a position with any number of America’s Cup teams).

            Having said that, translating all of that force into motion (including frictional losses) is a completely different proposition from saying that when point A on the radius of a circle moves, point B moves in a distance proportional to the radius. One is geometry, the other is physics. The FORCE required to *move* the finger still has to be sufficient to MOVE the opposing force at the axle and the outer diameter of the wheel. Unless your theoretical “axle-sail” can create that force without spilling wind (when the wind is behind it) or pulling air (when it exceeds the trailing wind-speed), your “thought experiment” needs “rethinking.”

            As a sailor, a kite flier, and an engineer, I can think of more than enough practical applications for your work, so I’d love to see it prove true, but so far, the background you’ve provided leaves me wanting for more before I’m willing to quit my day job.

          4. Rick Cavallaro says:

            >> …providing a sail that could do that would earn you a position with any number of America’s Cup teams.

            Hmmm… As it happens JB and I are only working for one America’s cup team at the moment. But we’re also working on a technology that will be used by all America’s Cup teams.

            >>As a sailor, a kite flier, and an engineer, I can think of more than enough practical applications for your work…

            As a sailor, kitesurfer, pilot, and aero engineer, I’d love to hear the applications you have in mind – because I assure you the cart works as advertised.

          5. clide says:

            This thought experiment is all about speed and has little to do with force. Since this is just a thought experiment the sails can be made however big is required to generate the necessary force.

            The point is that as the sails are deploying between the axle and the ground they are moving at half the speed of the vehicle. Yes this does require twice the force of just pushing on the back of the vehicle, but as I said that simply requires a bigger sail.

            Say you are going downwind in a 10 mph wind. Your vehicle is traveling 15 mph so that means the point between your axle and the ground is traveling 7.5 mph as would a sail attached to that point. So the sail would still have the same forces as if it was mounted directly to a vehicle traveling 7.5 mph.

          6. ThinAirDesigns says:

            Clide, you are of course correct and chopin is both wrong and missing the point of the thought experiment.

            As clide points out, the total system friction of the device can be made arbitarily low and/or the sails can be made arbitrarily big – thus a sail *can* work just as the finger. Those are the sort of things you get to do in thought experiments.

            The point of the experiment however was not to demonstrate that deploying sails from the spokes of a bike is a practical way to build a ddwfttw vehicle (it is not), but rather to leave lie to the concept that the wind cannot exert a force in the downwind direction on a vehicle moving downwind faster than the wind.

            JB

    2. Foozinator says:

      I think a good comparison is to triangle-shaped sailboats (not the square riggers). I don’t have a source, but IIRC, cats are capable of moving faster than the wind speed on a proper tack, mainly due to the lift the sail is producing.

      In this case, the wind is still providing the power, but the spinning prop is providing lift against the wind. The transmission between the wheels and the prop is just a mechanical way to allow a tack that wouldn’t otherwise provide lift.

  23. Rick Cavallaro says:

    @slaypak

    >> The frustrating part of this article, and even the proof, is the lack of real hard physics/math proofs of what is really happening.

    This article is not intended to delve into the analysis of the cart. I assure you we have posted analyses that are more complete and more accurate than what you describe above. But to be fair, your description is definitely more accurate than most.

    >>I hope this makes it a little clearer than the vagueness I’ve seem so far. If it really comes down to it, I could run the actual numbers, but in reality these numbers should suffice.

    If the mountain of data and analysis that we’ve posted over the years does not suffice, I can assure you that your brief description above will not set the skeptics straight. But thanks for trying.

  24. Rick Cavallaro says:

    @ NeuroPulse

    >>
    I thought I understood how this works, and have tried to help explain it. But, now I am confused again.

    When the vehicle is traveling at say 2.8 times wind speed, where does the energy come from to maintain that speed? Energy must be put into the vehicle’s system for it to maintain that speed in the face of air resistance and friction. The energy can not be coming from wind pushing on the vehicle from behind if it is going faster than the wind.
    << You're in good company if this still seems confusing to you. There are a number of ways to approach this. Perhaps the best way is to consider a basic power analysis. As the cart moves along at 2.8X wind speed, the prop accelerates air backward *relative to the cart*. However, this is actually slowing the air down relative to the ground. This means the cart is leaving in its wake a path of lower energy wind. That's where the energy comes from. But it's not a satisfying explanation as to *how* it happens. This may or may not help much there (but I will be happy to offer other approaches until I do make it clear)... If you imagine attaching a generator to the wheels and drive the prop with an electric motor, you can see whether the wheels can produce enough energy to accelerate the vehicle. Common sense might suggest there can't be. But in a tailwind there is. Because the wheels move over the ground much faster than the car moves through the air (remember there's a tailwind), the power generated is greater than the power required at the prop. Power generated = (retarding force of the wheels on the road) x (speed of the cart over the road). Power required = (prop thrust) x (speed of cart through the air). And of course if the prop thrust is greater than the retarding force (plus losses), the vehicle will accelerate. The vehicle is basically a lever trading a small force over a large distance for a larger force over a smaller distance. Does that help?

    1. NeuroPulse says:

      @Rick Cavallaro

      Thanks. I understand that explanation *until* the vehicle exceeds wind speed.

      “remember there’s a tailwind”

      But there is no tailwind relative to the cart when it is traveling faster than the wind. That is why I do not see any source of energy to maintain faster than wind speed.

      Or at least I did not. I just realized something. Because the propeller blades are angled and spinning there is till pressure from the tailwind.

      Imagine a board or blade tilted 45 degrees from the wind and sitting still with a 10mph wind blowing on it. There is 10mph-wind worth of pressure on one side. Now, if you move the board *perpendicular* to the wind, the pressure on the downwind side will decrease, and the pressure on the upwind side will increase. When the board reached 10mph, the pressure on both sides will be equal. So the board has a tailwind, but would not be pushed forward by it! For the same reason, the spinning blades of the propeller have pressure added to the upwind side even though the propeller as a whole has no tailwind! So the energy *is* coming from the tailwind after all! Eureka!

      1. NeuroPulse says:

        “if you move the board *perpendicular* to the wind, the pressure on the downwind side will decrease, and the pressure on the upwind side will increase.”

        I said that backwards. The pressure downwind upwind side will decrease, and the pressure on the upwind side will increase.

      2. cary says:

        So, then, it’s the wind pushing against the air mass the propeller is displacing behind it, that keeps it accelerating, yes?

        1. NeuroPulse says:

          Yes! There is pressure on one side of the blade due to the blade spinning, and that pressure is made greater because there is a tailwind, even though the vehicle is moving faster than the wind. I finally think I understand it.

          I think was has been lacking is a good complete conceptual explanation of exactly what is going on. Maybe I just have not seen it. I have seen conceptual explanations that are inadequate and mathematical explanation that are complete, but people want o understand conceptually exactly why this works. Even if the math proves it, that does not satisfy them or me. It has to make common sense.

    2. cary says:

      what accounts for the increase in speed? Is it the force of the wind against the whole cart, or a Bernoulli effect on the propeller, which is acting like a round sail? Or is it something else (which I can’t cudgel from my brain)? Apologies if this is so far off the mark as to be unanswerable.

  25. ThinAirDesigns says:

    @Greg:

    Greg makes a big deal about what he believes was needed to provide proof (sensors) and wonders why we didn’t do this.

    >>> “Include a few sensors!”

    >>> “…why wouldn’t you invest in some simple sensors to be done with it???”

    When it’s pointed out that there were NALSA monitored sensors all over the freakin’ place, he now says:

    >>> “The number of sensors are not the problem.

    The lack of mathematical proof or presentation of the true physics is the issue.”

    Well Greg — first you clearly said that a few simple sensors and we would be “done with it”. There were 18 sensors and yet somehow we’re not done with it in your mind. In fact, you completely change your tune and tell us that it’s not sensors we need at all, but rather “mathematical proof”.

    So now you’re ignoring BOTH the myriad of sensors that you said we needed (and we have) AND you’re ignoring the mathmatical proofs that you claim we lack yet have ***posted multiple times***.

    Why do you claim we need something and then when we show we have that very thing you switch up and say we need something else? Secondarily, why do you claim we lack things that we have already provided multiple times?

    Like Platt it appears you have classic ‘denier’ syndrome, rather than serious ‘skeptic syndrome’ (which we personally respect greatly)

    JB

    1. ThinAirDesigns says:

      As an add to the above post of mine here is another suggestion/request from Greg:

      @Greg
      >>> “Get an outside non-invested party to evaluate this.”

      Greg, who do you think NALSA was?

      Ok, we have sensors, data, an outside non-invested party, and math/physics proofs from noted sources.

      What shall we get you now Greg?

      JB

      1. cary says:

        If I can take a trip driving the cart, I will believe with all my heart!

        Just joking, the demo, the sensors and the independent evaluation by NALSA are completely convincing, even if I still can’t wrap my head around the physics of the thing. By the way, I don’t think it’s been said enough here, congratulations on a great and very entertaining achievement!

    2. Greg says:

      Again, my first comment on the lack of sensors was in regards to the video. Maybe I wasn’t clear enough in my comment. If there were some real-time sensor displays showing wind speed/vehicle speed while the action was taking place this would help take care of the casual internet viewer’s skepticism.

      Now, for the paper. This is a step above the casual internet viewer. To establish claims with data being presented as support of the claim, you need to be able to back up that data with much more explanation. From what I can determine, Mr. Cavallaro has a background in engineering and should be aware that a technical document making claims (especially on a claim of this significance) should have extensive explanation to satisfy the naysayers. Not links to pictures, not links to other theoretical data. You guys went so far to build a full scale model! This investment in cost/time/emotion deserves an equal investment in the presentation of data.

      I can see that you two are dedicated to making this work, but from what I have seen so far, your strengths are not in the presentation of the data generated in the tests. That is why I suggested another outside party to preform tests/give a presentation of the data.

  26. Jason says:

    That is very cool. I love how just about anyone can go out and do something like this now. I can’t claim to understand how it works, but seeing is believing. Very cool.

    Jason

  27. Rick Cavallaro says:

    @ Cary

    >> I don’t think it’s been said enough here, congratulations on a great and very entertaining achievement!

    Thanks very much.

    @ Greg

    >> your strengths are not in the presentation of the data generated in the tests

    What I’ve presented here is a story. In 1100 words I can’t begin to offer all the data and description that we’ve provided elsewhere – and that you’ll hopefully be able to read in a peer reviewed journal before too long.

    @ Jason

    >> I can’t claim to understand how it works, but seeing is believing. Very cool.

    That’s a healthy scientific attitude. Too many people seem willing to dismiss real-world results because it doesn’t fit their theories. Real scientists take exactly the opposite approach. Thanks.

    1. https://me.yahoo.com/a/UkvC4Mk9sYX7o4NxFBVCXhiOakY-#541ac says:

      I have found this to be a fascinating story over the past year… both in terms of physics and in terms of communication (and psychology).

      I have a BS in physics, and this has often seemed improbable (at least sustained), but I would love to be surprised. The smaller scale vehicle videos were intriguing! Those surprises can be the best parts of life.

      Anyway, in the video above, it was becoming very convincing but then the cart did not maintain a FTTW speed for very long… maybe 15 seconds? Why did the cart and prop slow down so quickly?

      1. ThinAirDesigns says:

        >>Anyway, in the video above, it was becoming very convincing but then the cart did not maintain a FTTW speed for very long… maybe 15 seconds? Why did the cart and prop slow down so quickly?

        The video is not the proof — it is merely one video taken during one run during the collection of the data that provided the proof.

        If one is to accurately compare the speed of the vehicle to the speed of the wind, one must remain within a few hundred feet of instruments which are accurately measuring the wind over the ground.

        While NALSA did record data from 18 sensors installed on the vehicle and lakebed, neither NALSA nor our team had the millions of dollars that would have been required to grid the entire miles square lakebed with anemometers every few hundred feet. (nor was it necessary).

        There was an area of the lakebed that was instrumented. The course was about a mile and a half long. Once we reached the end of the course and were thus out of relevent range of the instruments, the vehicle was braked to a stop.

        It should be noted that if one were to watch a video of street cars running at the local Saturday night drag races, one could also come to the conclusion that these cars were not capable of going 100mph for more than a few seconds — after all … why did they stop so quick?. But we all know different.

        The NALSA rules, instrumentation, tests and observations were totally designed to test the vehicle for *steady state* capabilities. Unfortunately, some feel that the only way to test for “steady state” is to test for forever.

        JB

  28. ThinAirDesigns says:

    @Cary:

    >>> “So, to begin with, the propeller acts like a sail, driving the thing forward through the mast.”

    I’m going to stop right there because before we can get any closer to accurately answer your question, we have to define what the term “acts like a sail”.

    An ordinary sail (even a simple one on a Sunfish or Hobie) functions in three different modes depending on it’s setting.

    A: it can act as a mere ‘bluff body’, creating a force on the boat aligned with the direction of the true wind.

    B: it can act as a turbine — that is an airfoil device which is used to slow the velocity of the air relative to itself. Within certain limitations, the direction of output force can be different than that of the true wind.

    C: it can act as a propeller — that is an airfoil device which is used to accelerate the velocity of the air relative to itself. As in “B”, the direction of output force need not be in the same direction as the true wind.

    (note that “A” is merely a very inefficient and more limited version of “B”)

    Now, to address your “the propeller acts like a sail” comment:

    Initially, the blades of the propeller act in the “A” mode. They aren’t spinning fast enough to come unstalled so they are merely creating bluff body drag same as the rest of the chassis. This bluff drag moves the vehicle downwind at a slow but steadily increasing rate.

    As the RPMs of the propeller increase, the tips of the blades are moving through the air fast enough to come ‘unstalled’. In a 10mph wind, this happens at about 5-7mph ground speed. With the tips no longer stalled they begin to produce thrust (mode “C”) With this steadily building thrust, the vehicle can continue to accelerate even as the force of the bluff drag steadily decreases.

    As the velocity of the vehicle and thus the RPMs of the propeller increase, more and more of the propeller ‘bites’ — that is more and more of the root section of the blades come unstalled and start producing thrust.

    At about wind speed, the entire propeller is fully utilized and creating thrust. This happens by design and not because there is some ‘phase change’ or other happening at wind speed.

    You could design a propeller that would hook up fully at well below wind speed (and then your top speed would be low) or conversely you could design one that didn’t hook up until 2x wind speed (and then you couldn’t take off from a standstill).

    So in short, the propeller starts out in mode “A”, gradually transitions from mode “A” to “C”, but in our implementation never operates in mode “B”

    But in *all* modes, it’s still acting as a sail.

    Hope this helps.

    JB

    1. cary says:

      Yes, that clears things up for me considerably. I appreciate your patience in explaining things in a way that the technically challenged can understand.

      1. Rick Cavallaro says:

        It’s surprising how often it’s the folks that claim to be technically challenged that are actually the ones open-minded enough to carefully consider the explanation and therefore come more quickly to an understanding.

      2. volkemon says:

        Well… I sit here educated and corrected.

        My only exposure to the entire story was from that MAKE magazine article, and I also saw how ‘impossible’ the theory was… Until now.

        @JB-

        Your everlasting patience and ability to ‘dumb it down’ brought it to the level of someone with little mathematical experience! Thanks!!!! I think I ‘Grok’ it :)

        1. Rick Cavallaro says:

          >> Well… I sit here educated and corrected.

          Excellent! I hope it’s a good feeling. That’s what brain teasers are all about.

  29. ThinAirDesigns says:

    @Greg:

    >>>: … but from what I have seen so far, your strengths are not in the presentation of the data generated in the tests.

    We weren’t asked by Make to present data from any tests. Had they asked, we would have taken a pass as it can’t be done in their allotted word limit. Additionally, publishing too much of the technical side can preclude acceptance of said material to technical manuals for peer review.

    Everyone thinks they know what we ‘should do’ or ‘should have done’. Generally those are just statements created by the lack of knowledge of what we were doing or are trying to do.

    Thanks for your suggestions.

    JB

  30. Rick Cavallaro says:

    >> Without sounding crass, you either need to pick up your Nobel Prize…

    I think you have to go to Stockholm to pick those up. I wonder if they’d send one?

    In point of fact we’ve gone to significant lengths to make clear that we’re not demonstrating any undiscovered principle. It’s only the skeptics that seem to claim we are.

  31. Marcel says:

    Well done! I can’t say that loud enough.

    You’ve taken a notion, cogitated it, tested it and succeeded in proving what few thought possible. Even if I and other can’t understand it, you’ve proven it. Well done.

    You’re not alone
    I’ve met, briefly, the local engineer who has fitted a wind turbine on top of a 30″ catamaran. The turbine drives a large water propellor. The craft can make headway in any direction – including directly into wind.

    His work has be bounced around the net and various forums and he has consequently been flamed more times than you can imagine (no, scratch that… ‘he has been flamed just like you’). Just because people can’t get their heads around the principles they think it’s impossible.

    He has chosen to no longer discuss his work online and he continues his research ‘in private’. It’s a shame, but I respect his decision.

    Stick with it guys. Keep at it. Some people will have to eat humble pie one day but may I encourage you to not lose sight of your goal in the process with all the distraction.

    [m]

  32. Jennifer Elaan says:

    I have to admit that I was skeptical at first. Then I ran the vector analysis. Surprisingly simple math, and when I ran it, it clearly said that this should work.

    There are a few key things to remember. First, motion itself doesn’t require energy – an object in motion will remain in motion, neglecting friction. Acceleration requires energy, but if you can extract any energy, you can accelerate.

    The second, and in my opinion, most important is that this isn’t an aircraft. The relative difference in speed between the ground and the wind is the same no matter how fast the craft is traveling. And this difference – not the windspeed itself, but the speed relative to the ground – provides energy that can be extracted, no matter how fast the craft is moving.

    At t0, the propeller is stopped and acts as a sail, moving the vehicle forward. If the propeller was fixed in place (acting as a sail), the vehicle would reach the same velocity as the wind (neglecting losses). At this point, the wheels are rolling under the vehicle, and power can be extracted from them. Doing so will add drag to the craft, of course, making it travel somewhat slower than the wind speed, and causing wind to be slowed by the fixed propeller.

    Now when the propeller is spun by power extracted from the wheels, it provides thrust which further accelerates the craft. This thrust isn’t free, and it doesn’t contribute to the power being extracted from the wheels: the ground speed is increased, but the effective wind speed is also increased, which means that the difference in speed between the ground and the wind remains the same. And again, it is this difference – not the absolute wind speed, or the absolute ground speed – that the craft extracts energy from.

  33. ThinAirDesigns says:

    Nicely put Jennifer. Well done.

    Only one comment:

    >>> “At t0, the propeller is stopped and acts as a sail,…”

    At all speeds the propeller acts as a sail — it’s just that like our propeller, sails operate in different modes in different situations.

    Minor nit. Nice work.

    JB

  34. Rick Cavallaro says:

    @ Jennifer

    > The second, and in my opinion, most important is that this isn’t an aircraft. The relative difference in speed between the ground and the wind is the same no matter how fast the craft is traveling

    True. And interestingly enough, the faster you go over the ground the more of that energy you can collect. For this reason you can harvest several times the power of a stationary windmill of the same diameter.

    > …This thrust isn’t free, and it doesn’t contribute to the power being extracted from the wheels: the ground speed is increased, but the effective wind speed is also increased

    You’re right that the energy density available is constant, but as described above, you can harvest more of it by going faster. Just think about the rate at which you produce a wake of slower moving air.

  35. Chuk says:

    > What I do not believe is that this vehicle can start from rest
    > with the wind behind it

    Any sail on wheels can do this, the prop clearly has surface area to act as a sail.

    > accelerate until it is moving at the same speed as the wind

    Any sail on wheels will reach NEARLY the speed of the wind. At which point there is still a tail wind for the generated wind force to push AGAINST, effectively INCREASING the propulsion force of the generated power, which would carry it beyond friction and efficiency losses ahead of the wind speed.

    > and then continue to accelerate faster than the wind, i.e.
    > into a net headwind

    Once propelled beyond wind speed, it will still accelerate until balancing against the wind resistance of the vehicle.

    Clearly there is a delicate balance of design, weight, wind resistance, prop dimensions, etc. So not “just any” home made model would prove this.

    It seems disbelievers don’t understand how the ratchet of the prop effects the system, creating a sail at standstill, and NOT driving the wheels.

  36. TwoShort says:

    That cart is really cool. I used to sail iceboats, which regularly reach points directly downwind much faster than the wind, so when this discussion came up previously, it never occurred to me to doubt it. It does amaze me how people can be shown demonstrations, have it explained, and still confidently declare it impossible without doing the math. Not that my favorite laymans description of the effect will convince any disbeliever who has read this far, but here it is:
    The cart derives energy from the ground rushing past underneath it and uses some of it to move forward through the air.

  37. Garry says:

    I’m happy to see some people successfully wrapping their heads around this in the comments. I think most of the doubters are missing the contribution of the ground moving relative to the vehicle in addition to the obvious wind moving relative to the vehicle. You couldn’t get this effect with an “air hockey puck” that was floating above the ground, you have to use the work that the ground is doing to turn the wheels that drive the prop.

    The other thing that most people don’t realize when they are trying to “get” how this works from a “common sense” perspective, is how much modern airfoils let you appear to “cheat”. If you push an airplane wing with 1 lb of horizontal force (thrust), it will generate maybe 8 lbs of vertical force (lift). Sort of like a lever can multiply an applied force. Propeller blades are pretty much just wings that are twisted. I’m sure wikipedia has a lot of info if you are curious… My point is that propellers are pretty efficient at generating thrust when you apply a torque to spin them.

    Maybe think of this kind of experiment that has no wind in it… If you laid down some railroad tracks that had notches in them and made a cart to run on those rails that has notched wheels, so there can be no slippage of the wheels on the rails. The notches mesh like gear teeth. Now connect that cart to a big truck that can drag the cart down the track. If I use gears to make the rotation of the cart’s wheels drive a propeller, I get to choose a gear ratio that will govern how many times the prop turns for every time the cart’s wheel turns. Since the wheels can’t slip in this example, I could choose a crazy high gear ratio so that if I can get the cart moving just a little, the prop would spin so fast that the cart will shoot forward and rear-end the truck. To make that happen, the truck has to do a crazy amount of work (pull crazy hard for some distance) to get the cart moving, so there is no free lunch. No laws of physics or thermodynamics are violated.

    The trick with doing this with a) no fancy rails and b) using wind instead of the truck, is to pick a gear ratio that gives you some useful thrust but doesn’t make the wheels so hard to turn that they skid across the ground or anchor you at the starting line.

    If there are competitions to break this record in the future, they will compete by how well they balance propeller efficiency, gear ratio, and mechanical friction to try to win.

  38. Rick Cavallaro says:

    @ chuck

    > I used to sail iceboats, which regularly reach points directly downwind much faster than the wind…

    Yes, but as you know, they can only to this by tacking downwind. Many people that understand that can’t make the jump from ice-boats to our cart which goes downwind faster than the wind without tacking (even with the explanation that the propeller blades are on a continuous downwind tack).

    > The cart derives energy from the ground rushing past underneath it and uses some of it to move forward through the air.

    As analyzed in the frame of the cart that’s exactly right – but MANY people will simply not accept such a heretic possibility.

  39. Rick Cavallaro says:

    @ Garry

    > If you laid down some railroad tracks that had notches in them…

    Unfortunately that experiment could not work steady-state. If there is no wind, the truck would always have to pull the cart. There is no speed at which the cart would try to move forward toward the truck.

    1. Garry says:

      Yes, to extend my example to steady state you would be stretching it too far. You’d have to have an infinitely long, massless rope and a variable tension winch or something. I was just trying to illustrate one part: how the ground turning the wheels does added, useful work, and that work is leveraged by a propeller. It is hard to explain this whole system to people who deny the validity of using the treadmill… Maybe an example that referenced “drafting” in NASCAR? A lot of people believe in that without really understanding it, so maybe it could be extended, as a concept, to help explain this… I’ll give it more thought.

      1. Garry says:

        Here is a better example of what I tried to illustrate with my truck pull example:

        http://www.youtube.com/watch?v=E7vcQcIaWSQ

  40. Rick Cavallaro says:

    @ Chuk

    > …At which point there is still a tail wind for the generated wind force to push AGAINST…

    Actually, this is a very common misconception. The prop doesn’t push against a tailwind any more than the prop on an airplane (flying in any direction) is pushing against a tailwind. It’s a very tempting analogy, but the reality is that the propeller is simply working its way through an airmass that happens to be moving in the same direction as the cart.

    Because the airmass is moving in the same direction as the cart, the prop has to do less work than the wheels harness from the ground rushing by at the higher relative speed. Make sense?

    1. Chuk says:

      @Rick

      Yes, that makes sense to me… however I was merely trying to phrase it in a way that would make sense to the disbelievers. Once every one is on board with it being possible, we can discuss how it works more accurately :)

      This will be even more fun when they go upwind next year!

  41. cdreid says:

    i believe your problem isnt in your engineering or science but in your ability to communicate the mechanism. Another poster asked you this but please explain it as i cannot find your plans to figure it out myself.

    This is what i get from your rather cryptic comments. The prop is never transmits mechanical energy to the wheels. But the wheels transmit energy to the prop. When the vehicle nears windspeed the prop, if it were simply a freewheeling sail, would be unable to propel the cart any faster as the energy it recieved would equalise with the resistance it met. AT that time the wheels use the energy they recieve from spinning to transmit energy to the prop increasing its speed and thus generating more thrust allowing for increased speed.

    If this is so it does indeed fit within known physics and is like all innovation a “cheat” to bypass what we Think are physics constraints. In that case i can think of only 2 logical reasons for this to work.

    1. You are in effect using the prop to transfer energy to the vehicle to store then using that inertial energy to accelerate for a truthfully limited amount of time (until the stored energy runs out). This would be the equivalent of spending the energy in a flywheel to accelerate.
    2. The energy the prop harvests is actually greater than the resistant force on the front of the cart. In effect using the fact that the cart is more aerodynamic from one direction than the other. This, especially combined with one, makes sense to me on a theoretical level (but ironically less so on an engineering/pragmatic level).

    Is this correct? If not please dont address these thoughts but rather explain the mechanism involved. I take it the wheels ARE used to provide power to the prop via mechanical means?

    Again i think the problem is in your explanation (and why are the plans so hard to find can you post a link?)

    1. ThinAirDesigns says:

      cdried:
      >>>why are the plans so hard to find can you post a link.

      The links to the build videos are given in the above publishd MAKE article. It’s a bit difficult to believe that makes them hard to find.

      JB

      1. cdreid says:

        Some reason youre being a jerk? obviously i didnt see them but ill look harder.

        1. ThinAirDesigns says:

          I don’t believe that when you make a mistake and I point it out, that makes me a jerk. But that’s just my opinion.

          JB

  42. ThinAirDesigns says:

    cdried:

    The mechanism is extremely simple:

    A garden variety propeller (of appropriate pitch and twist) is geared to the wheels through a fixed ratio transmission (of appropriate ratio).

    On the Blackbird, this transmission is merely a twisted chain running from a sprocket on the drive axle to a sprocket on the prop shaft.

    On the small vehicle shown in the build videos, the chain is replaced by a pair of mating gears forming a simple and fixed ratio right angle drive

    In either case, when the wheels of the vehicle turn in the direction needed to move the vehicle fowards, the gearing forces the propeller to move in the direction appropriate for producing forward thrust.

    That’s it.

    JB

  43. Rick Cavallaro says:

    @ cdreid

    >> i think the problem is in your explanation

    I rather pride myself on my ability to explain difficult concepts in ways that make it easier for people to understand. I hope you can accept that I can only do the explaining – and not understanding. There are at least a dozen different ways we’ve come up with to accurately explain this thing. Different explanations work for different people. You have to meet me half way.

    You have to understand as well that I conceived of this thing specifically as a brain-teaser. It’s *supposed* to seem wrong – and it’s done a pretty fair job in that regard (note: we’re aware that I was not the first to conceive of such a device).

    >> You are in effect using the prop to transfer energy to the vehicle to store then using that inertial energy to accelerate for a truthfully limited amount of time …

    No. The vehicle works steady-state. It never makes use of stored energy.

    >> …In effect using the fact that the cart is more aerodynamic from one direction than the other.

    No. The critical aspect is that the vehicle is exploiting the energy available at the ground/air interface – slowing down that relative velocity and thus making use of that energy.

    >>I take it the wheels ARE used to provide power to the prop via mechanical means?

    Yes. They are effectively geared directly to the prop. However, we use ratchets to allow the prop to freewheel so that it literally can’t put any of its stored kinetic energy back into the cart by driving the wheels.

    If you’ll refer to my explanation and tell me specifically where you’re having trouble, I’ll be more than happy to help clarify things. Alternatively, if you would prefer a different sort of explanation, please tell me which explanation you’re not liking, and I’ll take a completely different approach. If you approach this with an open mind and a little patience I’m nearly certain I can help you understand it.

  44. Prang says:

    That a real nice story you are telling, too bad none of it makes a bit of sense! Any wheel rolling on the ground requires a source of energy to overcome the rolling friction. Any propeller requires a source of energy to make it spin and move into a relative headwind. The wind is the only source of energy and once the cart reaches the speed of the wind, that source of energy disappears as the force of the wind disappears. You have not been able to explain any other source of energy as it sure as hell isn’t the “ground moving under the cart”. Like I said earlier, put the cart in a wind tunnel and present some mathematics to back up your claim. Charles Platt is and was right about this from the start.

    1. ThinAirDesigns says:

      @prang:

      >> “… present some mathematics to back
      up your claim.

      Rather than ask for something that we’ve presented over and over, why don’t you explain to us what’s wrong with the math we have presented.

      Once again I’ll post two very comprehensive math/phsyics treatements by renoun MIT professor Mark Drela:

      http://www.boatdesign.net/forums/attachments/propulsion/28167d1231128492-ddwfttw-directly-downwind-faster-than-wind-ddw2.pdf

      http://www.boatdesign.net/forums/attachments/propulsion/28168d1231128492-ddwfttw-directly-downwind-faster-than-wind-ddwe.pdf

      You will note that Dr Drela finds your conclusion to be compeletely incorrect and he states that a vehicle like ours can achieve steady state DDWFTTW “without too much difficulty”.

      Please share whatever it is that you and Platt know that one of the most accomplished and respected aerodynamicists in the world does not know.

      Again, please produce the flaw in the math treatments that have been posted repeatedly before asking again for math treatements.

      Thanks

      JB

      1. Prang says:

        @ThinAirDesigns, who wrote:
        “Once again I’ll post two very comprehensive math/phsyics treatements by renoun MIT professor Mark Drela”

        I note two things about these “papers” of Dr. Drela. For one, they are not published anywhere, but only appear to have been posted in Internet forums. Two, they are not peer reviewed, which must of course be the case since they are not published! For all I know they could be a practical joke, which is what I suspect this entire cart story is.

        1. Super Dave Osbourne says:

          http://web.mit.edu/aeroastro/people/drela.html

          Do you have a webpage on the MIT.edu domain, or any other reputable site with professional credentials relevant to this discussion? Please try being respectful and humble, it would help you quite a bit. Right now you come across as a bitter old avian Whine-a-be, bitter and spiteful.

  45. Rick Cavallaro says:

    @ Prang

    >> That a real nice story you are telling, too bad none of it makes a bit of sense!

    You need to end that sentence with “…to me”. It clearly makes sense to lot of the folks here. But more importantly it seems to make sense in the real world where the evidence speaks for itself.

    >> The wind is the only source of energy…

    Energy is frame dependent. It’s not an intrinsic property like mass. Measured in the frame of the cart moving steady-state, the ground is the only source of energy. But it might be best not to go too far down that road at this point.

    >> You have not been able to explain any other source of energy as it sure as hell isn’t the “ground moving under the cart”.

    To be more precise, you have not been able to understand the explanation. From the frame of the cart the source of energy most definitely is the ground moving under the cart. If you go flying off your motorcycle at 75 mph I think the concept will become startlingly clear to you as the road tears your skin off.

    >> Like I said earlier, put the cart in a wind tunnel and present some mathematics to back up your claim.

    I’ve done better than that on both counts. I’ve put carts on treadmills and operated them outdoors on dry lake beds to establish a very well witnessed world record in front of the established record ratifying body for such vehicles. I’ve also posted the math and many carefully constructed explanations. This article is not the place to lay these things out in any detail.

    If you understood how wind tunnels work, you’d realize that a downwind vehicle would zip past the test section in a fraction of a second – giving no useful data. That’s why we use a treadmill to test downwind vehicles under controlled conditions.

    >> Charles Platt is and was right about this from the start.

    You and Platt are entitled to your own opinions – but not your own facts.

    1. Prang says:

      @ Rick Cavallaro who wrote:

      “From the frame of the cart the source of energy most definitely is the ground moving under the cart. If you go flying off your motorcycle at 75 mph I think the concept will become startlingly clear to you as the road tears your skin off.”

      WoW! So it is the “ground energy” that tears the cyclist’s skin off? And here I thought all along that the rider flying off the motorcycle has kinetic energy with respect to the ground, thanks to that internal combustion engine that has been powering the bike the whole time. Gee and I thought that when he hits the ground it is the friction that tears his skin off and brings him to a stop. Now that we have learned that the ground is a source of energy we should make motorcycles that run on that ground energy and do away with all those gas guzzling internal combustion engines. Then you can collect your Nobel Prize!

      1. Rick Cavallaro says:

        Sarcasm is of little use in scientific analysis. Less when you fail to understand the nature of the quantity you’re discussing.

        If you’re moving along at 700 mph and you’re hit by a bullet that’s moving at 710 mph, do you suppose that bullet will kill you? No? That’s because the bullet has very little momentum and very little kinetic energy in the frame of your moving body. That is how momentum and kinetic energy work. They are not intrinsic properties. They are frame dependent. And when you perform a momentum or energy analysis you have to treat them that way.

        As you may be aware there is no such thing as absolute motion. The ground at the equator happens to be moving along at about 1000 mph as the planet rotates. It’s going much faster still as the planet orbits the Sun, etc. etc.

        You see the ground as “stationary” because you happen to live on it.

        It was not intended to be within the scope of this article to describe this background in physics. I hope you can understand that I can hardly offer a couple of physics courses and a couple of aero courses in this article as prelude to the story I was invited to submit.

        Your sarcasm is misplaced. Rule of thumb: When using sarcasm – be correct.

        1. B_Petschel says:

          Are you claiming that converting the kinetic energy of the rolling vehicle into rotational energy for the propeller will result in sustainable thrust?

          1. Rick Cavallaro says:

            No. This vehicle exploits the energy available at the air/ground interface. Energy that’s available specifically due to the relative difference in velocity of those two media. By using a design that has the effect of reducing that velocity difference we can use the energy harnessed to go in *any* direction faster than either of those media.

          2. B_Petschel says:

            No. This vehicle exploits the energy available at the air/ground interface. Energy that’s available specifically due to the relative difference in velocity of those two media. By using a design that has the effect of reducing that velocity difference we can use the energy harnessed to go in *any* direction faster than either of those media

            I am not mocking you when I ask this….

            When you refer to the relative velocity difference between the two media are you refering to:

            1) The difference in velocity between the air and the ground

            or

            2) The difference in velocity between the circular plane of your turbine and the wind.

            or something different entirely?

          3. Rick Cavallaro says:

            I’m referring to the difference in velocity between the air and the ground. As the cart moves over the ground is leaves a path or wake of wind that’s been slowed. That energy is used by the cart to go faster than the wind.

          4. B_Petschel says:

            Ok, this may be another instance when you appear to be differing from my understanding of how energy capture from wind works. In every case the “wake” is down wind and not up wind.

          5. Rick Cavallaro says:

            Imagine a car going 60 mph due south in a wind that’s going 30 mph due south. It’s wake will be downwind of the car in the vehicle’s frame and upwind of the car in the earth’s frame. This is exactly that situation. The wake is behind our vehicle.

        2. Doobster says:

          @Rick:

          >> As you may be aware there is no such thing as absolute motion. The ground at the equator happens to be moving along at about 1000 mph as the planet rotates. It’s going much faster still as the planet orbits the Sun, etc. etc.

          >>You see the ground as “stationary” because you happen to live on it.

          I believe what you’ve said here is misleading at best. The ground at the Equator does move at ~ 1000 MPH (remember, this is rotational speed we’re talking about) and surely faster than the ground in the Northern hemisphere. Does that mean that the machine will work better at the Equator than in North America?

          The ground is stationary indeed to us and everything on this planet, including the atmosphere. Wind power does not come from the rotation of the Earth against a stationary atmosphere (otherwise we’d have 1000 MPH winds at the Equator). Wind is produced because of pressure differences in the atmosphere.

          Doobster

          1. Rick Cavallaro says:

            >> I believe what you’ve said here is misleading at best.

            I hope it’s not misleading, and it’s definitely not wrong.

            >> The ground at the Equator does move at ~ 1000 MPH (remember, this is rotational speed we’re talking about).

            On the contrary. The earth *does* rotate. But the “true” velocity of any point is not defined. We can always take it’s instantaneous velocity to be our stationary reference.

            >> Does that mean that the machine will work better at the Equator than in North America?

            Nope.

      2. vf says:

        @ Prang

        Kinetic energy depends on the reference frame, so in frames where the ground moves, the ground can be the energy source.

        In the ground frame however it is still the air that provides the energy, while it itself gets slowed down by the cart.

  46. vf says:

    Charles Platt: “Again, what I do not believe is that his vehicle or any other can start with a steady wind behind it, accelerate to a speed equal to that of the wind, and then continue to accelerate so that it is moving faster than the wind, in one uninterrupted process.”

    Yet, this is exactly what happens in the first 3 videos linked just above Platt’s statement. :-D

  47. vf says:

    edgertronics:
    “What you describe is infact the basis of many perpetual motion machines.”

    Yes, many PMM are based on a feedback loop. But this doesn’t mean that every feedback loop is a PMM. Note that this is not a closed loop: The *true* wind is slowed down by the vehicle giving it’s energy into the loop. No overunity or perpetual motion here.

    Ironically, *your own* explanation of DDWFTTW constitutes overunity:

    edgertronics:
    “Like a wind turbine extracts kinetic energy from oncoming wind, the propeller (when the cart is moving faster than the downwind) will act as a turbine and should drive the wheels in order to take the cart forward.”

    Such a turbine that goes DDWFTTW would push the air forwards and therefore accelerate the *true* wind. The cart would create more wind out of nothing, instead of harvesting wind energy.

  48. ThinAirDesigns says:

    Prang says we need to produce the math. For the umpteenth time and from a notable source we do exactly that. Prang responds by saying that such papers are for “lining bird cages”. Prang says the paper could be a hoax and it’s not peer reviewed.

    Prang, there is a really good way to figure out if it’s even possibly a hoax — find the flaw in the math in the paper. If the math and the included assumptions are correct (and they are), it puts your ‘hoax’ theory beyond thin ice.

    I suspect what we have here with you is what we see commonly — someone insisting we “produce the mathematics” when they can’t do or check the math.

    In those papers from Drela there is no breakthrough math nor edgy physics principles that require peer review. What’s next — peer review required for the ‘equivalence of inertial frames’ principle? Everything in those papers can be reviewed and double checked by anyone with an undergrad level education (at most) in math, physics and aero.

    Before you line a bird cage with a perfectly valid math treatment, review it yourself and report back. You asked for the mathematics — the least you can do is show us that your request meant something.

    JB

  49. vf says:

    ThinAirDesigns: “Prang says the paper could be a hoax and it’s not peer reviewed.”

    Here is a paper that was peer reviewed and accepted at the EWEC2009 conference:
    http://orbit.dtu.dk/getResource?recordId=241183&objectId=2&versionId=1

    Quote from conclusions: “It is theoretically possible to build a wind driven car that can go in the downwind direction faster than the free stream wind speed (using a propeller in the air)”

    1. ThinAirDesigns says:

      @VF:
      >>>
      Here is a paper that was peer reviewed and accepted at the EWEC2009 conference:
      http://orbit.dtu.dk/getResource?recordId=241183&objectId=2&versionId=1

      Quote from conclusions: “It is theoretically possible to build a wind driven car that can go in the downwind direction faster than the free stream wind speed (using a propeller in the air)”
      >>>

      Well sure — you’ve now produced the *peer reviewed* math that Prang wants, but how do we know the entire peer review process isn’t also just a another hoax? (rolling eyes)

      If prang drove the Blackbird DDW and felt the 20mph wind in the face, he’d still claim it was a hoax and want something different.

  50. Rick Cavallaro says:

    >> Here is a paper that was peer reviewed and accepted at the EWEC2009 conference…

    Interestingly, that paper came about when the primary author learned of our project. He is heavily involved in the Aeolus upwind races in Denmark. It had not occurred to him that wind powered vehicles could actually go downwind faster than the wind. But he’s a pretty sharp guy, so it didn’t take him long at all to work out the math. He’s also given a talk on the topic that can be seen on YouTube.

  51. Subduction Zone says:

    Here is the video by MrMacgaunaa on YouTube. It is part 5 0f 6:
    http://www.youtube.com/watch?v=qssCU_GeU3g

    The whole lecture is worth watching. And to the naysayers, he has no problem using treadmill to demonstrate this. It has been 30 years since I have done any serious math so my differential equations work is not quite up to snuff, but I can still do basic one dimensional vector addition. That is all that it takes to do the Galilean Transformation required to show the equivalence of the cart on the treadmill to running on the road at wind speed. If it advances towards the front of the treadmill it is going faster than the wind. In fact it is going faster than the wind relative to the floor in the room. The speed of the wind relative to the floor is zero in the room with the treadmill, the cart is going faster than that.

  52. asymptotic says:

    About the treadmill – almost 400 years ago, Galileo Galilei understood something very surprising (well, it was very surprising 400 years ago). He realized that the laws of physics are unchanged when you add a constant velocity vector to everything in the system. In other words, physics is invariant under Galilean transformations, it’s the same in all velocity frames. (For culture, he was not quite right – Einstein and Lorentz improved this to Lorentz transformations, but those are nearly identical to Galilean transforms at wind cart speeds.)

    Take the treadmill in a room. It has a surface that is moving to the west, air that is still. Now change frames to a frame that’s moving along with the surface of the treadmill. In that frame, there is a wind blowing east, and the surface of the treadmill is stationary. Exactly like being outside on stationary ground in a wind blowing east.

    If the cart can maintain its position on the treadmill, it’s advancing downwind at the speed of the wind. If it can advance against the treadmill (move east in the original frame, the air rest frame), it is going downwind faster than the wind.

    This is actually a **better** demonstration of ddwfttw than the full-scale model outdoors was, because it’s a very well controlled environment with a perfectly steady wind. If the cart advances it proves beyond any doubt that it is possible. And anyone that wants to can easily do the demonstration for themselves by spending about $40 and following spork’s very detailed build videos.

    The fact that there was any debate over this after the treadmill demos just goes to show that some people still think about physics the way a medieval peasant would have.

  53. yootskah says:

    Great work on this Rick and JB, this must have been one heck of a fun project to work on.

    I think that this is kind of hard for people to accept because it really is quite different from how we generally see wind power harvested, and this simple disconnect makes it difficult to wrap your head around.

    If it helps anyone, here is how I tried visualizing it:

    Imagine the cart is floating, like a balloon, in stationary air. It is not moving in any direction. There is no ground, nothing besides the air.

    A wind comes up and pushes the air past the cart. Since the cart is buoyant, it starts accelerating in the direction of the wind, eventually matching it (we’re ignoring losses here). In this case, the propeller is not rotating, the whole cart is just floating along like a balloon. So in the frame of reference of the cart, as long as the wind remains constant, essentially nothing is moving. It is now completely still, floating in the still air. Just as it was before.

    Now, lets add the ground to the equation. The ground is moving backwards, relative to the cart, at the same speed as the wind. So, the cart can see that it is moving relative to the ground, but the air still feels motionless. Now, if the ground is pushed up into the wheels of the cart, it forces the wheels to turn, and consequently, the propeller. Which then pushes on the air and propels the cart forward.

    Basically, in the frame of reference of the cart, the moving ground is the source of the energy. It’s as if a giant treadmill has been placed underneath it, supplying energy to spin the wheels, which in turns spins the propeller and pushes the cart forward. This is the source of energy.

    So, it’s only to us, standing still on the ground, that this seems confusing. To the cart, it makes perfect sense. To it, there is a giant treadmill spinning its wheels, which spin it’s propeller, which makes it move forward through the still air.

    1. cdreid says:

      as that of JB.

      I understand this based on PURE PHYSICS. But i think a lot of people have a problem either making the physics click in their head.. OR the physical engineering click in their heads. I was in camp one then in camp 2.

      What is apparently happening is that the cart (think of it as a generator) is harvesting the wind energy.

      The only way i can make this work in my head is to dispense with ideas about “sails” and “propellers” etc and think of the cart as a wind turbine. Forget motion completely. Medium A (wind) is travelling at velocity x+1. Medium B (the earth) is travelling at velocity X. You harvest energy in the velocity difference. You then apply that energy to motion.. the means are irrelevant.. you could use an electric motor. That is the Energy.
      Mathematically this should work and should be simple to understand. YOu’re simply harvesting an energy difference and converting it to motion. It is the Engineering of that generator that is confusing all of us. Mathematically only friction and momentum should limit the speed of the cart.

      As you approach the speed of the wind you switch generators (and this is the hard part to fathom) and rather than the prop harvesting the energy the wheels begin to harvest the energy difference. Maybe its easier to think of the CART as a SAIL (ignoring the propeller). Then once the cart/sail is almost at the speed of the wind.. the wheels harvest the energy difference and use the propellers to push against the wind. Again, to me the math/physics makes perfect sense. The engineering boggles the mind though.

      1. Rick Cavallaro says:

        >> As you approach the speed of the wind you switch generators (and this is the hard part to fathom) and rather than the prop harvesting the energy the wheels begin to harvest the energy difference.

        Your energy approach is accurate. But we don’t have to think of the cart harvesting energy from the ground OR the wind. Like a watermelon seed squeezed between my thumb and finger, the cart is being squeezed between the ground and air. Remove either one and we got nothing.

      2. Alsee says:

        You post was good except for the word ‘turbine’. This vehicle never operates the propeller as a turbine. This is an extremely important point because a huge amount of confusion comes from people endlessly imagining the prop operating in a turbine mode. Turbine thinking turns everything backwards and leads to a dead end.

  54. ThinAirDesigns says:

    @B. Petcshel
    >> “It is difficult to talk about a concept that explicitly violates Newton’s Second Law and conservation of energy”

    Slowing down the air relative to the ground (otherwise called slowing down the wind) and using the recovered KE to power devices has been done for centuries and then some without violating any natural laws. There’s no reason whatsoever it needs to start the violations now.

    >> “-In absolutely still wind conditions pull the car up to 5 to 10 mph.”

    Nothing in our claim nor nothing about our device would lead to a rational conclusion that it will work when there is no relative motion between the air and the rolling surface (otherwise called wind). Our device is powered only by the KE harvested from slowing this relative motion. If this motion doesn’t exist to start with (otherwise called ‘no wind), there is no KE available to harvest.

    >>Are you claiming that converting the kinetic energy of the rolling vehicle into rotational energy for the propeller will result in sustainable thrust?

    Nowhere and not once have we claimed anything resembling any such thing.

    You need to understand the claims before leveling criticism. Currently you’re merely building strawmen and setting them on fire. It’s not terribly productive for your case.

    1. B_Petschel says:

      >> “It is difficult to talk about a concept that explicitly violates Newton’s Second Law and conservation of energy”

      Slowing down the air relative to the ground (otherwise called slowing down the wind) and using the recovered KE to power devices has been done for centuries and then some without violating any natural laws. There’s no reason whatsoever it needs to start the violations now.

      I understand how a wind turbine works. What I balk at is this: How do you recover energy from wind by slowing it down when there is no relative motion difference from which to extract energy?

      In the most idealized model of your system you will reach a point where your forward velocity is equal to the wind velocity. At that point the wind will impart no further energy into the system. The only energy in the system is the KE of the cart and the rotational energy of your turbine.

      Where am I getting this wrong?

      1. ThinAirDesigns says:

        @B_Petschel

        >> “I understand how a wind turbine works.”

        A turbine is but *one* mechanism capable of slowing down the wind. It also happens to be the one way that the Blackbird does *not* use.

        >> “Where am I getting this wrong?”

        Simple. You keep assuming that when we say we are slowing down the wind, we are using a turbine — we are not.

        A turbine harvests energy by slowing down the relative motion between the air and *itself*. This means that a turbine has a nasty characteristic (for our application) such that when there is no wind blowing through it’s blades disk, it comes to a stop.

        All of the thing that you say about us and the vehicle (Nobel if it worked, violates natural laws, should work in no wind, etc.) would be absolutely true *IF* we were using a turbine to slow the air and *IF* using that turbine it were still able to go DDWFTTW (it wouldnt’). BUT WE’RE NOT USING A TURBINE!!

        So, I ask you – no snark and our past interactions forgotten … Do you really want to know how it works? Will you leave your assumptions at the door and at least *listen* and answer questions as we walk through it together? I promise you I will listen to you and answer any of your questions in return.

        If we can have that above deal, I’m happy to take you through it and you might well be surprised at the results.

        What’da say?

        JB

      2. Subduction Zone says:

        You are getting it wrong when you say when the cart is moving at wind speed there is no relative movement. Both the cart and the wind are moving relative to the ground when the cart is at wind speed. The propeller is still turning, you will at least admit that, since it is driven by the wheels. What happens to the air when a propeller spins in “still air”? It is accelerated away from the propeller, the direction determined by its angle of attack and direction of spin. If you look at the cart you will see that the air is accelerated to the rear. If the air that is originally not moving relative to the cart when it is traveling at wind speed is accelerated to the rear its velocity relative to the ground is decreased. Relative to the ground the affected air ha less energy after it passes through the prop than it had before. Where did that energy go to? For the cart to work the losses from friction has to be less than the loss of energy by the wind.

      3. vf says:

        @B_Petschel

        >> How do you recover energy from wind by slowing it down when there is no relative motion difference from which to extract energy?

        If there is a *true* wind, then there is always a velocity difference between ground and air, no matter how fast you move relative to the two media. You can always reduce that velocity difference by levering the two media against each other, to extract wind energy.

        Here an animation that shows the lever analogy:
        http://www.youtube.com/watch?v=g8bxXRQtcMY

  55. Rick Cavallaro says:

    @ yootskah

    Thanks for your explanation. Perhaps that will help some of the folks that are struggling with the equivalence of inertial frames.

    But you bring up an interesting point – and one we debated for some time. If you have a balloon floating weightless in still air when a gust comes along, the balloon does not lag the gust. Perhaps surprisingly, it tracks the gust almost perfectly so there is no wind relative to the balloon even during what should seem like a transition period.

    Here’s an experiment I conned the wife into helping me with:

    http://www.youtube.com/watch?v=johyzKImRTw

    1. B_Petschel says:

      “If you have a balloon floating weightless in still air when a gust comes along, the balloon does not lag the gust. Perhaps surprisingly, it tracks the gust almost perfectly so there is no wind relative to the balloon even during what should seem like a transition period.”

      That is only because the mass of the balloon is really low relative to the forces acting upon it. There is a lag and it will become more apparent as the balloon gets bigger. An extreme case would be a blimp.

    2. yootskah says:

      I hadn’t really thought of that, but it makes sense. If the balloon is neutrally buoyant, it would move just as the air around it does. Like a piece of flotsam in the ocean bobbing in the currents.

      I guess in my mind experiment I left the cart as an inertial mass that had to be accelerated by the wind, not with it, to keep it more in line with reality.

      Ah, the fun that can be had with physics. If only science classes posed questions such as this, along with the opportunity to build them, I would think the state of our math & science education would be so much better.

      Really great job guys, and if anything, the controversy surrounding this has only brought more attention, thereby educating that many more people. I’m sure your team has inspired a few young minds out there. Well done. My hat is doffed.

      1. Rick Cavallaro says:

        It’s a bit of a mind twister itself. I wouldn’t have subjected my lovely wife to that silly science experiment if it were obvious.

        And yes, I really do wish they posed problems like this for engineering students (and perhaps more so for PhD’s). We think it’s important to remind people not to substitute intuition for science.

        Thanks for the kind comments.

  56. Rick Cavallaro says:

    @ B_Petschel

    No, it doesn’t just “appear” that way. It *works* that way. The balloon is neutrally buoyant. If it were lighter than air it would lead. Heavier than air and it will lag. Same density of air and it will track.

  57. Subduction Zone says:

    It only makes sense that a neutrally buoyant balloon tracks a gust exactly. The mass of the gas and the balloon in the neutrally buoyant balloon are the same as the mass of the air of the volume that it occupies. Answer me this, Why would two identical masses subject to an identical force accelerate differently?

  58. B_Petschel says:

    I must apologize to Rick and Company. I get it now. I went hunting and finally found an explanation that works. Skepticblog has a post by Brian Dunning on the subject: http://skepticblog.org/2010/05/27/sailing-directly-downwind%E2%80%A6-faster-than-the-wind/

    His explanation and the first couple comments on the page make it crystal clear how this works. I feel like an idiot for not seeing it the first time.

    I, and most people who appear to be repulsed by this concept at first(from reading other forums), appear to be making the same mistake. When I first imagined the model of this device I assumed the blades were pitched as to extract energy from the air in two ways:

    1) The turbine rotates transferring energy to the wheels and thus forward motion.

    2) As the turbines rotates faster it provides a pressure differential which pushes the car down wind…. A sail in other words.

    Like an idiot I didn’t look at the orientation of the blades. It is always in “propeller mode”. My previous assumptions were all wrong.

    The physics that make this work(If I am repeating anyone, I am sorry. I am working this out in my head as a type):

    1. One wants to assume that the only forces at work here is the pressure from the wind and the drag. In reality, because the blades are pushing air away from the car AT the wind you have a higher net force than the wind pressure alone.

    2. One wants to assume that the energy being captured by the car for forward motion is only a function of the area (the car’s structures and blade rotational area) and the velocity of the wind. Thus, one would expect that as the “relative” velocity of the car approaches the wind velocity the energy going into the car is decreasing. In short: “fixed area… fixed energy capture potential”

    This part is huge and kind of a guess on my part: Because the propeller is pushing air towards the wind it is creating a high pressure turbulent pocket that maybe increasing the effective area of the craft, and consequently the available energy to be captured. Again, this is just a theory. However it would seem to answer the question why a craft could be getting more energy than the apparent velocity-density-area analysis indicates should be available.

    Great job on the proof of concept. Sorry for the initial attitude. I have had to explain one too many times why you can’t run your car on water or why you can’t attach a generator to the wheel of your hybrid and drive forever. I am getting too cynical.

    1. vf says:

      >> Like an idiot I didn’t look at the orientation of the blades. It is always in “propeller mode”

      Please don’t call yourself an idiot for being stuck at this for a day. After 3 years Charles Platt still thinks it must be a turbine, so you have to “reverse the gearing”, “switch modes” or whatever at wind speed, when the apparent wind changes direction.

      By insulting yourself, you insult him 1000 time more. This will be not tolerated here!

  59. Rick Cavallaro says:

    @ B_Petschel

    >> I must apologize to Rick and Company. I get it now…

    Excellent! And thanks for the apology. There are VERY few who have managed an apology without an attack to go along with it. Well done.

    As to the SkepticBlog, Dunning actually doesn’t get it quite right. He has the prop turning the wheels. You can see my posts on the blog as “spork”.

    Incidentally, we have and did try our best to make clear the points that you now understand. We are never trying to be coy or make this hard to understand. Unfortunately, it often takes a real wake-up call before people are willing to read our descriptions without the preconceived bias that it simply must be wrong.

    I’m glad you get it. You are now indoctrinated as one of the “hoaxters”. I predict you may learn our frustration as you try and convince others now.

    1. B_Petschel says:

      I know Dunning got it slightly wrong but he had a few points that sent me in the right direction. That is why I included my audit of the flaws in my original assumptions.

      This is a problem that is perfectly designed to trip up those who are very familiar with physics. When I read other peoples posts on every site I have visited they are making the same mistake as me.

  60. Prang says:

    @ Subduction Zone, who wrote:

    “If you look at the cart you will see that the air is accelerated to the rear. If the air that is originally not moving relative to the cart when it is traveling at wind speed is accelerated to the rear its velocity relative to the ground is decreased. Relative to the ground the affected air ha less energy after it passes through the prop than it had before. Where did that energy go to? For the cart to work the losses from friction has to be less than the loss of energy by the wind.”

    This is just wrong. As the cart is pushed towards wind speed by the force of the wind, it is acquiring all of its kinetic energy from the wind. By forcing the wheels to turn the propeller, the cart has convert some of that kinetic energy into the rotational energy of the wheels and the propeller. The propeller will produce some thrust but that thrust can never give the cart more kinetic energy than the cart has used to spin the propeller and the wheels. The cart actually loses some net kinetic energy due to spinning the propeller, in accordance with the well tested principle of conservation of energy. You ask where does the energy of the slowed wind go? Some of it is converted to heat in the air turbulence and friction with the prop blades, and some of it does go into the cart as kinetic energy. However, the cart has used up far more kinetic energy in spinning the propeller than it ever gets back from the propeller thrust. In order for the cart to work as claimed it would need to spend x number of joules of energy to spin the propeller, and get back more than x joules in return. This is clearly impossible as anyone who has studied elementary principles of physics can verify.

    1. B_Petschel says:

      My Turn….

      “This is clearly impossible as anyone who has studied elementary principles of physics can verify.”

      It only appears that way. If you are serious about the physics then start with the forces and work from there.

      There is a feedback loop that looks highly suspect at first. When the propeller pushes air towards the wind, you effectively sum the forces to get the net effect on the car. This will in-turn increase the speed of the car thus further increasing the thrust…..This will immediately make you think its BS. However, it doesn’t run away forever like a PMM. The car will accelerate until the drag forces reach equilibrium with the thrust (the sum total of the propeller thrust and the force due to the wind).

      The real eye opener is the third or forth comment on Dunning’s page. It is all about the relative wind velocity.

      1. Rick Cavallaro says:

        >> When the propeller pushes air towards the wind, you effectively sum the forces to get the net effect on the car.

        Unfortunately, this is a very common misconception. The prop is pulling itself through a volume of air that happens to be moving. This brings it closer to the static thrust case, but there really is no “air cushion”, “convergence of flows”, summing of forces. The flow and analysis is exactly the same as for a prop on an airplane that just happens to be flying downwind (or upwind or in no wind).

    2. vf says:

      Here some hints:

      – Energy is frame dependent, so you have to specify the frame when you talk about energy. Otherwise your statements are meaningless.

      – Forces determine if the cart accelerates. Force is not energy. Force is not conserved, so the braking force(that spins the wheels) can be smaller than the propeller thrust, if the ground-relative velocity is greater than the air-relative velocity. This is called leverage.

  61. ThinAirDesigns says:

    @B_Petschel

    Glad to see your Uturn and apology accepted. It should be noted however that the explanation that convinced you is absolutely and fundamentally wrong.

    Here is the relevent quote from Dunning’s explanation:
    “However, when you change things up by replacing the sail with a windmill that drives wheels, it is possible to go directly downwind faster than the wind.”

    This is not true at all. A vehicle which is equipped with a windmill driving the wheels CAN NOT travel DDWFTTW.

    Dunnings incorrect explanation reminds me of a statement we have grown fond of relating to this device:

    “99% of the people say it’s impossible. 99% of the rest don’t know what the hell they are talking about”

    What this says is that even those who believe it’s possible generally can’t correctly tell you how it actually works.

    While due to our efforts and the results of our tests and demonstrations, that initial 99% has been reduced dramatically, the second percentage is still very, very high.

    Congrats again on your new understanding, but beware that Dunning’s explanation is truly and totally wrong.

    1. B_Petschel says:

      As I mentioned earlier. It was more the comments after the article that helped me to piece it together.

      Does my explanation, largely for my own benefit, work?

      1. ThinAirDesigns says:

        Oh, OK — I misunderstood you. When you said “third or fourth comment”, I thought you meant the third of fourth paragraph of Dunnig’s blog comment.

        Poor interpretation on my part as “comment” has a pretty specific meaning on these blogs.

        >> “Does my explanation, largely for my own benefit, work?”

        I believe in a recent post, Rick addressed one common misconception that you included in your understanding.

  62. Rick Cavallaro says:

    @Prang

    >> This is just wrong…

    JB and I are busy with a patent search. I’ll let B_Petschel straighten you out.

  63. Rick Cavallaro says:

    @ B_Petschel

    >>This is a problem that is perfectly designed to trip up those who are very familiar with physics.

    That’s exactly right. I don’t go into detail in this article, but I developed it specifically as a brain-teaser. I then posted it on a forum under the heading “Annoying brain-teaser”.

    >> When I read other peoples posts on every site I have visited they are making the same mistake as me.

    That’s also exactly right. We try very hard to explain to people exactly where they go wrong. Suffice it to say – it’s still difficult.

  64. ThinAirDesigns says:

    @B_Petschel

    >> “I know Dunning got it slightly wrong but he had a few points that sent me in the right direction.”

    Again I’m glad for good you received, but there is nothing “slightly” wrong about stating that a windmill/turbine drives the wheels.

    That’s 100% wrong and *can’t* work. This turbine vs propeller point isn’t some little ‘slight’ side issue that’s only tangentially relevent — it’s the KEY to the entire problem.

  65. Subduction Zone says:

    @Prang: This “The propeller will produce some thrust but that thrust can never give the cart more kinetic energy than the cart has used to spin the propeller and the wheels.” is not supported by any science, that is your prejudice speaking. And what about the treadmill? Galileo, Newton, and Einstein all have no problem with the treadmill being a case of the cart running directly downwind faster than the wind. As the saying goes “All forces are local”. That means the cart cannot tell the difference between the cart running on the road at the speed of the wind or running in place on the treadmill. In both cases it is running off of the difference in speed between the air and the surface underneath it. Outside it runs on the difference between the speed of the air and the speed of the ground, inside it runs off of the difference between the speed of the air and the speed of the treadmill surface.

  66. asymptotic says:

    @Prang: The cart reduces the relative velocity between the air and the surface – just like a windmill or sailboat – thereby acquiring energy that can be used to do work. If you want to convince anyone that the cart violates conservation of energy, you’ll need to show that the energy taken from the wind and/or earth is less than the energy required to accelerate the cart or keep it moving against friction.

    You’ve presented no such argument, so your assertion is (literally) groundless.

  67. asymptotic says:

    @Subduction Zone: “It only makes sense that a neutrally buoyant balloon tracks a gust exactly.”

    Actually it doesn’t track gusts **exactly**. It literally cannot, because a gust consists of a huge number of air molecules, all moving in different directions at different speeds. Clearly the balloon cannot track all of them since they aren’t moving together (not even close – the typical speed of an air molecule at room temperature is around 1000 mph, while the averaged velocity of the gust might be 10 mph).

    You can average the velocity of the air molecules over a region, but the answer you get will depend on the size, shape, and location of the region, as well as when you do the average. If you average the velocity of the molecules over a region around the balloon and not much bigger than it, that should be close to the velocity of the balloon. In practice the motion of a neutral density balloon is probably the best way to determine average wind speed on that scale.

    But in general there’s no reason for balloon speed to agree with the speed on larger or smaller scales or at different locations. For example consider a mini-tornado running into a blimp or very large balloon. Obviously the blimp cannot “track” it. Similarly the average speed of the wind in the earth’s atmosphere is probably quite large, since it’s dominated by very fast winds in the stratosphere. Obviously those don’t affect balloons near the ground. The argument that a neutral density balloon must move exactly like a volume of air fails because the balloon is an approximately rigid object, unlike air (which can sustain shear flows).

    So in short, no, a neutral density balloon does not track gusts exactly. However its velocity is a good indicator of the average air velocity, where the average is taken over a roughly balloon-sized region around the balloon .

  68. Rick Cavallaro says:

    @ asymptotic

    You and I debated this long ago, and sadly it didn’t end well.

    It appears that what you’re doing now is to disallow the abstraction of the concept of a gust by talking about the velocity of individual air molecules. We can de-construct almost any argument in that way – without adding meaning or clarifying the issue at all. Your argument would suggest that a hot air balloon doesn’t “drift with the wind” since the wind is in fact made up of countless molecules each moving in its own direction at 1000 mph.

    In the sense that you used to argue the balloon does not track the gust, that’s been demonstrated to be simply incorrect. In the sense that you want to debate whether a million particles all moving 1000 mph constitutes a gust is pointless semantics. Arguing that a hot air balloon is “approximately rigid” is just silly. Perhaps I should post a link to a photo of a recent mid-air between a hot air balloon and a paraglider.

    I’d prefer we not go too far off on this particular pointless tangent on this thread. Generally I’m fine with going on these tangents on discussion forums, but I’d prefer we stick more closely to the topic of DDWFTTW here. What do you say?

  69. Subduction Zone says:

    You are right, I should have not said exactly. I was objecting to some peoples beliefs that the mass of a neutrally buoyant balloon was different from the surrounding air and would have a pause before it reacted to a gust. I do agree that shear forces are different for air and a balloon which does have at least a semi-rigid behavior.

  70. asymptotic says:

    @Rick: I do recall debating this with you, but I don’t recall it not ending well. I thought it was an interesting discussion and I learned something from it.

    I don’t understand your objection about molecules. In my post I defined wind velocity by the average of the velocity of the air molecules over some region at a specific time. Then a gust is simply a region where the wind velocity differs from its value in most other regions nearby. An important point however is that the “wind velocity” depends on the region you do the average over, sometimes quite strongly.

    Of course balloons are “approximately rigid” compared to a fluid (like air). Free air can sustain a shear flow; a balloon obviously cannot. That’s a basic difference that has consequences.

    As for sticking to the topic, I’m responding to a topic you yourself brought up here. If you don’t want to discuss it, don’t discuss it.

    @Subduction: Agreed.

  71. Rick Cavallaro says:

    @ asymptotic

    > As for sticking to the topic, I’m responding to a topic you yourself brought up here.

    Yes, I recognize that – sort of. I was actually responding to an analogy in which another poster brought it up.

    > If you don’t want to discuss it, don’t discuss it.

    My point was not that I don’t want to take brief tangents, but that I feared we might go off for 10 pages on this one. I’m satisfied that your understanding is effectively the same as mine on this, and I’m inclined not to discuss it further. So if anything more needs to be said on the topic, you get the last word.

  72. fordworth says:

    Other than a claim of “we did it” I see nothing to support the claim or any explanation of how on earth this could actually work. But I do see the acknowledgment that “If you ride your bike downwind at exactly wind speed you won’t feel any wind either.”. What this admits, at least to me, is that if such a wind powered vehicle did manage to make it all the way to the speed of the wind, then it would no longer “feel” any wind effect on it. With no wind effect on it, it would be unable to go any faster (and in a world with friction it wouldn’t even get that fast). So the article claims it was done, but also gives good reason to believe that it can’t be done and no argument for why it could be done.

  73. Rick Cavallaro says:

    see above

  74. Jon Seymour says:

    Is there any reason why, having achieved 2-3x downstream windspeed you can’t turn into the wind and head upwind with the same speed?

    jon.

  75. Subduction Zone says:

    It would take a different gear ratio and preferably a different rotor. For directly downwind faster than the wind the rotor is acting as a propeller. For directly upwind the rotor acts as a turbine. For best results they have different profiles. That is a project for next year. spork and ThinAirDesign are reworking the cart so that it will hopefully attain directly upwind faster than the wind. I am sure that Rick will fill you in with more detailed, and correct information.

  76. Rick Cavallaro says:

    @ Jon Seymour

    There are several reasons that we can’t just turn into the wind and head upwind at the same speed. However, this winter we plan to make the necessary modifications so the cart can go directly upwind faster than the wind.

    As it is now the wheels turn the prop. To go upwind the “prop” (actually turbine) will have to turn the wheels. So we’ll be making a turbine to replace our prop and will be modifying the transmission a bit.

    Also, when going downwind at 3X wind speed we feel a 2X wind speed headwind. If we go upwind at 3X wind speed we’d feel a 4X wind speed head wind. Twice the wind speed means 4 times the effort, and 8 times the power. That’s just not in the cards for Blackbird. I kind of hope to achieve 2X wind speed directly upwind. That will still give us a stronger headwind than we had on our downwind runs.

    1. Jon Seymour says:

      Thanks for the explanation.

      Impressive work, btw.

      I am amused by the epistemological aspect to this story. It reminds me of the fracas over the Monty Hall problem.

      jon.

  77. Subduction Zone says:

    See, a more correct version. And yes, I should have realized that with a turbine the rotor will turn the wheels. We will be cheering you on from the sidelines Rick.

  78. AllParadox says:

    Downwind faster than the wind: Blackbird sets a record
    Preview your Comment

    My Bachelor’s degree is in Physics. I have some understanding of what is going on.

    As a project for a graduate degree, I developed a device that allows a canoe to advance directly into the wind.

    That, I understood.

    When there is a motion differential, some energy is available. Wind drives windmills.

    The kicker is that the frame of reference is irrelevant. My instinct is that this is not true, but I believe it anyway. I learned it in Physics classes, and, practically speaking, my canoe device works. I didn’t think it could, initially.

    At the time, I also considered the challenge of running downwind, faster than the wind, on some kind of exotic tricycle. I just could not wrap my head around it.

    So how can we understand Blackbird working, at all?

    First, imagine Blackbird with a fixed prop and no connection to the wheels. An adequately strong breeze will push it downwind. It rolls efficiently, so it will quickly get near the speed of the wind.

    Next, imagine Blackbird with the wheels geared to the prop, but with a high ratio: lots of wheel revolutions to a prop revolution. The prop will free-wheel, due to the design. The max speed will be near that of the fixed prop.

    Next, set the gear ratio so that the wheels drive the prop at the rate it spins in free-wheel. No energy is being transferred from the wheels to the prop at top speed. Note that this is still less than the wind speed.

    You guessed it: reduce the gear ratio some more. Now, when the Blackbird moves, the wheels drive the prop faster than the wind does. The propeller accelerates the Blackbird. Again: at this gear ratio, the wheels drive the propeller, not the other way around.

    Our intuition fails us here. In spite of “understanding”, we often do not. It is very hard to ditch the image of the farmer’s many-bladed, fixed, windmill.

    It may help to compare a Blackbird with a free-wheeling prop with the Blackbird with the reduced gear ratio.

    Start them at the same place, next to each other, with a tailwind. Initially, they are identical, and move together down the course.

    At some point before either reaches final velocity, the wheels will be turning just fast enough to exactly match the prop. The prop is not free-wheeling, and the wheels are not driving the prop. No energy is being transferred.

    However, we are still accelerating: we are not at the final velocity for either racer. When the Blackbird with the reduced gear ratio accelerates a little more, the wheels start driving the prop. That Blackbird starts to outpace its competitor.

    Eventually, the Blackbird with the reduced ratio reaches the speed of the wind. At that speed, the prop is pulling the Blackbird forward. The wheels are driving the prop, and creating drag. That drag is the source of the energy to drive the prop.

    The reason that this is not an eternal motion machine is that the wind is blowing. If the wind were to stop, the Blackbird would roll to a stop as well. Because of the wind, the wheels are extracting more energy from being forced to roll, than is being expended by the prop.

    My intuition tells me that this is completely wrong. However, the Blackbird, not knowing about my intuition, accelerates anyway.

    When there is a motion differential, some energy is available. Frame of reference is irrelevant.
    ____________________________
    AllParadox

  79. Rick Cavallaro says:

    @ Jon Seymour

    >> Thanks for the explanation.

    Certainly. Thanks for your interest our little project.

    >> Impressive work, btw.

    Thanks very much. You might think differently if you saw it up close : )

    >>I am amused by the epistemological aspect to this story.

    I was absolutely amazed in so many ways about that aspect. I thought I was a guy that’s hard to surprise, but I’ve had my share of them with this project. I honestly was silly enough to think I could post this as a brain-teaser, toss it around a bit, post the answer and move on. I never imagined the controversy it could cause, and I certainly never imagined that some of the most highly qualified engineers and scientists would absolutely refuse to accept what I thought were simple explanations, and ultimately the real-world evidence.

    I’ve come to decide that being open-minded but skeptical is far better than simply being highly educated.

  80. Rick Cavallaro says:

    @ AllParadox

    yup.

  81. Prang says:

    @Fordworth, who wrote:
    “Other than a claim of “we did it” I see nothing to support the claim or any explanation of how on earth this could actually work. But I do see the acknowledgment that “If you ride your bike downwind at exactly wind speed you won’t feel any wind either.”. What this admits, at least to me, is that if such a wind powered vehicle did manage to make it all the way to the speed of the wind, then it would no longer “feel” any wind effect on it. With no wind effect on it, it would be unable to go any faster (and in a world with friction it wouldn’t even get that fast). So the article claims it was done, but also gives good reason to believe that it can’t be done and no argument for why it could be done.”

    Yes that about sums it up. What you have not counted on is the claim of “ground energy”. Yes, that is right; the proponents of this really do claim that once the cart reaches wind speed, the ground moving under the cart turns the wheels and drives the propeller and that creates thrust which pushes the cart faster ever faster! Now we all know that the ground presents friction to the rolling wheels which will slow the cart down, but don’t let that get in the way of a good story! No no, you must believe that it is the energy from the ground that turns those wheels and drives the cart faster ever faster! That is what they want you to believe. Do you????Does anybody???

  82. Jon Seymour says:

    @Prang

    The problem with your analysis is that you are proposing that no matter what the gearing ratio, no matter what the propulsive force generated by the action of the wheels turning the propeller, the net propulsive force generated by the propeller always exactly matches the force of rolling resistance.

    How exactly do the wheels know that they are meant to feeling rolling resistance in direct proportion to the gearing ratio of the prop?

    Are you seriously proposing that there is some law that requires that the force of rolling resistance always matches the net propulsive force of a propeller, irrespective of the gearing mechanism that connects the wheels to the propeller?

    The law of conservation of energy only applies in a closed system. This is not a closed system. There is wind, it is moving relative to the ground. Remove the wind, there is no net force and no acceleration.

  83. Subduction Zone says:

    @Prang

    You seem to be conflating the conservation of energy with the conservation of forces, the first is a law of physics, the latter is nonsense. By your understanding of physics a simple lever would not work since the force on one side is more than that of the other. Forget your prejudice for a while and look at the air that the cart interacts with. It leaves a trail of air that is moving slower relative to the ground after it passes through the cart’s rotor. You should be asking where that energy went to.

  84. Prang says:

    I am seriously proposing that all the cart has to work with is the kinetic energy given to it by the force of the wind as it was pushed up to wind speed. That is the only energy the cart has. Now you can take some of that energy and convert it into rotational energy for the propeller. Guess what happens; the cart loses kinetic energy and slows down. Are you seriously proposing the propeller can create thrust to not only replace the lost KE of the cart, but actually increase the KE so the cart goes faster? Energy is conserved! You cannot spend a few joules here and expect to get back more than you spent. In fact, it is well established that you always without exception, get back LESS! That is all there is to it, really. Regardless of how creative the story-tellers are there is no way to get around the law of conservation of energy.
    Once at wind speed, if the cart should ever get there (it won’t) there is no longer any wind force on the cart and no longer any source of kinetic energy. End of story. You can believe what you want, but there is NO evidence to support this claim. I predict NALSA will be forced to retract their certification of this “record” and issue an apology. By the way, what good owning a record that has no competitors? That seems like crowning yourself the “King of the Outhouse” because there is nobody else in there!!! LOL!!!

    1. Jon Seymour says:

      @Prang

      So, if you are proposing that all the energy the cart has comes from the wind, how is it that on a treadmill the propeller spins even though there is no wind?

      Are you proposing the propeller spins in that case because it feels like it?

      Alternatively, are you proposing that when the cart is also pushed by wind, it stops feeling any force from the ground?

      How does the cart know it is meant to transmit energy from the treadmill to the propeller, but know that it must not transmit energy from the ground to the propeller?

      There absolutely is evidence to support the claim. You are the one that has no evidence. All you have is a completely unsupported and unsubstantiated claim for a hoax for which you have precisely no evidence. [ Apart for your wild theories, do you have any actual evidence? ] All you really have is the desperate insistence that you understand physics better than the rest of us.

      As has been explained energy is conserved. It is removed from the moving airmass and transferred to the cart by the propeller. If the airmass stops moving, there is no transfer of energy from the airmass to the cart.

      jon.

    2. Alsee says:

      Prang, you’ve almost got it. You just don’t fully follow the energy budget and the force balance yet.

      At all times energy is going into the propeller to create thrust. At all times a load in placed on the wheels to extract energy. Yes, the wheels are always pushing to slow the vehicle down. The normal reaction is to shout “conservation of energy”, with the knowedge that the energy consumed by the wheels MUST be greater than anything you can get back out of the propeller. Obviously any losses in the system mean the vehicle can only slow down. However that overlooks the energy differential between the wind and the ground. It overlooks the fact that the propeller is linked to the wind at one speed while the wheels are linked to the ground at a different speed. The fact that wheels and propeller are linked to different places BREAKS the normal reflex expectations you had. Energy exists in the difference between the wind and the ground. That energy can be harnessed.

      The vehicle’s ground speed is greater than the vehicle’s air speed, therefore the wheel’s ground travel distance is large while the propeller’s air travel distance is small. Energy = force * distance. The energy balance is:
      small force * large distance = large force * small distance

      The wheel’s wrong-direction force against the ground is small while the propeller’s thrust force against the air is large!

      The large forward propeller thrust minus the wheel’s small backwards force sums to a net forward force. The vehicle accelerates. The energy that exists between the wind and the ground first pays for any inefficiency losses and then goes on to buy us acceleration.

  85. Rick Cavallaro says:

    @ Prang

    >> Conservation of Energy . . .no way to beat it!

    And no need to.

  86. Prang says:

    Jon wrote: “So, if you are proposing that all the energy the cart has comes from the wind, how is it that on a treadmill the propeller spins even though there is no wind?”

    Maybe you have not noticed the tread is motor-driven? The tread is nothing more than a belt drive from the motor to the wheels allowing the motor to turn the wheels and the propeller. Thanks for noticing there is no wind! The cart on the treadmill is motor-driven!

    Jon: “Are you proposing the propeller spins in that case because it feels like it?”

    I have proposed no such thing and it would be appreciated if you do not try to put words in my mouth. Thanks in advance.

    Jon: “Alternatively, are you proposing that when the cart is also pushed by wind, it stops feeling any force from the ground?”

    Not at all! The cart always feel a resistive force from the ground and it must have a source of energy to overcome that resistance. The wind is that source of energy right up to where the cart theoretically reaches wind speed. At that point the wind force and wind energy disappear and only the ground resistance remains. The cart can do only one thing: slow down!

    Jon: “How does the cart know it is meant to transmit energy from the treadmill to the propeller, but know that it must not transmit energy from the ground to the propeller?”

    The same way a wheel “knows” when a motor is turning it and when a brake is working against it. The motor is an energy source, the brake is an energy drain. Follow the heat flow! The ground is cool until the cart passes over it and then it is heated by the cart losing kinetic energy to the ground due to friction. The wheel of the cart on the treadmill are cold until the motor-driven belt runs under them and turns them, turning the propeller. The heat flow never lies and it is the most reliable indication of what is the energy source, regardless of reference frames!

    Jon: “There absolutely is evidence to support the claim. You are the one that has no evidence. All you have is a completely unsupported and unsubstantiated claim for a hoax for which you have precisely no evidence. [ Apart for your wild theories, do you have any actual evidence? ] All you really have is the desperate insistence that you understand physics better than the rest of us.”

    I have not made any claim. I am the one who is disputing the claim made for this cart here on the almighty internet, with no published papers, no mathematical proof and only youtube videos to back the claim. I say it is internet woo. I have no wild theories only the tried and tested fundamentals of physics which nobody can call into question.

    Jon: “As has been explained energy is conserved. It is removed from the moving airmass and transferred to the cart by the propeller. If the airmass stops moving, there is no transfer of energy from the airmass to the cart.”

    I am the one who has explained the conservation of energy, not the claimants for this cart story. The airmass does stop moving, relative to the cart, if the cart ever reaches wind speed and there can be no transfer of energy to the cart from the wind at that point. I’m glad you at least understand that!

    1. theflyingtinman says:

      >> … if the cart ever reaches wind speed and there can be no
      >> transfer of energy to the cart from the wind at that point.

      So can you explain how the ruler in this experiment : http://www.youtube.com/watch?v=k-trDF8Yldc continues to transmit energy to the cart when the cart is clearly moving faster than the ruler?

    2. Alsee says:

      Prang: “conservation of energy…if the cart ever reaches wind speed and there can be no transfer of energy to the cart from the wind at that point.”

      This is an *extremely* important point. Conservation of energy is an extremely powerful argument. In fact it is so powerful that it completely dominates all argument and reasoning following it. You already *know* which side is automatically right and which side is automatically wrong. You can’t engage in any serious argument from then on, all you can do is try to humor and correct the person who is violating conservation.

      Humor me for one moment and imagine someone who *mistakenly* thinks they see a violation of conservation. That misimpression will be powerful. It will be powerfuly misleading. In fact it is so powerful that it completely dominates all argument and reasoning from that point. They (mistakenly) “know” which side is automatically right and which side is automatically wrong. They can’t engage in any serious argument from then on, all they can do is (mistakenly) try to humor and (mistakenly) correct the person who is violating conservation.

      You said “if the cart ever reaches wind speed and there can be no transfer of energy to the cart from the wind at that point”, but note that you were following a *conservation* line of reasoning! Lets check that:

      Given a cart rolling on the ground at the same speed as the wind, can we agree:
      (1) Energy *does exist* between the air and the ground.
      (2) Energy that *does exist* can move.
      (3) It is not a violation of *conservation-of-energy* for that energy to move into the cart.

      The actual problem is that you don’t (yet) see any way to *get* the energy to move into the cart while the cart is moving at windspeed. Solving that is a very nasty *engineering puzzle*, but it doesn’t inherently violate conservation or any other law of physics.

      Consider the treadmill again. Did you notice that the cart accelerates “the wrong way”? It accelerates *against* the tread!

      How does it do that? A force exists between the wheels and the tread. The cart then transfers that force to the propeller. The propeller creates a force pushing against the “not moving” air. Note that the cart accelerates *against* the tread. That is because the propeller’s force against the cart is *greater* than the wheels’ opposing force on the cart. Read that again – the propeller force is greater than the opposing wheel force. There is no violation of conservation there. The wheels are used to capture one unit of energy from a *small* force against the tread, and 99% of that 1 unit of energy goes into the propeller to generate a *larger* force against the air. Conservation of energy does not mean conservation of force. You really can use the energy captured from small force on the wheels to produce a larger force against the air, even when the air is not moving relative to the cart.

      That also precisely describes how a cart moving at windspeed can and does work. Copy-paste with “tread” changed to “ground”:

      How does it do that? A force exists between the wheels and the ground. The cart then transfers that force to the propeller. The propeller creates a force pushing against the “not moving” air. Note that the cart accelerates *against* the ground. That is because the propeller’s force against the cart is *greater* than the wheels’ opposing force on the cart. Read that again – the propeller force is greater than the opposing wheel force. There is no violation of conservation there. The wheels are used to capture one unit of energy from a *small* force against the ground, and 99% of that 1 unit of energy goes into the propeller to generate a *larger* force against the air. Conservation of energy does not mean conservation of force. You really can use the energy captured from small force on the wheels to produce a larger force against the air, even when the air is not moving relative to the cart.

      It is extremely counter intuitive, but it does successfully tap into the energy that *exists* between the air and the ground. A cart which can accelerate *against* a tread in non-moving air can also accelerate *against* the ground while seeing what appears to be “non-moving air” of equal speed.

      The cart doesn’t know it’s own absolute speed. The “moving air” frame of reference has identical physics to the “moving tread” frame of reference. All the cart knows is that an energy difference exists between the air on one side of this machine and the tread/ground on the other side of this machine. The cart doesn’t know (and doesn’t care) whether a motor is driving the tread or if weather is driving the air. An energy difference *exists* between the air and the ground. The opposite sides of this machine separately link up to those two places and tap into that energy to actively push against the air. It can push against air even when the air is not moving relative to the machine.

  87. Subduction Zone says:

    @Prang:

    How does the cart know that it is on the road, and can’t go faster than the wind, and when it is on the treadmill, and can go faster than the treadmill? How are the forces the cart receives any different in those two cases?

  88. vf says:

    Before you guys waste more time on explaining this, you should consider that some are not willing/capable to learn anything. Here some symptoms of such hopeless reality denial:

    – Subject doesn’t accept the treadmill test, but cannot tell which forces acting at the cart would be different compared to natural wind, causing a different acceleration.

    – Subject cannot prove his point mathematically, just write walls of imprecise statements and ambiguous words.

    – Subject cannot understand/disprove the analyses done by MIT Prof. Drela or the paper by Gaunaa, so he just keeps lying that no mathematical proof was provided.

    Keep your energy for the intelligent sceptics, who usually get it within a few hours/days.

    1. Subduction Zone says:

      That is true, but you can some very entertaining physics if you keep poking these deniers.

      1. vf says:

        Subject doesn’t accept the treadmill test because it uses a motor to create relative movement between surface and air (a.k.a. wind):

        “The cart on the treadmill is motor-driven!”

        But he demands a wind tunnel test:

        “Put the cart in a wind tunnel since it is wind-powered.”

        Subject obviously doesn’t even realize that wind tunnels also use motors to create relative movement between surface and air (a.k.a. wind).

  89. kamakazi says:

    This was definitely an interesting thread of comments, but I think the thing that really surprised me was the apparent efficiencies involved.

    I think I, like many people who looked at the pictures, thought this was a windmil driving the whels, and was surprised it could exceed wind speed, but didn’t really put much thought into it. When I actually read the discussion, and discovered I had it backwards I was amazed.

    A hard wheel on a hard surface has very good efficiency, obviously compromised at times to increase traction. What I “felt” was that a propeller would be much less efficient. I guess I just assumed that the coupling between the blades and the air would be lossy enough that in real life this would not work.

    This makes me very curious to see a graph of propeller coupling rates over time, as in how the Wright Brothers did, through the biplane era, and up to modern turboprops.

    I am sure that hand carved wooden propellers with thick root profiles and fixed angles of attack would never couple well enough for this to work.

    1. Subduction Zone says:

      Or worse yet, a propeller made out of popsicle sticks as an author for this magazine tried to do. If you look at the photograph that accompanies this article:http://www.make-digital.com/make/vol11/?pg=60&pm=2&u1=friend#pg60
      You will see that he had his propeller spinning in the wrong direction as well. Charles Platt still doesn’t believe that DDWFTTW is possible, even after all of the numerous treadmill tests, outdoor tests, certification by an international governing body, or scientific papers. For some of the deniers who have little or no science education that is to be expected, but from someone who is supposed to be writing science papers for a amateur journal? Unexcusable.

    2. Jon Seymour says:

      @Prang

      > Not at all! The cart always feel a resistive force from the ground and it must have a source of energy to overcome that resistance. The wind is that source of energy right up to where the cart
      > theoretically reaches wind speed. At that point the wind force and wind energy disappear and only the ground resistance remains. The cart can do only one thing: slow down!

      You claim that only the force of ground resistance remains. This is clearly not correct. The propeller is driven by the relative motion of the cart with respect to the ground (as it is on the treadmill). The propeller generates thrust. If this thrust is greater than the resistive force experienced by the cart, there will be net acceleration as given by Newtown’s Second Law: F=ma.

      Rick and JB claim that they have built a system which is capable of generating sufficient thrust from the propeller to overcome the resistive forces acting on the cart and hence achieve net acceleration that ultimately results in DDWFTTW motion. They have demonstrated this claim repeatedly with working prototypes, despite your numerous completely unsubstantiated claims to the contrary.

      > The same way a wheel “knows” when a motor is turning it and when a brake is working against it. The motor is an energy source, the brake is an energy drain. Follow the heat flow! The ground is
      > cool until the cart passes over it and then it is heated by the cart losing kinetic energy to the ground due to friction. The wheel of the cart on the treadmill are cold until the motor-driven
      > belt runs under them and turns them, turning the propeller. The heat flow never lies and it is the most reliable indication of what is the energy source, regardless of reference frames!

      You claim that there is no source of energy other than the wind to drive the propeller and that at wind speed, there is no apparent wind, and hence no energy to drive the propeller. However, you appear to be forgetting the Law Of Conservation Of Energy. In particular, at wind speed the cart has 1/2 m * w * w energy accumulated as it reached wind speed. This is the energy that will be dissipated by resistive forces but it is also energy that can be used to drive the propeller.

      Yes kinetic energy is lost to resistive forces. But kinetic energy is also gained as the cart accelerates to its maximum speed as a result of the net force imparted directly by the action of the propeller against the airmass. Your models only ever substract the resistive kinetic energy loss. They never add kinetic energy gain caused by acceleration gained from the net thrust added by the propeller. Why?

      The reason the cart can travel faster than the wind is that at wind speed, the net force (including the propeller thrust derived from the relative motion of the ground to the cart)
      is greater than the resistive force caused by the motion of the cart on the ground and its drag through the air. The causes the cart to accelerate and hence increase its speed.
      As the speed rises, these forces will eventually match exactly. When this happens there is no net force acting on the cart and the cart attains its maximum speed.

      There is a law of nature that says the speed will increase until such time as there is no net force acting on the cart at which time the cart will remain in a state of uniform motion. This is Newton’s First Law. The maximum speed is thus determined by the design of the cart and the effects of Newton’s First Law.

      As Rick and JB have noted, the maximum speed/ratio ultimately attainable depends on the efficiency of the propeller. It is exactly that speed the cart experiences when the net forces acting on the cart are zero.

      There is no law of nature that says that this maximum speed must be less than or equal to the wind speed. All that matters is that all the laws of physics, including the Conservation Of Energy Law _and_ Newton’s Laws, are satisfied. If a propeller produces any thrust at all, then the maximum speed will be that speed at which the net force experienced by the cart is zero, irrespective of what that actual speed that is.

      Energy is conserved because in the presence of a propeller that converts thrust into translational kinetic energy, the moving airmass loses more momentum than it would otherwise. There is no net loss of energy, so no violation of your treasured Conservation of Energy Law.

      If you doubt that a spinning airfoil can remove energy from a moving airmass, you also need to explain where wind turbines extract their energy from. In the case of wind turbine, the loss of energy is converted into rotational energy and thence into electrical energy. In the case of the cart’s propeller, the rotational energy imparted by wheel’s action on the gearing mechanism, removes energy from the moving airmass by translating that loss of airmass momentum into an increase of momentum of the cart.

      > I have not made any claim. I am the one who is disputing the claim made for this cart here on the almighty internet, with no published papers, no mathematical proof and only youtube videos to
      > back the claim. I say it is internet woo. I have no wild theories only the tried and tested fundamentals of physics which nobody can call into question.

      You have made a claim that the videos do not demonstrate what they purport to demonstrate. You are implying that these videos are faked, a hoax or fraud. You have specifically claimed it is “internet woo”. You do not have any evidence for this claim other than a failure of reality to behave as you think it should behave. This is not evidence of internet woo. This is merely evidence that reality does not behave as you think it should. If I might politely suggest, enlightenment awaits if you can grasp this possibility.

      If there is “internet woo” it is only with that camp that insists that Newton’s First and Second Laws do not apply here.

      > I am the one who has explained the conservation of energy, not the claimants for this cart story. The airmass does stop moving, relative to the cart, if the cart ever reaches wind speed
      > and there can be no transfer of energy to the cart from the wind at that point. I’m glad you at least understand that!

      A physical law is only as good as the model it is applied to. Your model lacks a propeller or the force imparted by a propeller and as such is irrelevant to the physical situation being described. Simply asserting the primacy of the laws of nature is pointless if you have picked the wrong model to apply them to.

      You also don’t get to choose which laws apply. Not only does the Law Of Convservation of Energy apply, so to do Newton’s First, Second (and Third) Laws.

      Your model does not account for propeller thrust. As such, your model does not respect Newton’s Second Law that a mass subject to a net force will accelerate. The only way a propeller driven cart cannot accelerate is if the thrust the propeller generates is less than or equal to the resistive force.

      Your model may be useful for analysing a propeller-less cart, but until such time as it is adjusted to take account of propeller thrust it can’t be used to describe a cart which does have a propeller.

      All you have to do to prove your argument that DDWFTTW motion is impossible, is to show that the thrust generated by the propeller is always less than or equal to the net resistive force. You have not done this. I encourage you to try. The exercise might be englightening. There is, of course, no law of nature (physical or human) that guarantees that it will be.

      jon.

  90. Gareth Branwyn says:

    I thought maybe it was time for another reminder for folks here to keep the conversation solely about the science and technology involved here. Questioning peoples’ intelligence or making sarcastic, condescending statements, or accusing them of being unwilling to learn because they don’t see it your way, is not staying focused on the experiments and results themselves.

    For such a high-heat topic, most people here have done an admirable job of keeping it civil. Thank you all for that. And please, let’s keep it that way. I don’t want to have to start removing comments.

    1. vf says:

      >> or accusing them of being unwilling to learn because they don’t see it your way,

      Asking for math, ignoring it when posted several times, and continuing to falsely claim that no mathematical proof exist … is IMHO unwillingness to learn.

      I was just pointing out, that to keep the discussion not only civil, but also productive, one should concentrate on those who actually are listening.

  91. Rick Cavallaro says:

    @ kamakazi

    > am sure that hand carved wooden propellers with thick root profiles and fixed angles of attack would never couple well enough for this to work.

    I think you’d be surprised. Goodman’s model used a hand carved wooden propeller and worked quite well. Additionally, his prop design was pretty far from optimal. This will keep you from achieving high multiples of wind speed, but not from going DDWFTTW.

  92. Subduction Zone says:

    I wasn’t too civil either. Too make it up I offer to Prang a simple series of yes/no questions that will show that the treadmill tests are the same as running in the wind at wind speed. So if the cart can move forward on the treadmill it is in fact going faster than wind speed. To start it out, Prang do you believe the many videos, done by various different people on the internet of DDWFTTW carts on a treadmill are genuine?

  93. drsteevo says:

    I just read this article last night and I have been thinking about it for the last 24 hours. This is quite fascinating. It took me a while to understand what I was seeing. You don’t need a vector analysis to understand this is correct.

    For the skeptics, maybe I can help explain in a non-physicist/non-engineer way:

    1. Look very carefully at the prop and the direction it spins.

    2. It spins like an airplane prop, NOT like a windmill.

    3. When the sled moves forward, the prop produces thrust in the rearward direction.

    4. The wind pushes the sled and makes the wheels turn.

    5. The wheels drive the prop and makes it turn like an airplane propeller.

    6. The propeller pushes the sled.

    If this doesn’t make sense to you, then OBSERVE THE VIDEO AND STILL PHOTOS VERY CAREFULLY AND THINK ABOUT IT A LITTLE MORE!

  94. Rick Cavallaro says:

    @drsteevo

    >> I just read this article last night and I have been thinking about it for the last 24 hours.

    That’s all we ask. I’m just concerned that some people might read it and think about it for only 16 hours. : )

    >> If this doesn’t make sense to you, then OBSERVE THE VIDEO AND STILL PHOTOS VERY CAREFULLY AND THINK ABOUT IT A LITTLE MORE!

    Your description is correct, but it sounds a bit too much like perpetual motion to most folks. The secret (so to speak) is that less power is needed at the prop than can be harnessed by the wheels. This is true because the cart moves through the air more slowly than it moves over the ground (being that it has a tailwind and all).

  95. AllParadox says:

    @Prang

    >>> “Other than a claim of “we did it” I see nothing to support the claim or any explanation of how on earth this could actually work. <<< If that works for you, then OK. Nobody is trying to force you to believe anything. I find "We did it" to be strong evidence. _______________ >>> What this admits, at least to me, is that if such a wind powered vehicle did manage to make it all the way to the speed of the wind, then it would no longer “feel” any wind effect on it. With no wind effect on it, it would be unable to go any faster (and in a world with friction it wouldn’t even get that fast). <<< The whole point is that, contrary to your intuition, and mine, in a world with friction and wind, it can, and does go faster, in the right circumstances. _______________ >>> So the article claims it was done, but also gives good reason to believe that it can’t be done and no argument for why it could be done. <<< "We did it" works for me. This is the Make magazine site, after all. _______________ >>> Yes that about sums it up. What you have not counted on is the claim of “ground energy”. Yes, that is right; the proponents of this really do claim that once the cart reaches wind speed, the ground moving under the cart turns the wheels and drives the propeller and that creates thrust that pushes the cart faster ever faster! <<< Yes, that sums it up. _______________ >>> Now we all know that the ground presents friction to the rolling wheels which will slow the cart down, but don’t let that get in the way of a good story! No no, you must believe that it is the energy from the ground that turns those wheels and drives the cart faster ever faster! That is what they want you to believe. Do you????Does anybody??? <<< Well, yes, now I believe. I did not, at first. "Friction" is not exactly correct. Yes, there is some rolling friction. The non-friction drag far exceeds the rolling friction. Blackbird was designed to reduce rolling friction to the absolute practical minimum. The ground also presents non-friction drag to the rolling wheels, because the ground is forcing the wheels to turn faster than they would otherwise. There is that silly prop at the other end of the drive train, holding them back. If the cart could have the propeller driven by another source, perhaps a gasoline engine, and the wheels allowed to freewheel, then Blackbird would accelerate faster and reach an even higher top speed. _______________ >>>I am seriously proposing that all the cart has to work with is the kinetic energy given to it by the force of the wind as it was pushed up to wind speed. That is the only energy the cart has.<<< A cart with a freewheeling prop will accelerate until the aerodynamic drag force from the wind exactly matches the rolling resistance from the wheels. The wind is still blowing. There is additional energy available. This is far more energy than the kinetic energy inherent in the mass and velocity of the Blackbird. Whether this wind energy can be harnessed, and used, is the question. _______________ >>> Are you seriously proposing the propeller can create thrust to not only replace the lost Kinetic Energy of the cart, but actually increase the Kinetic Energy so the cart goes faster? <<< Yes, precisely. _______________ >>> Energy is conserved! <<< I believe "energy is conserved" with a faith that is unshakable. _______________ >>>You cannot spend a few joules here and expect to get back more than you spent.<<< Umm, yes, we can, because we are stealing it from the wind. _______________ >>>In fact, it is well established that you always without exception, get back LESS!<<< My intuition tells me exactly that. My intuition is wrong. You always, without exception, get back LESS when there are no other forces acting. Here, there is another force, the wind. >>> Regardless of how creative the story-tellers are there is no way to get around the law of conservation of energy.<<< Ok, certainly, for sure. I agree absolutely. Blackbird obeys the law of conservation of energy, with no exceptions. However, Blackbird knows nothing about creativity, storytellers, or intuition, so it runs downwind, faster than the wind, anyway. ________________________ I have seen some competent analysis in other posts here, so I will not attempt to duplicate them. It is devilishly hard to get away from the wind-turbine concept, where the wind drives the propeller that drives the wheels. If ever you start down that road, you are lost. Let us start somewhere else. Let us start with a driven propeller on a support, driven at a reasonable speed. A driven propeller is just what it sounds like, a propeller being rotated by an energy source. Many aircraft propellers seem to like being run at 2400 RPM. Larger props turn less quickly. Large, high-efficiency props like the one on Blackbird, turn slower still. In a dead calm, a driven propeller exerts a force on the support. It “pulls” on the support, if you will. In a brisk wind, blowing into the face of the propeller, there is less force on the support from the driven propeller. Call it summing of forces, call it decreased efficiency, call it whatever you want, there is less force on the support from the propeller. If you do not accept this, then stop here. You will never understand. We can also turn the driven prop’s axis of rotation around on the support. Now the brisk wind is blowing into the back of the propeller. There is now more force on the support than from a driven prop in a dead calm. Call it summing of forces, call it increased efficiency, call it whatever you want, there is more force on the support from the propeller. Again, if you cannot accept this, then stop here and do some experiments on your own. You will not understand until you believe this. Here is the kicker: there is more than a minimal, theoretical, force. There is a LOT of force. There may be more force than your intuition will allow. Propellers can be surprisingly efficient when driven in their speed range. When Blackbird is traveling downwind at wind speed, the portion above the wheels is at a relative dead calm. At that instant, the propeller is being driven, and is exerting force on its support. That force is greater than the force necessary to turn the propeller at that speed. Thus, Blackbird is accelerating. This is only because of the tailwind and the efficiency of the propeller. If the wind were to cease, the efficiency would drop immediately, and soon Blackbird would roll to a stop. Also, the efficiencies are limiting. Aerodynamic drag goes up as the cube of the relative wind speed. The energy available from the wind is finite. Traveling at five or six times the speed of the wind may be completely impossible. Any number of seemingly small changes would kill the efficiency of the Blackbird as a whole. That Blackbird works at all is quite an achievement. _____________________ AllParadox

  96. Rick Cavallaro says:

    @ AllParadox

    Very nice. Unfortunately anyone not yet convinced isn’t likely to be led to the answer easily. I would be curious to hear what the skeptics have to say after reading Mark Frauenfelder’s article here:
    http://blog.makezine.com/archive/2010/11/what_ive_learned_about_wind_carts.html

    I think he did a heck of a job. And it would be very hard to claim he showed any bias.

    I have only a couple of minor nits to pick (it’s what I do)…

    >> Aerodynamic drag goes up as the cube of the relative wind speed.

    Drag is roughly proportional to the square of wind speed. Power however is proportional to the cube.

    >> Traveling at five or six times the speed of the wind may be completely impossible.

    I suspect you’re right that 5X or 6X isn’t likely to happen. But somewhat surprisingly there’s no real upper limit to the multiple of wind speed achievable. It just comes down to how efficient you can make the vehicle.

  97. ElecEngr20 says:

    Being an EE and having much pride in my understanding of Physics, I found this article alarming at first.

    Let me start by noting one thing about the propeller that a lot of people seem to miss: In steady state, it accelerates air towards the BACK of the vehicle (THRUST!).

    Now to make an interesting point, but first, a disclaimer:

    I am no ASE, and I have no clue what I’m talking about, so feel free to shoot me down.

    Isn’t the wheels driving the propeller effectively trading increased rolling resistance because of the propeller load to make the aerodynamic coefficient of friction effectively negative?

    1. Alsee says:

      “aerodynamic coefficient of friction effectively negative?”

      That is attempting to combine thrust and wind drag into “negative friction”. I think that’s a bad idea. It badly distorts the concept of friction. An actual negative friction would pump out magical energy, it would inherently tend to runaway into infinite speed, and it would have a magical refrigerator effect on anything it touched :)

      It’s best to think in terms of three forces: (1) propeller thrust (2) wheel drag (3) aerodynamic drag.

      The propeller force plus wheel force basically defines your drive force. That sum is always positive (or zero force at zero speed).

      Wind drag gives you the initial push-start (positive aerodynamic drag in the forwards direction). Forwards drag then declines to zero force at wind speed. Above wind speed the drag eventually grows to place a cap on your top speed.

  98. ausrandoman says:

    For the sake of argument, let the propeller be right-handed, ie, it turns like a right handed screw thread. For the wheels to turn the propeller in a direction that allows the propeller to pull the cart along, the drive from the wheels to the propeller must turn the propeller clockwise as viewed from behind.

    Put a clutch in the drive between the wheels and the propeller. Disengage the drive clutch.

    Call the wind speed V. Let the cart blow down wind. It will eventually get to a speed where the aerodynamic drag equals the rolling friction. This speed will be less than wind speed, say kV, 0

  99. Prang says:

    Of course there is thrust (I am not aware of “lots of people missing that point” because it should be obvious). However, that thrust comes at a price: it must be generated by forcing the wheels to do work. The price is paid in wheel drag. Wheel drag is computed very easily: Drag = Power/Velocity. The propeller thrust is computed in the same way: Thrust = Power/Velocity.

    To give you an example where there is NO wind and No losses due to friction and the cart gets pushed up to 10 m/s and the wheels are producing 10 Watts of power: Wheel Drag = 10 Watts / 10 m/s = One Newton. With No losses, the propeller gets the full 10 Watts of power and the Thrust =10 Watts / 10 m/s = One Newton of Thrust. As you can see, with No losses, the thrust exactly equals drag. Introduce even the tiniest real world frictional losses and the thrust is insufficient for sustained motion and the cart comes to a complete stop.

    Here is what the cart proponents claim: Since there is a tail wind, even if the cart is going faster than the wind, the tail wind reduces the velocity of the propeller in the air, so the thrust can be greater than the drag. For example, the cart is moving at 10 m/s in a 5 m/s tail wind. They compute the wheel drag based on the cart velocity on the ground of 10 m/s to be the same as the previous example as One Newton However, when they compute the thrust they only use a wind velocity of 5 m/s (because of the tail wind) so the thrust comes out to be 10 Watts/ 5m/s = Two Newton! And there is what their claim rests on!

    Now I will show you why that claim is false: Any cart with any sort of sail (including a propeller) and low rolling resistance will be blown down wind to reach very nearly wind speed. That is a “free ride” down wind, courtesy of the bluff wind force blowing against the frame and the sail. This is a well-known fact. Sail boats sailing directly downwind can reach well over 90% of wind speed. So achieving wind speed is pretty much a given and involves NO wheel drag, only rolling friction. To achieve twice the wind speed requires something more than wind force alone, the cart proponents say they do it with propeller thrust. So the thrust is associated only with exceeding wind speed, not in reaching it! That means the wheel drag is also only associated with exceeding the wind speed and not in reaching it! What that means is the wheel drag can only be calculated for the increase in velocity above wind speed, in this case 5 meters/second same as the thrust calculation is done. Now you will find that wheel drag is TWO Newton exactly the same as the Thrust as indeed it must be, and the cart cannot ever advance beyond wind speed.

    There is NO “free energy” and NO way to beat conservation of energy and NO way to go directly downwind faster than the wind!
    Again, anyone is entitled to believe what they want, but no one is entitled to make up their own creative mathematics to support this unscientific claim, and that includes Dr. Drela. Dr. Drela can of course come on this forum and defend his analysis at any time if he feels it can be defended. I eagerly await his posting!!

    1. Subduction Zone says:

      Sorry Prang, that is not a proof. You assumed the thrust would equal the wheel drag. Where is your evidence that that would be the case. Those are forces, and there is no law of conservations of forces, otherwise levers would not work. You don’t seem to want to engage me on my informal yes/no question proof that a treadmill is the same as running at wind speed.

      By the way there is another site where you can argue this called Talk Rational. You can get as mean and nasty as you want there:

      http://talkrational.org/forumdisplay.php?f=5

      1. ElecEngr20 says:

        @Prang: Thrust must equal drag: This is what the definition of steady-state is (ie. no acceleration — forces balance).

        Prang does make a good point, although the wording is a bit ambiguous and the conclusion unsound?

        @Prang:

        I agree with your 2nd paragraph.

        However, I’m somewhat perplexed by your 4th paragraph. So let me try to get this straight. You can correct me if I’ve gone wrong anywhere:

        The drag on the wheel is 2 Newtons because the propeller poses no load onto the wheel before it reaches windspeed, and only poses a load after it reaches windspeed.

        The propeller provides two Newtons of thrust because it is only 5 m/s relative to tailwind.

        Thus thrust = drag and the cart stays at 10 m/s.

        Here is the part I’m getting confused on, you said that the windspeed is 5 m/s but clearly the cart is moving in 10 m/s in your example.

        1. Alsee says:

          “Thrust must equal drag: This is what the definition of steady-state”

          Steady-state is reached at around 3 times windspeed. Steady state doesn’t really interest us much. What we are interested in is the fact that it *accelerates* continuously from zero up to windspeed and continues accelerating until steadystate.

          There are three forces. Propeller thrust, an opposing force applied at the wheel, and wind drag. We don’t actually care about drag because it merely gets us started at the first instant and merely limits out top speed. In the vicinity of windspeed drag is essentially zero. All we really care about is the propeller thrust and the opposite load on the wheels.

          The propeller pushes at the wind. This force on the wind *slows* the wind relative to the ground. The wind therefore contains *less* energy relative to the ground. This energy must go somewhere. This energy goes into the cart! For every force there is an equal and opposite force. If the propeller actively PUSHES at the wind, this creates an equal and opposite force of the wind pushing on the propeller. This solves the “slow wind” problem. It doesn’t matter if the cart is moving at windspeed or even faster. The “slow” wind does PUSH on the propeller because the propeller is actively pushing on the wind. The cart gains speed and energy.

          Now it’s a little tricky here. We have an energy loop, but it is NOT a free energy loop. Energy loops are perfectly legal if they loses energy. We can loop and recycle energy so long as some of it leaks away like in any normal machine.

          As noted before, the wind’s energy enters the cart as a forwards force. The cart is pushed forwards against the ground. The ground pushes backwards against the wheel. The faster the cart moves the faster the wheel turns. We pull the wheels’ rotation up to turn the propeller. We are drawing some of the energy from the wheel and using it to power the propeller. The propeller which pushed us in the first place.

          The important thing of course is to check conservation of energy. To proving that we do in fact legitimately pay our energy bills.

          Power = force * distance.
          The wheel has a small backwards force while the wheel travels a long distance against the ground. The propeller has a larger force, but the FORWARD MOVING WIND means the prop covers a smaller distance relative to the air. That is critically important. Lets compare energy pulled out of the wheels with the energy spend running the propeller:
          Small wheel force * large wheel distance = large prop force * small prop distance.
          That power transfer balances. Actually a bit of energy will be lost before reaching the prop, but that’s not a problem. As long as the wind is blowing the prop force will easily be bigger than the opposing wheel force. As long as the propeller force is greater than (wheel force + drag) the car will accelerate gaining energy from the wind.

          Note that at higher speeds the wind is moving slower than the cart. The prop is tied directly to the wheel speed. The faster the cart goes the faster the prop spins and the harder the prop actively “reaches back” to push against the wind. The prop speed automatically adjusts to solve the problem of pulling power out of slow wind, no matter how fast the cart speeds up.

          The energy exists in the wind-relative-to-ground. We can actively reach back to pull it out. That energy does pay our machine energy bills. Like any legitimate machine all of our energy dies away unless we have a steady supply of wind energy to power it up. No perpetual motion, no free energy. Everything coasts to a halt if the wind stops.

      2. ElecEngr20 says:

        That first @ was addressed to Subduction

    2. Alsee says:

      Prang: “So the thrust is associated only with exceeding wind speed, not in reaching it!”

      I’m not sure if you are knowingly ignoring low-speed-thrust as irrelevant, or somehow missing it. This machine has the prop hardwired to the wheels with a fixed gear. The wheels always have a load and the prop always generates thrust from the instant it starts to move.

      “What that means is the wheel drag can only be calculated for the increase in velocity above wind speed”

      Wheel drag always exists. It exists below windspeed, at windspeed, and above windspeed.

      “Now you will find that wheel drag is TWO Newton exactly the same as the Thrust”

      If there is a wind blowing the thrust is always greater than the wheel drag. Thrust exists below windspeed, at windspeed, and above windspeed.

      Consider conservation of energy in a zero wind and a zero loss analysis. CONSERVATION says the energy and force at the wheels must *exactly balance* the energy and force at the propeller. If you introduce a wind it breaks that balance. The wind effectively *adds* to the propeller thrust. The wind ALWAYS raises the propeller thrust above the wheel losses.

      “There is NO ‘free energy’ and NO way to beat conservation of energy”

      Correct and correct. We’ve tried to tell you that repeatedly. Energy exists between the air and the ground. That energy is consumed to drive the vehicle. You appear to have missed my last post dedicated to conservation of energy (Posted by: Alsee on November 8, 2010 at 1:34 PM).

      “NO way to go directly downwind faster than the wind!”

      Many people have independently experimentally proved that it can and does work. The top wind racing society has used professional testing equipment to independently document a manned cart continuously accelerating from a cold start to 2.8 times windspeed. When multiple well documented independent experimental demonstration conflict with your theoretical analysis it strongly indicates you should go back and recheck analysis for misunderstandings, mistakes, or oversights.

      It can and does work because it does *not* involve any sort of “free energy” and it does *not* involve any violation of conservation of energy.

    3. Rick Cavallaro says:

      @ prang

      >> There is NO “free energy”

      Correct

      >>…and NO way to beat conservation of energy

      Correct

      >>…and NO way to go directly downwind faster than the wind!

      Wrong.

      >>
      Again, anyone is entitled to believe what they want, but no one is entitled to make up their own creative mathematics to support this unscientific claim
      << Science relates to explaining what we observe in the real world - NOT insisting that the real world conform to our incorrect assumptions. >> Dr. Drela can of course come on this forum and defend his analysis at any time if he feels it can be defended.

      I’m sure he’ll be show up if he thinks it needs defending.

      >> I eagerly await his posting!!

      I think you’ll be waiting a long time.

    4. Jon Seymour says:

      @Prang

      I agree with @ElecEngr20. I am very much looking forward to your explanation of why a cart travelling at 10m/s, precisely twice the tail wind of 5 m/s is not regarded as traveling faster than the wind.

      Perhaps you would like to enlighten us on your definition of the number 2.

      jon.

    5. cdreid says:

      -In your thoughts- when the cart reaches the speed of the wind there is >no thrust on the propellers therefore there is no way to harvest the energy.

      The Wind Is Not Pushing the Propellers. I say again – The Wind Is Not Applying Force to the Propellers-

      The Wind is Pushing the Airstream created by the Propellers.

      Evaluation of the run.
      1. Wind pushes cart/propellers.
      2. Wheels propeller begins to freewheel in the wind acting as a sail.
      3. Wheels which are one-way-geared force the propeller to turn.. generating THRUST. Youre thinking the propeller is spinning the OTHER way… like a wind turbine.. and that the prop is just a big sail. The prop is functioning as a Prop.. not a wind turbine blade. The prop is nearly from instant zero functioning as a propeller.. exactly as in an airplane.

      Therefore the wind isnt pushing against the propellers.. just as it is not pushing against the propeller in an airplane. It is pushing against he AIRSTREAM generated by the propeller. So long as the wind moves faster tha nthe GROUND (not the cart) the wind will be generating force on the WINDSTREAM generated by the propeller thus you will always have a difference in energy.

      Again – the wind isnt pushing against the propeller. The wind is pushing against the airstream. The physics works. There is a difference in energy. You are harvesting that difference in energy. It is classic physics. The engeneering is only confusing because we keep thinking of the propellers as a sail when they arent.

      1. Rick Cavallaro says:

        >> The Wind is Pushing the Airstream created by the Propellers.

        Unfortunately this is a very common (compelling but incorrect) interpretation.

        There is no “air cushion” or convergence of flows behind the propeller. The flow in the region of the cart is identical to the flow in the region of a plane in flight – upwind, downwind, or no wind.

        1. cdreid says:

          That made no sense. And there is an airstream behind the propellers of an airplane. If there werent they wouldt move.

          The wind is pushing against that airstream. This is the reason the craft doesnt stop accelerating once it reaches windspeed.

          1. Rick Cavallaro says:

            >> That made no sense.

            I’m sorry that that made no sense *to you*. I assure you it’s correct.

            >> And there is an airstream behind the propellers of an airplane.

            Of course there is. I didn’t say differently. What I said is that the prop on our cart works exactly the same as the prop on an airplane – which works exactly the same way whether the plane is flying upwind, downwind, crosswind or in no wind. There’s simply no real-world phenomena in which the tailwind pushes on some sort of air cushion or airstream created by the prop.

            >>The wind is pushing against that airstream.

            Nope. It’s not.

            >> This is the reason the craft doesnt stop accelerating once it reaches windspeed.

            Incorrect. The reason the craft doesn’t stop accelerating when it reaches wind speed is because the prop is creating thrust by working on the airmass in which it’s submersed.

          2. Alsee says:

            I understand you, but I think it’s perfectly legitimate to consider the wind pushing on the prop wash. Just reverse the entire chain of force. The ground transfers a force against the wheel. The wheel transfers force to the frame and then the prop. The prop transfers force to a back wash. And that backwash will in fact impose a backwards force against a large volume of oncoming wind as it comes to rest in relation to the general airmass. Without that final step the backwash would become a low pressure zone sapping speed from the cart.

          3. Rick Cavallaro says:

            It’s find to say that the air being thrust backward pushes on the blade just as the blade pushes on it. What’s not correct is to say that the tailwind pushes on the air being thrust aft. The vehicle is submersed in a moving mass of air. Everything about the flow over the prop is no different than it would be for an airplane flying on a calm day.

            People are tempted to say that the wind can’t push on the cart because the cart is outrunning the wind – but it can push on that stream of air that’s being thrust aft. Unfortunately this just isn’t the case.

            And by the way, the cart can outpace the wind, but it doesn’t outrun the wind – because it’s submersed in the wind*

            * interestingly enough we outran a whole bunch of wind gusts while on the playa. We would wait for a gust to pass, give it some time, and then start. We would often catch up to the gust, accelerate through it, and come out the front.

          4. cdreid says:

            but for some reason youre trying to redefine physics terms here. Your “ground/air interface” talking point is nonsensical scientifically. As is your idea that “there is a sea of air and therefore nothing could push on it”. You also, above made a claim that there is no stream of thrust behind a rotating prop. A thought that is ridiculous on its face.

            Basically i believe youve created something incredibly clever but that either your understanding of science or your ability to communicate is extremely limited. Either that or youre intentionally setting out to confuse people for some reason. The explanation i gave, though it took several days to come to it.. is why your machine works. Not some mystical “ground air interface” or other semantic creation. Physics is physics.

          5. ThinAirDesigns says:

            @cdreid:
            >> “Your “ground/air interface” talking point is nonsensical scientifically.”

            >> “Not some mystical “ground air interface” or other semantic creation. Physics is physics”

            Unbolt a standard wind turbine from its interface with the ground and place it on top of a sufficiently large floating balloon. Wait for normal wind to blow and measure output. Return and report on the “semantically created” ground/air interface.

            There’s nothing “mysical” or “nonsensical scientifically” about the ground/air interface — it’s truly the key to wind power.

            Depending on the frame of reference it’s just as scientifically accurate to state that those turbines on the hill are powered by the ground. Truth is they are powered through the ground/air interface. Physics is physics.

            JB

          6. Rick Cavallaro says:

            >> either your understanding of science or your ability to communicate is extremely limited

            Your interest in understanding this is clearly extremely limited, and your interest in insulting the people that designed, built, and demonstrated it is clearly not.

            My interest in taking further abuse from you has gone to zero.

          7. vf says:

            >> That made no sense. And there is an airstream behind the propellers of an airplane. If there werent they wouldt move.

            In this case that “airstream” is just a trail of air that moves slower relative to the ground (and faster relative to the cart)

            >> The wind is pushing against that airstream. This is the reason the craft doesnt stop accelerating once it reaches windspeed.

            Now that makes no sense. What do you mean physically by “wind is pushing against that airstream”? Air pushing against air? Why would that affect the cart? The only air that affects the cart it the air that interacts with the cart (not with some other air somewhere behind).

            Here is the source of confusion:

            – in the *ground frame* the air moves *forward*
            – in the *cart frame* the air is accelerated *backwards*

            People mix these quantities from two different reference frames, and get the wrong idea that the prop wash somehow collides with the true tailwind, air cushion is created etc.

  100. Subduction Zone says:

    @EE29, If you look at my statement again you will see that it is correct. The force from the prop, once it is no longer in stall mode, is always greater than the drag from the wheels. When the cart gets to steady state all forces do balance, but they are wind drag, wheel drag, various frictional forces all in the “negative” direction and thrust from the prop in the “positive” direction. The negative forces equal the positive forces and since none of the negative forces are zero the thrust from the prop must be greater than the wheel drag.

    1. spleen says:

      A 15kmh wind pushes a 2km long vehicle with a sail mounted at the front of the vehicle at 10kmh. The sail is of a size such that it captures enough wind to overcome the frictional properties of the drive plus the electrical generators powered by the wheels momentum.
      At the vehicles maximum speed an electrical winch at the rear of the vehicle pulls the sail down a track at a rate of 10kmh causing an increase of vehicle speed of approx 6.66kmh.
      When looking at this method you may think that to pull the sail down the length of the vehicle would require more energy than can be generated, but if the drag related to the vehicle is minimal, then the energy required to pull the sail down is simply the mechanical losses plus the drag of the vehicle.

      1. spleen says:

        I see that the sail or wind capturing device is not moving faster than the wind, but the vehicle does.

        1. spleen says:

          similar to a sailboat tacking, the sail moves less or equal too but not faster than the speed of the down wind velocity while the hull speed of the boat is the value cos-1 minus losses. The increased rotation of the propellor blade actually runs slower than the prevailing wind however like the sail yacht scenario, the vessel moves faster.

          1. spleen says:

            Assuming the previous posts have content, I wonder whether increasing the surface area of the blade has more effect on the speed of the vessel rather than the efficiencies of the profile?

    2. ElecEngr20 says:

      @Subduction

      I was under the impression we are ignoring minor 2nd-order effects here such as bearing/chain friction. These are fairly static at the speeds we are talking at here and are fairly insignificant compared to aerodynamic drag and rolling resistance.

      Also, there is no effective wind-drag here (or rather, I was defining thrust as true prop thrust minus apparent drag from the rest of the body here)

      The intent of the original @Subduction was that I wanted to point out that your statement about the Law of Conservation of Forces is not technically correct. In this example there actually is a “law of conservation of forces” because there is no acceleration; this is perhaps better known as Newton’s Third Law.

      1. Subduction Zone says:

        Except that forces will not be balanced until those 2nd order forces are significant. When the cart is going at wind speed the forces are definitely unbalanced. If the cart is built well enough so that it can go two or three times the speed of the wind it will always be accelerating when it is going the speed of the wind. At wind speed the cart is still accelerating, going right along with the second law of motion. We ignore the secondary forces, wind resistance, because they are practically zero at that speed, or losses due to transmission etc. because those can be engineered to very low levels. We don’t ignore them and st the forces equal at that speed.

        1. ElecEngr20 says:

          “Except that forces will not be balanced until those 2nd order forces are significant.”

          Perhaps we aren’t talking about the same thing when we’re talking about 2nd-order effects here. So let me define them:

          1st-order effects (forces that vary significantly with the velocity of the craft):

          Rolling resistance (caused mostly by deformation of wheel and/or road)

          Wind drag (caused by the shape of the craft)

          Thrust (caused by the propeller spinning)

          2nd-order effects:
          All other order effects. (mostly Transmission losses from what I can see here).

          The reason why the craft has a top speed is because the effective thrust (true thrust minus aerodynamic drag) balances the rolling resistance.

          “We don’t ignore them and st the forces equal at that speed.”

          The 2nd-order effects (as defined above) ARE ignored because they are not likely to effect the craft’s speed by more than a couple of % (if engineered properly).

          1. Rick Cavallaro says:

            >> The 2nd-order effects (as defined above) ARE ignored because they are not likely to effect the craft’s speed by more than a couple of % (if engineered properly).

            What you’re defining as 2nd-order effects are actually not 2nd order effect; and they do have a profound effect on the vehicle’s speed.

          2. ElecEngr20 says:

            Could you please elaborate on this? Besides transmission losses, what else is there?

            I can’t imagine a simple chain between your wheels and propeller to be generating a large amount of frictional forces (compared to rolling resistance and wind drag)

          3. Rick Cavallaro says:

            I’d guess that our prop efficiency is in the neighborhood of 90% and that our chain drive may be in the neighborhood of 95%. Then there’s rolling resistance at about 0.01 gross weight. Finally there’s aero drag. I assume a projected frontal area in the area of 15 sq-ft with a Cd in the 0.3 ballpark.

          4. ElecEngr20 says:

            So I’m a bit clueless here, it’s been quite a while since I studied the fluid dynamics. Wouldn’t it be safe to include the prop loss inside the aerodynamic drag values?

            So, besides rolling resistance, and aerodynamic drag, the other losses you can think of is the transmission and prop-loss.

            85% of your losses are coming from rolling resistance and aero-drag, which is why I labeled them 1st-order effects.

            Perhaps 1st and 2nd order was a poor choice of words (since that might imply that these effects are linear/squared with velocity). Would it make more sense if I replaced 1st order with primary and 2nd-order with secondary?

            Am I starting to make sense here or am I still missing something?

  101. Rick Cavallaro says:

    If someone can tell me where to post a motion .GIF I’ll post a little animation I made long ago. It uses this concept of sails going backward, but they fold down at the back and are brought back forward on a conveyor.

    1. drsteevo says:

      You could post it on http://photobucket.com/ or http://www.flickr.com/ and post the link here. I don’t think you can embed photos.

  102. Rick Cavallaro says:

    @ spleen

    > the sail moves less or equal too but not faster than the speed of the down wind velocity…

    Nope. The whole thing can move much faster than the wind. In fact the direct downwind component of both the hull and sail can be much faster than the wind.

    1. spleen says:

      I get that the hull moves faster, but the angle of a attack of the blade denotes that against the direction of the air particles is either the same at 100% efficient or less. Same idea as you have with posting the GIF image of a sail on a conveyor.
      As mentioned earlier by another poster, just levers and sail (energy collection) components. Rotating blades of a propeller are both. Unless of course I am so fixated that I am missing something… will think about it more.

      1. spleen says:

        When you consider that the hull speed of a yacht can vary due to the angle of attack (variable) hull length (drag constant) wind speed(lets fix that at 25 km/h) – and if you can appreciate that the hull speed (in it’s magnitude) can exceed the wind speed due to those factors, then it is not such a leap to consider, the angle of attack of a fast rotating propeller which is pretty much doing the same.

        Rick this has been a mind F(thought) for the past couple of days but this answer does kind of feel right.. I will do a couple of animations and calcs over the next few days.. very cool examination.

      2. Rick Cavallaro says:

        >> I get that the hull moves faster, but the angle of a attack of the blade denotes that against the direction of the air particles is either the same at 100% efficient or less.

        Ahh… but now we’re talking about the apparent wind rather than the true wind. Yes, the apparent wind has to hit the sail, prop, wing, from the upwind side. We make this happen through a combination of blade angle and sweeping the blade across the flow – even when the blade itself is moving downwind faster than the wind.

    2. spleen says:

      If the blade rotates at an angular velocity of 600rpm, has an angle of attack of 15 degrees at radius 2m the linear velocity of the surface area of the blade is

      10 rps * 4* PI = 12.56m/s
      * sin (15) = 3.252m/s
      or 11.7 km/h

      If the vehicle is traveling at 15kmh and the prevailing wind is 10km/h then the surface of the blade at 2m radius is traveling 6.7km/h slower than the wind

      15 – 11.7 = 3.3km/h which is 6.7 km/h slower than the wind.

      If the vehicle is being propelled by the force of the wind acting on the sail (surface of the blade), we can see that the linear velocity of the blade surface is less than the speed of the wind due to the drag of the hull and the mechanical losses.

      1. Rick Cavallaro says:

        >>
        If the blade rotates at an angular velocity of 600rpm, has an angle of attack of 15 degrees at radius 2m the linear velocity of the surface area of the blade is

        10 rps * 4* PI = 12.56m/s
        * sin (15) = 3.252m/s
        or 11.7 km/h

        If the vehicle is traveling at 15kmh
        << Nope. You left the 15 kph that the prop is moving forward out of your first formula.

        1. spleen says:

          I did look at it again but cannot see where it is wrong

          Vehicle and propeller travel at 15kmh(+)
          Linear speed of the blade is 11kmh(-) (relative to the vehicle)
          Relative speed of the blade to the earth 4kmh(+)
          Wind speed is 10kmh(+)
          Therefore the linear speed of the blade is 6kmh slower than the wind while the speed of the vehicle is 5kmh faster than the wind.

          1. Rick Cavallaro says:

            >>
            Vehicle and propeller travel at 15kmh(+)
            Linear speed of the blade is 11kmh(-) (relative to the vehicle)
            Relative speed of the blade to the earth 4kmh(+)
            << Why would you subtract the blade tip speed from the vehicle speed? The blade tip is moving perpendicular to the vehicle in the vehicle frame. The blade tip speed will be: b.t.s. = sqrt(15^2 + 4^2) = 15.5 kph >>
            Wind speed is 10kmh(+)
            Therefore the linear speed of the blade is 6kmh slower than the wind while the speed of the vehicle is 5kmh faster than the wind.
            << Nope. Blade tip speed is 5.5 kph faster than the wind in this scenario.

          2. spleen says:

            Consider the blade slicing through the air at an angle of 15 degrees to the direction of rotation. At 600rpm and about 2m radius it generates a thrust velocity of approx 11kmh if it was very efficient. This is because at any given vector perpendicular to the rotation and at that radius, the leading edge to trailing edge moves at 11kmh. This is happening in a direction opposite to the direction the vehicle is travelling which is why I subtract the 11 from the 15.

          3. spleen says:

            the blade is just your sail on a conveyor belt moving at 11kmh

          4. Rick Cavallaro says:

            >> I think that the linear velocity of the blade is slower than the wind speed.

            I’m going back to this. Perhaps you’ll have to tell me what you mean by the “linear velocity of the blade”. I thought you meant the velocity of a point on the blade tip. Now I’m guessing that’s not what you’re talking about.

          5. spleen says:

            yeah I am sorry Rick, my terminology is probably incorrect. I am talking about the linear speed or thrust of the blade. Imagine putting a rod on the leading edge of the blade and fixing the x and y position of the rod while allowing the rod to move in the z direction when the blade rotates. If you imagine the propeller to be a screw thread, it is the speed the nut would move along the axis.

            This means that the surface of the blade is constantly moving away from the vehicle at 11km/h. It is this surface that the wind pushes against and the net result minus losses is the speed of the vehicle.

          6. Rick Cavallaro says:

            Ahhhh… now I see what you’re saying. I probably should have figured that out more quickly.

            You’re onto the most basic parameter that defines the cart’s performance. It’s what we call the “vehicle speed ratio” or VSR. The VSR is simply the distance the prop would advance in a single revolution with no slip divided by the distance the wheels would move forward in the same single revolution of the prop.

            This parameter defines whether we have an upwind or downwind vehicle as well as its “design speed” as a function of wind speed.

            For a VSR < 1 we have a vehicle that goes DDWFTTW. For VSR > 1 we get an upwind vehicle. As the VSR approaches 1.0 we get a vehicle design speed that approaches infinite times the wind speed – either upwind or downwind depending on whether VSR is < or > 1.0

            Of course you can’t simply choose a VSR of 0.9999 to get an incredibly high multiple of wind speed. The closer you set the VSR to 1.0, the smaller your losses had better be if you want the vehicle to work at all.

          7. spleen says:

            What I am suggesting is that the wind is that rod pressing up against the surface of the blade at 10km/h. So the net result at 100% efficiency is the vehicle having to travel at 21kmh.

          8. vf says:

            He means the intersection of the blade with a line parallel to driving direction. This intersection moves slower than wind, so the air along that line hits the back side of the blade, and pushes it forward.

            This line of air parallel to driving direction is shown here:
            http://www.youtube.com/watch?v=FqJOVHHf6mQ

          9. Rick Cavallaro says:

            It is true that the AOA and rotation rate of the blade is such that the free-stream encounters the back side of the blade. But to say the wind hits the back side of the blade and pushes it forward is not accurate.

            It’s slightly better to say the blade hits the air molecules and pushes them backward – but even that’s not right, because that’s not quite how an airfoil works. Generally it has much less to do with molecules hitting a surface and bouncing off, and more to do with the way the airfoil affects the flow around it (and therefore the pressure distribution around it).

          10. spleen says:

            …or you have a giant screw slicing at 11km/h through a river of air with flowing at 10km/h. When the bow wave and other forces balance the thrust caused by the screw thread wanting to go faster than the vehicle, you will have reached the vehicles terminal velocity.

          11. vf says:

            >> But to say the wind hits the back side of the blade and pushes it forward is not accurate.

            I said “air” not “wind”. Air is an object. Wind is the motion of air relative to something. Pushing means exerting a force on something. The air does exert a force on the blade with a positive forward component, so it does push the blade forward

            >> It’s slightly better to say the blade hits the air molecules and pushes them backward

            That is the same due to Newtons 3rd Law: If the blade pushes the air backward, then the air pushes blade forward.

          12. Rick Cavallaro says:

            You said “the air along that line hits the back side of the blade, and pushes it forward.”

            And that’s not right.

            What we have is a spinning prop pulling itself through an airmass in which it’s submersed. You’re free to try and see it as something else. But I’m rapidly losing interest in this word game you seem to want to play.

          13. vf says:

            >> You said “the air along that line hits the back side of the blade, and pushes it forward.” And that’s not right.

            I think you just misunderstand it. I mean the same as what you said here:

            >> It is true that the AOA and rotation rate of the blade is such that the free-stream encounters the back side of the blade.

            But for small AOA this is too simplified as you already noted.

            >> What we have is a spinning prop pulling itself through an airmass in which it’s submersed.

            Sure, no contradiction here. This just a higher level of abstraction. On the atomic level though there is no “pulling”, just “pushing”: Propeller pushes air back, air pushes propeller forward.

          14. Rick Cavallaro says:

            I think perhaps I did misunderstand your meaning. It’s sometimes tricky to discuss these things without colored markers and hand waving.

            JB and I just discussed the potential meanings in some detail. What I’m suggesting is that no one would say when playing T-ball that the ball hits the bat and pushes it backward. That’s exactly the case of the air interacting with the prop on an airplane – or on our cart when going downwind faster than the wind. Admittedly, this largely comes down to the notion of thinking of the air as being stationary and the prop as moving (it actually IS turning). And obviously we can’t make such absolute distinctions in reality.

            But yes, it seems now that you and I are on the same page. I believe we both understand that the flow and interaction of air and prop on our cart is exactly the same as that of the air and prop on an airplane in flight (with or without wind).

            I apologize for any misunderstanding and/or shortness.

      2. spleen says:

        10 rps * 4* PI = 12.56m/s
        * tan (15) = 3.365m/s
        or 12.1km/h

        1. Rick Cavallaro says:

          You’re still forgetting that the prop is on a vehicle that’s moving forward.

  103. spleen says:

    Getting more absorbed in my perception but also thinking that a balance needs to be proved. let me attempt the simpler static math tomorrow and see what I come up with..or do it yourself would be also..still think yr great idea works!!

  104. phemto.myopenid.com says:

    I think where many of the arguments go astray is in the assumption that the belt/rod/chain between the prop and the wheels is transmitting the drive torque that make the cart move forward. In either direction, you’re going to run afoul of thermodynamic laws.

    I’ve done some thought experiments that have convinced me that the blade tips are acting as sails, not propellers. It’s well known that a land yacht can go much faster than the wind, but not directly with it. So how does this work? It’s a cheat. The cheat is that the sails, in this case, are being constantly moved sideways (think the top and bottom of the prop). The drive train isn’t carrying the driving force for the cart, it’s encoding the lateral speed of the blade tips so that the apparent wind direction that they experience is always appropriate to obtain forward lift. This could be done without a propeller. That just happens to be the simplest way to do it.

    If it’s not too self-promoting. I’ve posted my thought experiments (with diagrams) on dadstruction.com. If there is a way to post the diagrams here, I’d be glad to do it.

    1. vf says:

      >> I’ve done some thought experiments that have convinced me that the blade tips are acting as sails, not propellers.

      This is not a contradiction, just two different abstraction levels:

      1) The individual blades act as sails, and the wheels just constrain them to certain path (like a keel) which is not directly downwind.

      2) The entire rotor spins opposite to the moment created by the air, so it is a propeller. The wheels generate a greater moment than the air in the direction of the rotors rotation, so they are driving the rotation of the rotor.

      >> I’ve posted my thought experiments (with diagrams) on dadstruction.com.

      Yeah, this extending catamaran is one way to show that DDWFTTW is possible, based on stuff that iceboats do easily: downwind VMG > windspeed. The handwaving answer to this, from the “Prangs” is: “You could never build such a connection in reality, it would break” :-D

      Here some related animations:
      http://www.youtube.com/watch?v=IMEerIkOVZ0
      http://www.youtube.com/watch?v=UGRFb8yNtBo

    2. Rick Cavallaro says:

      >> the blade tips are acting as sails, not propellers.

      When a sailboat is going fast on a broad reach its sail is acting as a propeller – not a turbine.

  105. drsteevo says:

    Rick,

    If you turned this sled around, would it travel forward straight into the wind? Seems like it might if you prop to efficiently collect the wind like a windmill.

    1. drsteevo says:

      That should read: “Seems like it might if you select a different prop to efficiently collect the wind like a windmill.”

      1. Rick Cavallaro says:

        If we change out the prop for a turbine it will do exactly that. We plan to go directly UP wind faster than the wind next summer.

      2. Alsee says:

        The final answer is “yes” you can use this basic design to go up wind. However you overlooked a detail needed to get there. Note that the wind tries to turn the propeller one way, while the wheels try to turn it the opposite way. The wheels win out because of the gear ratio gives the wheels greater leverage. If you simply flip the cart around the wheels are still going to win that battle. The wheels are going to drive the propeller the wrong way and you’ll still go down wind. You simply need to change gears to give the wind greater leverage to beat out the wheel’s force. Then the wind will drive the prop as a turbine (windmill). The prop will then have leverage to drive the wheels pulling you up wind. And as you noted you want to adjust the propeller blade angles to run efficiently in windmill mode. If your design is efficient enough and your wind drag is low enough you could go upwind at double or more the speed of the wind.

  106. ausrandoman says:

    I’d like to have data on the craft
    – the gear ratio, propeller rpm divided by wheel rpm
    – drag coefficient of the craft
    – rolling resistance
    – propeller thrust vs rpm for various ground speeds

    I’d also like to have data from the trials
    – the wind speed
    – the maximum ground speed achieved
    – a plot of ground speed vs distance or time, from rest

    Where can I see this data?

  107. Rick Cavallaro says:

    >> Where can I see this data?

    Download the reports from NALSA.org. If there are specifics that aren’t there let me know what they are.

  108. bryanM says:

    It would appear a moving river is just like the treadmill and that in theory one could build a boat that could travel upstream (and presumably downstream faster than the river). No idea how big the propeller would have to be but if one could be built it would definitely create a lot of interest and might even be commercially viable (it would be a great tourist attraction for a river boat operator if nothing else). I bet there are a few people who would say it is impossible though :-)

    1. AllParadox says:

      >>>
      It would appear a moving river is just like the treadmill and that in theory one could build a boat that could travel upstream (and presumably downstream faster than the river).
      <<< The treadmill is operating in what is in effect a dead calm inside the house. The differential between the calm air and the treadmill is what powers the cart. I like to go canoeing on our local rivers here. Generally, the conditions are anything but dead calm. Dead calm only tends to happen right at dawn or sunset. The amount of fall to the river is unlikely to be enough, even in a dead calm, to provide enough power to drive upriver. I have towed an empty canoe upriver in Missouri's fast streams. It was exhausting. Hydrodynamic drag is much stronger than aerodynamic drag, owing to the high relative density of water. _________________ AllParadox

      1. Subduction Zone says:

        No, the treadmill operates in a way that exactly reproduces the forces on the cart when it goes from the speed of the wind, that speed is represented by the hand holding the cart in place until it is rolling at the same speed as the belt and then releasing it, to faster than the speed of the wind, that is shown by the cart advancing on the belt. Remember in physics all forces are local. That means the cart reacts the same to the wind when moving at wind speed, as it does when held on the treadmill until its wheels are up to speed and then released.

        1. AllParadox says:

          Whether or not I understand the cart on the treadmill, and I do, the question we are discussing is how to use a wind propeller to propel a boat upstream.

          If there is not a dead calm as at dusk, a nearly magical time on the river, then there is a breeze blowing.

          Rivers wander. Eventually, around some bend, the breeze will be blowing the same direction, and nearly the same speed, as the water in the river.

          There will be no velocity differential, and thus no power. If Mr. Murphy’s law is correct, that will happen just as you begin to approach the waterfall, or the vortex, or some other unpleasant condition or obstruction.

          From where I sit, that is bad JuJu.
          _____________________
          A different scenario:

          On a lake, the only flow of water is the relative flow – your speed relative to the water.

          If you are canoeing on a lake, then the pernicious wind is a problem.

          When it is coming at you, bow on, it can be a long, hard paddle to get where you need to be.

          How nice it would be to have a device that captures that wind energy and drives you into the wind. Yes, a sail would work, if you are an accomplished sailor and know how to practice the tacking technique, and happened to bring along the sail, mast, rudder, and side-board.

          For us less-accomplished navigators, something that you could just sit back and aim, and let the wind-power drive you into the wind would be wonderful.

          I have developed that thingie.
          __________
          AllParadox

          1. Rick Cavallaro says:

            >> I have developed that thingie.

            Please show us that thingie. Is it in use? Patented? Is it a turbine that powers a prop under water? Inquiring minds…

          2. AllParadox says:

            I will probably get in trouble for breaking all the rules.

            Rick, please email me directly at topgunis at sbcglobal dot net.
            __________
            AllParadox

          3. Rick Cavallaro says:

            Done.

  109. Rick Cavallaro says:

    >> It would appear a moving river is just like the treadmill and that in theory one could build a boat that could travel upstream (and presumably downstream faster than the river).

    Correct. In fact we use that argument to show that the treadmill is a valid experiment. If you were on your sailboat in a heavy fog, but felt a breeze on your face, you could certainly tack into that breeze – right? But while you’re tacking upwind into that breeze, the fog might clear, and you’d learn that you were being carried down the river on a day that actually had no wind at all. The breeze was entirely a result of the current of the river – but now you’re tacking into that breeze and therefore going down-current faster than the current.

    >> I bet there are a few people who would say it is impossible though

    We’ve encountered more than a few.

  110. Rick Cavallaro says:

    >> Violation of the original terms of the challenge

    Wrong again – in so many ways. First of all, you never accepted my challenge, nor did you offer me a challenge. Secondly, our ratchet exists to allow us to push and tow the cart around without the prop spinning. The ratchets were engaged during the runs. Our model carts have no ratchets (because we can pick them up and carry them around) – yet they still work just fine. These ratchets are a convenience for handling the cart and to allow us to brake at the end of the run without putting reverse loads on the transmission. They have nothing to do with the terms of the challenge you had with a different person.

    >> … which Rick told me was merely a reflection of my stupidity and lack of understanding of aerodynamics.

    Perhaps I should post the entire email thread between you and I. I’m pretty sure I never used that word. You’re the one that seems intent on making this abusive – and on getting it wrong at every turn.

    >>I look forward to an admission from Rick that he had to add the ratchet to make the cart work. I also look forward to the $10,000 he pledged, although not with much expectation.

    You simply refuse to read anything at all on this topic – huh? The arrangement between you and I was to be a bet. I offered you $100K against your $10K (as Mark Frauenfelder noted in his article). You refused the bet for reasons that only you know and we can only guess – even though it was 10:1 in your favor.

    You’re darn right that you’ll get no admission from me that I had do use a ratchet – because I did NOT have to use a ratchet. If you even made an honest attempt to understand this thing you would understand that.

    Instead I get exactly what I got throughout our email chain – evidence that you didn’t even read what I wrote. Instead you asked what made me so arrogant as to think I could write an article. What I wrote was an article to correct the article you wrote. And unlike you, I did my homework. What you did was to attack a man that you never even bothered to contact. That’s not journalism!

    >>Lastly I understand that the propeller on the cart has variable pitch. This, too, is a violation of the terms that I set out.

    There are three problems with that:
    1) You did not set out any terms for me. You’re in no position to do so. You refused to take my bet.
    2) Variable pitch is not required – we proved that with the small carts and with the manned cart in Ivanpah.
    3) If you understood this cart you would understand that this doesn’t violate the terms you set out for YOU to give $10K to Peslel. The only terms involving me paying anyone were based on a bet that you refused – and would have lost.

    If you insist on continuing to misrepresent our conversations I will post our entire email exchange here.

  111. Subduction Zone says:

    Personally, I would love to see those emails. I know that people who are wrong often think that people who are right are arrogant, having been on the wrong side of this argument. Unfortunately my opinion of Platt keeps going down. You would think he would at least ask for an explanation of why the treadmill tests are extremely strong evidence that the cart goes faster than the wind, the evidence being that it is a case of DDWFTTW. If the vectors involved are too complex it is explainable in everyday English. In fact it is strange that none of the deniers want to engage on the treadmill tests.

    In that spirit I make an open offer to Charles Platt, I will gladly ask you a series of yes/no questions that show the treadmill to be a case of DDWFTTW. Sometimes the Socratic method is the best.

  112. Rick Cavallaro says:

    >> If you understood this cart you would understand that this doesn’t violate the terms you set out for YOU to give $10K to Peslel. The only terms involving me paying anyone were based on a bet that you refused – and would have lost.

    @ myself

    I blew it here. Platt offered Peslel $1K if Peslel could demonstrate DDWFTTW under Platt’s terms (which Peslel did).

    The arrangement between Platt and I was to be a 10:1 bet in Platt’s favor – my $100K against his $10K.

    I simply made an error when I spoke of Platt’s arrangement with Peslel being $10K.

  113. ThinAirDesigns says:

    Yowza Charles — through your lack of research on the vehicle you just continue to dig your hole deeper.

    I can only assume that this lack of research is willful since rather than simply asking us questions about the vehicle (like you should have done with Goodman to start), or read the extensive construction blog (that you are well aware of), you just continue to make stuff up. There are so many things fundamentally wrong in your comments that it’s hard to know where to start.

    Charles:
    >>I recently learned, from a source whom I
    >>trust, that Rick’s vehicle has a ratchet …

    The ratchet in question is no secret — it’s fully documented on the construction blogs, pictures and everything.

    http://www.fasterthanthewind.org/2010_04_01_archive.html
    http://www.fasterthanthewind.org/2010/05/higher-torque-upgrades.html

    Let me point something very important here — the ratchet has absolutely *nothing* to do with normal operation of this craft. The ratchet has *not once* released and gone ‘click, click, click’ during the running of the vehicle. Easily demonstrated physics dictates this and video documentation shows it.

    Here’s the purpose for the ratchet on the prop shaft — if the driver of the Blackbird has to make a panic stop, the spinning mass of the propeller would try to keep it spinning while the brakes (on the drive axle) try to bring it to an immediate stop. For safety reasons, the brakes are much more powerful than the propulsion system (just as on any car). We wanted strong brakes (for obvious reasons), but we don’t want our transmission to snap just because the driver had to slam on the brakes.

    Without the ratchet, the mass of the prop would backdrive the transmission and overload it. With the ratchet, in a panic stop the vehicle can be brought to zero quickly, while propeller can at its own pace, coast to a stop independent of the vehicle movement — just going ‘click, click, click’ until it RPMs are zero.

    As an additional plus, the ratchet allows us to tie the prop down and push the vehicle around by hand (backwards) without the prop spinning and getting in our way or hitting something.

    Again, since under normal operation the torque on the shaft is always in the same direction and the ratchet is FORCED by this torque to remain engaged the entire time during and run.

    We’ve never had to make a panic stop so it’s never even been used for it’s primary purpose.

    Charles:
    >> [this ratchet] allows the propeller to
    >> spin freely (i.e. not driving the wheels)
    >> when the wind blows from behind.

    First, the prop NEVER tries to drive the wheels forward under ANY circumstances other than panic stop, so this parenthetic notion you put forth above (i.e. not driving the wheels) is ALWAYS true — no ratchet needed to influence that.

    Second, since I’m familiar with every single observer who was at every one of our tests, I can assure you that you are misundstanding whatever it was that they told you.

    Ask ANY of those observers this question:

    “Before and during each run that you observed, when the Blackbird was sitting still in the tailwind or running below windspeed (i.e. “wind blowing from behind”):

    A: was the propeller spinning freely as Charles Platt asserts the ratchet allowed?

    or

    B: was the propeller’s rotational speed directly tied to and spinning directly proportional to the rotational speed of the wheels?”

    In other words, “did the prop every spin freely in the early stages of the run, or was the propeller always tied directly to the wheels?

    Their answer to a man will be “B”.

    I also know that each of them is willing to talk publicly about their observations and they know we would encourage them to do so and actually hope that they would.

    In light of that I encourage you to email your observer that question and ask him to respond with “A” or “B”. and post the answer here. Thanks.

    Let’s be clear — I’m not accusing you of lying here, but I am most assuredly stating that just like your continued misunderstand of the fundamental operation of the craft, you have misunderstood the operation of the ratchet. You owe it to yourself, your readers, Pelesl (whom you owe $1,000 to as he fulfilled your original requirements, word for word btw) and especially the observer you have just misrepresented to get and post the observers answer to that above question. What it will show is that again are attempting to defend yourself through lack of knowledge and understanding.

    Additionally, as someone else already posted, there is plenty of video around showing NALSA observed runs (in Ivanpah) where the prop shaft ratchet was NOT installed (it broke just before we left — again documented in the construction blog) and there was NO variable pitch hub in use (not built yet — again documented in the blog) and still we went near 3x the speed of the wind from a standstill. These points themselves demonstrate the fallacy of your position.

    Your lack of knowledge on this topic, and your willingness to make stunningly false assertions based on that lack of knowledge is a serious setback to any credibility you might wish to have as a journalist. You really should step up to the plate, do the sort of research the your Editor-in-Chief did and return wiser and accurate. Your not doing yourself favors at this point.

    JB

  114. Neil says:

    Rick,

    I’m thinking if some clever inventor thought of this in 18th century, we would still have canals/rivers all over the east coast, with barges power by first paddle wheels followed later with turbine/propeller engines. They would take energy from the moving water and a side rail or over head cable to connect the wheel to land. I’m thinking the moving water would provide far more energy density vs. the wind, hence much smaller turbine/propellers could be used. Speeds much greater than the speed of the moving water would easily be achieved with the available energy. (I’m assuming water in canals is always moving, filled on one end, used in the middle to power the locks, then empties on the other end).

    Question, which is more efficient for extracting wind/hydraulic energy, a turbine, or a propeller?

    Do you think a propeller/turbine engine would be practical on say the Mississippi river? My guess is you could move up/down river, much faster than the water speed. Of course the impractical part is you’d need a rail or cable to grab onto to spin the wheel that turns the propeller.

    –Neil

    1. AllParadox says:

      Please forgive me for answering where Mr. Cavallaro has not, but I believe I know the answer.

      There is a tremendous amount of energy available in flowing water.

      If there is a cable tied to land to provide a physical reference, perhaps with a driven wheel in contact with the cable, there is no good reason that an adequately hydrodynamic body could not be powered upriver on most rivers.

      If the river has a reasonable flow, and you cannot get your craft to travel upriver, you are not trying hard enough.

      Maximum speed would depend on many factors.

      There are still ferries in Missouri that operate without a motor.

      They use a cable across the river, and the ferry is a simple barge.

      The barge is connected to the cable with another cable. There is a crank and a bracket holding the two cables together. There are rollers that allow free movement on the first cable. The second cable is tied at each end to the ends of the ferry. To cross the river, the operator cranks the second cable – rolls along the second cable – so that the front of the ferry is further upriver than the back of the ferry. The flow of the river pushes the ferry across. To return, the operator cranks the second cable the other way. Now the back of the barge is further upriver than the front, and the river pushes the barge back to the origin. Ingenious.

      When the river floods, a frequent occurrence, debris carried by the river tends to tear out the cable, so it is not a perfect solution.
      ___________
      AllParadox

      1. Rick Cavallaro says:

        >> Please forgive me for answering where Mr. Cavallaro has not…

        D’OH! It looks like I missed that one – but yes, you got it right.

        >> If there is a cable tied to land to provide a physical reference, perhaps with a driven wheel in contact with the cable, there is no good reason that an adequately hydrodynamic body could not be powered upriver on most rivers.

        Yup. Check out the Brennan torpedo invented over 100 years ago: http://en.wikipedia.org/wiki/Brennan_torpedo

  115. Robin Debreuil says:

    Great project! As the scientific process goes, you have a theory, test it in a reproducible way, publish the result and steps to reproduce, and then people can try to duplicate it or fail to. It seems you’ve done your part, any one who is skeptical (a good thing) needs to attempt to reproduce your results. One ‘I’m unable to reproduce X’ is worth an infinite amount of mockery and learned opinion here…

    I really enjoyed reading the whole thing, thanks for the multiple explanations in the comments. I think I’m starting to get how this is possible in my head. My imaginary experiment goes like this:

    Imagine there is a motor on the wheels, and it turns at a very high rate (say 10000 rpm). This turns the prop, which makes the car move forward (by thrust). Initially, the prop is very hard to turn, because it is displacing a large amount of still (relative to the cart) air. As the cart speeds up, the prop is easier to turn, because the rmp being equal, it needs to displace less air (the amount the cart moves in t seconds is the amount the prop didn’t have to displace).

    As it hits its maximum speed for 10000 rpm, the prop now turns relatively easily, and the motor doesn’t have to do as much work due to the cart’s inertia. If there was zero loss in the whole system, maintaining this state would require zero energy. That would be true at any rmp of the prop.

    Of course there is loss, which is why this doesn’t work as is. The crucial point though, is there is also gain in this system if there is wind. The wind is essentially just making the prop easier to turn at the outset, which means the wheels have to do less work to hit any given rmp. If the wind is stronger than the loss in the system, it is the equivalent of a lossless vehicle with an easier than normal to turn prop (which is like adding an engine of that power to the wheel).

    That is why it can’t work without wind, and why it is able to harvest the wind/ground difference regardless of the speed it is going.

    Ok, at least that is how it satisfies my sense of it, fully aware I may be missing parts of the picture or plain wrong, and happy to be corrected : )

  116. Robin Debreuil says:

    If I understand it right, the opposite should also be true, that if the prop powers the wheels, it should be possible to move against the wind. While this would probably be harder to make a efficient machine, maybe it is easier to intuit.

    A large windmill creates a tremendous amount of energy. if it was unreasonably light, its easy to visualize how the power could enable to to move along a rail. As it moved faster it would gain more wind energy. Because it is always moving the same direction against the wind, it is easier to see that the initial wind conditions are always the ‘extra’ part that gets added to overcome losses and have a net energy gain.

  117. Rick Cavallaro says:

    >> I really enjoyed reading the whole thing, thanks for the multiple explanations in the comments…

    Thanks so much. It’s really nice to find people that enjoy the project and really want to know how it works.

    With regard to your explanation, I think it’s easier not to complicate it initially with the acceleration. It’s better to first see that it can maintain it’s speed once it gets into a steady-state downwind faster than the wind. Then we can try and prove to ourselves that it will self-start in a tailwind – although that can be a bit tougher to do on paper.

    The idea that the prop gets easier to turn when the cart gets up to speed may be misleading. What’s important isn’t how hard or easy it is to turn the prop, but rather how much thrust it produces relative to how hard it is to turn. With no tailwind, the amount of thrust would never equal the amount of retarding force created on the wheels in order to turn the prop. Thus the cart would coast to a stop if you dragged it up to speed.

    But with a tailwind, the same amount of retarding force on the wheels produces more than enough power to create the thrust to win the battle against that retarding force.

    The reason we can get more thrust for less input power when we have a tailwind is simply because power = thrust x velocity. In the tailwind case the velocity of the wind over the cart is smaller than the speed of the wheels over the road. Since power = thrust x velocity, and we have a lower velocity the prop is working on, it can generate more thrust for a fixed amount of input power.

    1. Robin Debreuil says:

      Yes, that makes sense in a more direct way. For me the block was, how can it use the wind if it is going faster than the wind – so I needed an elaborate construct to visualize that. Now that I’ve digested that, I’m able to discard it for your much simpler explaination. Specifically:

      “power = thrust x velocity, and we have a lower velocity the prop is working on, it can generate more thrust for a fixed amount of input power.”

      Really want to build one of these on some scale, will be pouring over your models and plans… Thanks again for all this : )

      1. Rick Cavallaro says:

        >> Really want to build one of these on some scale, will be pouring over your models and plans… Thanks again for all this : )

        That would be great. Take a look at my build videos linked above. Be sure and let us know how it goes.

  118. vf says:

    @ Charles Platt

    >> Rick’s vehicle has a ratchet that allows the propeller to spin freely (i.e. not driving the wheels) when the wind blows from behind.

    Mr. Platt I have two questions here :

    1) Have you watched the first three videos linked under the article?

    2) Do you see the the propeller spinning freely at the start when the wind blows from behind, as you describe?

    >> A ratchet allows the propeller to spin freely without exerting a reverse force via the wheels when there is a following wind.

    This is not what I see in the videos. The propeller is obviously NOT spinning freely with an apparent tailwind.

    Instead the propeller is in fact, creating a braking force via the transmission at the wheels. However the gearing is such that the forward pointing aerodynamic force is greater that this braking force, so the cart accelerates forward.

    Hint: The ratchet works the other way around to what you assume, and it doesn’t free wheel at all during the acceleration from 0 to above windspeed.

    >> Lastly I understand that the propeller on the cart has variable pitch.

    The prototype tested at Ivanpah (video 2 & 3) didn’t have a variable pitch. Yet it accelerated from 0 to above windspeed. Something that you continue to claim is impossible.

  119. Neil says:

    @ Rick

    Of all of your observations, this one in your reply to Jennifer is the most profound “interestingly enough, the faster you go over the ground the more of that energy you can collect. For this reason you can harvest several times the power of a stationary windmill of the same diameter… energy density available is constant, but as described above, you can harvest more of it by going faster” .

    Do you see a future where wind farms use propellers on tracks for energy production? Could the use of propellers be the key to making wind energy competitive with fossil fuel? Maybe these are the patents you are working on?

    Directly Down the River, Faster than The River, Powered By the River, Powering Electric Generation
    The hydraulic potential for energy production could be even bigger. I’ve seen talk of loading up slow moving rivers with slow moving turbines. Given your observation above, I envision fast moving turbine/propellers hybrids zipping up/down slow moving rivers while generating power for a fraction of the cost of a large bank of slow spinning turbines.

    Another application; propeller generators (on under water tracks) zipping up and down tidal rivers as the river goes in and out with the tide.

  120. Neil says:

    @ Rick & Company

    One brain teaser I can’t wrap my head around. I presume wind powered vehicles could operate in either propeller or turbine mode while traveling at a diagonal to the wind, but which mode (propeller or turbine) would provide maximum speed?

    Which mode has the best potential for energy production diagonal to the wind? Alway being on the diagonal also solves the problem of optimizing blades for turbine or propeller. No need for dual mode operation, just pick the best mode, and zip side to side. Ditto for hydraulic energy, turbine or propeller for diagonal power sources?

    Another energy production idea, turbines on kites, zipping side to side collecting energy from a larger volume of wind energy with a relatively smaller turbine, or propeller for that matter.

    Seems there are endless ways to utilize propeller powered wind/water machines.

  121. Rick Cavallaro says:

    >> Do you see a future where wind farms use propellers on tracks for energy production?

    Not likely on tracks – though that would work. You’d probably use long oval tracks in that case. I think one likely scenario for wind power involves “flying windmills”. The fact that we asked Joby Energy to sponsor us wasn’t purely by accident. Joby is one of the leading companies developing such technology.

    In addition to putting the turbine at altitude where the winds are steadier and stronger, you can take advantage of the very concept I described above.

    >>Could the use of propellers be the key to making wind energy competitive with fossil fuel?

    Possibly. In the “flying windmill” case, you’d typically operate in turbine mode rather than propeller mode, but the principle actually works both ways. I personally think that wind will be one significant component of the answer to the issue of Peak Oil. I think the secrets to success will be higher fuel prices, possibly government incentives, and economies of scale. When people believe things are possible – most things are.

    You may have seen that our other sponsor (Google) has backed the offshore windfarm backbone:
    http://gigaom.com/cleantech/google-backs-offshore-wind-farm-backbone/
    Our sponsor within Google happens to be a guy that is extremely sharp and extremely interested in the solution(s) to the energy problems we face.

    By the way, if you want a real good scare read “Out of Gas” by Caltech professor David Goodstein.

    Incidentally, have you noticed that Google’s market cap has increased by $60 billion since its low the day before Blackbird set its record? : )

    >> Maybe these are the patents you are working on?

    Nope. All my patents so far relate to broadcast enhancements like the virtual first down line, virtual strike zone, etc. There is one patent that JB and I are working on outside of Sportvision that relates to energy, but isn’t related to props and turbines.

    1. russingram says:

      Hey, I think I remember you doing a presentation to my MBA class at the University of Florida! Good job on this forum, and good job on the presentation!

      1. Rick Cavallaro says:

        Was the presentation relating to the work we do at Sportvision (computer generated enhancements for sports broadcasts)? If so I think that might have been Bill Squadron. He started the company with us back in 1998.

        Thanks for your comments.

        1. russingram says:

          Yes, it was in regard to Sportvision – it must have been Mr. Squadron, then.

  122. Rick Cavallaro says:

    >> I presume wind powered vehicles could operate in either propeller or turbine mode while traveling at a diagonal to the wind, but which mode (propeller or turbine) would provide maximum speed?

    You can use turbine or prop mode for traveling diagonally downwind (although turbine mode in this case is sort of a degenerate case). For going diagonally upwind (as in your example below) you have to go with turbine mode.

    >> Which mode has the best potential for energy production diagonal to the wind?

    This depends on how fast you can go, and how efficient your machine is generating power, aero drag, rolling resistance, etc. I have some charts and formulas that get into this in a paper I’m submitting to a peer reviewed journal.

    >> Alway being on the diagonal also solves the problem of optimizing blades for turbine or propeller.

    Yes, but this will always have you on an upwind tack. So you’ll always be in turbine mode (which is still OK). It actually has other distinct advantages. For example, you could effectively handle a MUCH wider wind range. The greater the wind the slower you drive the cart. But the disadvantage of complexity is a significant one.

    >> Another energy production idea, turbines on kites…

    Check out our primary sponsor: http://www.jobyenergy.com/

    Incidentally, when I first called Joby requesting they sponsor our project, I wasn’t able to get through 2 minutes of my pitch. They said yes before I could tell them all the great reasons for sponsoring us.

  123. Neil says:

    @ Rick

    I figured you’ve already though about this kite potential. I’m sure I saw this one in popular science when I was a kid.

    I still think the hydraulic energy source could be another big application. Slow rivers could give up energy more cheaply with fast moving propellers or turbines.

    Its amazing humans didn’t figure this one out hundreds of years ago. I imagine the Romans, masters of hydrology using aqueduct canals to power barges into and out of Rome. There would a track along the side for the wheel connection to land, with the propeller/turbine (paddle wheel) in the water pulling down stream, and powering the wheel for upstream travel.

  124. Neil says:

    @ Rick

    Fantastic answers. My goof, I meant perpendicular to the wind; turbine or propeller?

    IMHO, Peak oil/fossil-fuel/coal is not the primary issue. The problem is that we have too much of it. The planets going to be dead before we can burn through half of it.

    Best Regards,
    Neil

    1. Rick Cavallaro says:

      >> I meant perpendicular to the wind; turbine or propeller?

      Hmmm…. If you’re running perpendicular to the true wind you’ll always be experiencing a relative headwind. Without straining my brain too much I’m going to guess that you might do just as well with a fixed sail in this condition. There’s probably no need to spin your sails when you’re translating them quite nicely just by following the path perpendicular to the wind. In this case you just attach the generator to the wheels and convert wind to electricity. For a given vehicle and wind there would be an optimal speed. In a sense you would have just “unwrapped” a traditional windmill in this case.

      >>IMHO, Peak oil/fossil-fuel/coal is not the primary issue. The problem is that we have too much of it. The planets going to be dead before we can burn through half of it.

      I share your concerns. But I actually think we’ve used very nearly half of our fossil fuel reserves if we haven’t hit that point already. And the half-way mark is where things get very scary. It’s true that if we develop effective alternative energies that are cheaper than fossil fuels it will drive the price of oil down. This will tragically insure that we do the most damage possible with the available fossil fuels.

  125. Rick Cavallaro says:

    >> Its amazing humans didn’t figure this one out hundreds of years ago.

    On the one hand I’m inclined to agree with you. But on the other hand, we have a real-time information network that spans the globe today. I can perform an experiment to your specifications, upload a video, and you can see it 10 minutes later. Given the analysis, the physical evidence, detailed build instructions so that people can build their own and reproduce our results, plus a world record, you’d think it would be pretty easy to convince folks today – but not so much.

    The idea of using these underwater as a means for extracting energy from slow moving bodies of water is an interesting notion.

  126. Neil says:

    @ Rick

    The induced resonance of a back and forth water propeller could also enhance the energy extraction from a slow river.

    Imagine a bank of propellers (as wide as the river in a long concrete channel) zipping down a river (faster than the river), sending a wave front backwards (or at least holding some water back). Just as the wave front reflects back down river, the propellers reverse to operate in turbine mode. Pretty soon, the slow moving river is resonating back and forth, with the turbine/propeller in sync to capture the maximum energy.

    I’m think more of like an antenna pulling energy out of space, not a perpetual motion device. The water is the energy source.

    This seems a bit far fetched, maybe the 3rd beer is clouding my though. It’s been good chatting with you.

    Best Regards,
    Neil

  127. Alsee says:

    Imagine someone describes a helicopter to you, and they state it can hover in still air. If you mistakenly model an airplane you would be completely correct to state *the machine you modeled* can’t hover. However if you model the *wrong machine* then obviously your none of your reasoning or conclusions apply to the *actual* described machine.

    This machine is not a windmill powered car.
    This machine never operates in windmill powered car mode.
    This machine is a wheel powered airplane.
    This machine always operates in wheel powered airplane mode.
    The wheels always operate as brake against motion.
    The prop always operates in thrust mode.
    These statements are true when traveling below windspeed.
    These statements are true when traveling above windspeed.

    If the machine you are thinking about does not operate as described above then you are thinking about *the wrong machine*. So long as you think about the wrong machine you cannot form any valid conclusions about *this* machine.

    Once you consider the right machine you will see that power flows from the wheels to the prop. Energy is conserved: the power going to the prop is slightly less than power taken from the wheels. The key is that power = force * distance. When there is wind the prop distance through air will be less than the wheel distance over ground THEREFORE the prop force is greater than the wheel force.

    1. ThinAirDesigns says:

      Alsee — You nailied the problem Charles Platt especially is having.

      As long as he continues to insist to himself that *he knows* how the vehicle *should* work, and that there is required some sort of ‘mode change’ from below windspeed to above he is doomed in his understanding.

      There is NO mode change. No adjustments necessary. No clicking ratchets on one side or the other. No direction of force change in the drive system.

      He keeps falling into his own “ahaaa — I GOTCHA NOW” trap as he comes up with stranger and stranger reasons why he just *has* to be right and everyone else wrong.

      Nice analysis on your part. Thanks

      JB

  128. windmasteruk says:

    I’ve not seen this explanation yet, it seems the simplest to me, many, particularly non-believers, get hung up in formulas, energy flows and other things which just seem to cloud and complicate the issue. Making it look very magical when it is a very simple device really.
    Maybe Rick or JB can comment it I say anything which is not correct.

    If you accept that a windmill powered car or boat can progress upwind against the wind.
    (And I do because I have built a few).
    It is an example of making progress on one medium (the ground – or water) driven by the other medium which is in contact with it (the air)- in other words it’s an interface vehicle.

    Therefore the converse must also be true, that a vehicle can make progress through the air, driven by the other medium (the ground).
    Such a craft might be an airship (part of the airmass) with a wheel on the ground connected to a propeller in the air. It should be easy to imagine that as the airship drifts along with the wind the wheel drives the airship prop and enables the airship to move through the medium it is in (the air.

    The Blackbird cart is exactly the same case, the fact that the wheels also do the job of supporting it is irrelevant. When it reaches wind speed there is no wind flowing over it, it can be said to be part of the air (because there is no relative movement)but the ground is moving underneath and can provide power to turn the prop and push it through the air medium.

    Treadmill Test
    Those that discredit the treadmill test and say that it is not valid must also believe that any wind-tunnel test of anything is not valid also.
    A wind tunnel uses a reversed frame of reference in exactly the same way as the treadmill test.

    In a wind tunnel the test object is stationary and the air is moving – to replicate the real life situation where the test object is moving and the air is stationary.

    In the treadmill test the cart is stationary and the ground moving – to replicate the real life situation where the cart is moving and the ground stationary.

    To believe the treadmill test was not valid you would also have to believe that all the testing done in wind-tunnels by the entire aviation and auto industry was also not valid!

    Hat’s off to anyone who has had the patience to read and understand the foregoing!

  129. Rick Cavallaro says:

    >> Another way of explaining it

    Yes. I’ve used that very argument on occasion. One thing we’ve learned for sure is that it takes different explanations for different people. There isn’t one explanation that everyone will accept.

    More interestingly, when you finally hit upon the right one for someone, they will often say “why didn’t you start out with that one? That makes it obvious to *anyone*!” When in reality tons of other people will tell you that explanation is complete bunk.

    w.r.t. wind tunnels, I have to agree with you there too. I’ve often warned people to avoid flying on planes that relied in wind tunnel testing if they don’t accept the relevance of the treadmill.

  130. Prang says:

    @windmasteruk, who wrote:

    “Such a craft might be an airship (part of the airmass) with a wheel on the ground connected to a propeller in the air. It should be easy to imagine that as the airship drifts along with the wind the wheel drives the airship prop and enables the airship to move through the medium it is in (the air.

    The Blackbird cart is exactly the same case, the fact that the wheels also do the job of supporting it is irrelevant. When it reaches wind speed there is no wind flowing over it, it can be said to be part of the air (because there is no relative movement)but the ground is moving underneath and can provide power to turn the prop and push it through the air medium.”

    That is a great example of why the cart does not work! Think about the airship floating along with the wind. What happens when it touches a wheel to the ground? It develops friction with the ground, loses some of its kinetic energy and slows down! If you use the wheel to drive a propeller, now the wheel feels drag as well as friction and slows down even more. Sure, you will get some thrust from the propeller but it is absolutely impossible to get enough thrust to replace the lost kinetic energy due to friction and drag. If that were possible, every automobile would be dragging a fifth wheel on the ground that drives a propeller on the roof! This is just good old fashioned crack pottery at its worst! As you say, the cart is exactly the same as that airship, and the wheel driving the propeller does not work in either case. It is a real shame that a few videos of a cart being blown along by the wind plus a tall story has confused so many people into believing in nonsense!

    1. ThinAirDesigns says:

      Prang’s ‘5th wheel on the car’ position is correct — in that case, both the propusions system and generator are running off media of the same speed. in fact, in this case it’s the same media (the pavement). Now delta available.

      The ‘airship’ dragging a 5th wheel has it propulsion system in one media (the air) while the generator is in another (the pavement). Even here the airship would encounter the same problem as the car if both media were traveling at the same speed (meaning no wind).

      Unfortunately, prang continues to simply ignore that in a tailwind scenario, one media is moving relative to the other and it’s been known for centuries that with relative movement, power can be extracted.

      Simple “power = force * distance” calculations (and the everday world) show that a device can extract power from the faster moving media and expend it on the slower media with a usable delta remaining. Craft all over the world to this every day.

      But prang refuses do the math.

      1. ThinAirDesigns says:

        With no edit function I must offer a slight correction to my above post. I miswrote that with the car above, both propulsion and generator were working on the same media. In prangs described scenario there was a prop on the roof and thus the above is not true.

        Prang’s scenario would be true if there is no wind, then both media in use (air and pavement) would be running the same speed. If there were wind and the system was properly designed, there would be a delta to utilize.

        It should be obvious however that a propeller large enough to be of value (~ the size of the Blackbird) wouldn’t be an overall benefit in real world wind conditions nor in and around bridges, cables and traffic.

  131. Rick Cavallaro says:

    >> That is a great example of why the cart does not work!

    Given that the real world evidence, analyses, and foremost experts show clearly that the cart *does* work, perhaps a more fruitful use of your energy would be in examining how that can be so – rather than trying to explain how bees can’t fly.

    Prang wrote:
    “Think about the airship floating along with the wind. What happens when it touches a wheel to the ground? It develops friction with the ground, loses some of its kinetic energy and slows down! If you use the wheel to drive a propeller, now the wheel feels drag as well as friction and slows down even more. Sure, you will get some thrust from the propeller but it is absolutely impossible to get enough thrust to replace the lost kinetic energy due to friction and drag.”

    It would seem that way to most. And if there were no tailwind that would be true. That’s what makes this a good brainteaser. When the wheel is working against the ground and the prop is working in a completely different media (the air) that happens to be moving relative to the ground, the equations work out a little differently. We can (and do) exploit that velocity differential by slowing some of that air down.

    >> If that were possible, every automobile would be dragging a fifth wheel on the ground that drives a propeller on the roof!

    And if we always drove everywhere with a tailwind that just *might* be the case. But even then I wouldn’t bet on it.

    >> This is just good old fashioned crack pottery at its worst!

    I’m pretty sure that doesn’t meet Make’s “be nice” policy, but far worse – it’s clearly wrong. For $40 in parts you can do like many other have and build your own. Don’t you owe it to yourself? Repeating my experiments and showing me they don’t work will be FAR more effective than calling me a crackpot. Show me I’m wrong.

    >>It is a real shame that a few videos of a cart being blown along by the wind plus a tall story has confused so many people into believing in nonsense!

    Yes. And even more shameful that real-world physics has also conspired to go against your theories.

  132. Prang says:

    @Rick Cavallaro, who wrote:

    “It would seem that way to most. And if there were no tailwind that would be true. That’s what makes this a good brainteaser. When the wheel is working against the ground and the prop is working in a completely different media (the air) that happens to be moving relative to the ground, the equations work out a little differently. We can (and do) exploit that velocity differential by slowing some of that air down.”

    Ah yes, you claim that the propeller can develop more thrust in the air because of the tailwind. Is that so? Does a propeller in a tailwind develop greater thrust than a propeller in still air? How about we take a closer look at the “mathematics” you are using to calculate that thrust? You are saying Thrust = Power / Velocity where you are using the velocity of the propeller in the moving air. So what happens when the cart is moving at exactly the speed of the wind? The velocity of the propeller in the moving air is ZERO! Plug that into your “math” and the propeller thrust is INFINITE! How about that, infinite thrust! That sucker must really burn rubber when it reaches windspeed, huh? And what about when the cart is moving at LESS than windspeed? According to that same “math” the propeller must have negative thrust! So how can it ever reach windspeed, huh? The only time your math works in your favor seems to be when the cart is already going faster then windspeed and that makes the math inconsistent and should give you a clue that your calculations are nonsense. The fact is, at windspeed there is no energy available and the wheels cannot generate any power. The Power term goes to zero and so does the thrust and the cart if it ever should reach windspeed would start slowing immediately. That is what the real math says, regardless of any story anyone is telling here. As for the real world “evidence” I don’t accept any of it as it was not scientifically gathered, independently analyzed or peer reviewed. Let’s face it; this is an Internet claim only and that just reinforces the crack pot flavor of it all.

    1. Alsee says:

      Prang: “How about we take a closer look at the “mathematics” you are using to calculate that thrust? You are saying Thrust = Power / Velocity”

      Correct. In the THEORETICAL case an infinitely large ideal propeller can generate an arbitrarily large force for zero distance. As like Rick Cavallaro pointed out for an iron rod wedged between fices walls can impose arbitrarily large force for zero power consumption. An ideal propeller could do the same.

      “The velocity of the propeller in the moving air is ZERO! Plug that into your “math” and the propeller thrust is INFINITE!”

      Obviously we don’t have an infinitely large ideal propeller. Due to inefficiencies the power consumed will obviously never reach zero and the thrust is obviously going to have a maximum value, no matter what the effective velocity is.

      “And what about when the cart is moving at LESS than windspeed? According to that same “math” the propeller must have negative thrust!”

      Your physics is wrong. There is no such thing as a negative distance.
      When calculating the amount of work done distance is always positive or zero. A force preforms EQUAL work accelerating a forward moving object as it does decelerating a backwards moving object. In fact “forwards” and “backwards” are entirely relative to an observer, and they do not change if the observer is moving.

      “The fact is, at windspeed there is no energy available and the wheels cannot generate any power.”

      False. If you hook up a generator to the wheels then you UNDENIABLY get wattage coming out of that generator. Period. The power generated at the wheels equals the distance the wheels travel times the load force on the wheels. Period. We simply send that wattage to the prop.

      Conservation of energy? YES!
      Conservation of power? YES!
      Conservation of force? NO!! Levers and gears can multiply force!

      Levers and gears can trivially trade off a smaller distance of travel to obtain force multiplier.

      The power is undeniably available at the wheels. The gear ratio CAN and DOES produce a larger force at the prop. A wind in necessary to effectuate that smaller distance value at the prop.

      “As for the real world “evidence” I don’t accept any of it as it was not scientifically gathered, independently analyzed or peer reviewed.”

      If you know enough math and engineering you can run the calculations your self proving it works. To help you along here’s a PDF from MIT professor Mark Drela already working out those calculations:
      http://www.boatdesign.net/forums/attachments/propulsion/28167d1231128492-ddwfttw-directly-downwind-faster-than-wind-ddw2.pdf
      The North American Land Sailing Association is the world’s foremost independent authority on measuring and certifying wind-vehicle results.
      Countless people have independently built small versions and reported that they work. The parts list has been posted many times and you can build it and test it yourself for about $40 in parts.

      So far the only person I’ve heard of building one of these and reporting failure is Charles Platt, and he got the gear ratio wrong. He had the prop driving the wheels, as proven by the fact that he reported it rolling INTO the wind in his test. If this thing doesn’t work then why do we have NO negative reports except from the one person who got the specs wrong and built the wrong machine?

  133. Rick Cavallaro says:

    Sorry Prang, no time to respond to your silliness at the moment. Gotta go meet JB to work on our current project. Your sarcasm is far less effective than an earnest discussion. Perhaps we can try that?

  134. Neil says:

    @Rick I just sent you an email at thin air. I don’t know if this is your email or not, I copied David so that he could forward if if needed.

  135. Rick Cavallaro says:

    @Prang

    Okey dokey, I’ll try and address this now…

    >> Ah yes, you claim that the propeller can develop more thrust in the air because of the tailwind. Is that so?

    Sort of. It’s more the lack of headwind. If the cart were moving over the ground at 20 mph and moving through the air at 20 mph, then your concerns would be well founded. I could never produce as much thrust as I produced drag to create that thrust.

    But that’s not the situation. Because I have a 10 mph tailwind, the cart only feels a 10 mph relative headwind. Producing thrust in a 10 mph headwind takes only half the power as it would to produce that same thrust in a 20 mph headwind.

    >> Does a propeller in a tailwind develop greater thrust than a propeller in still air?

    That gets complicated. In a tailwind a significant portion of the prop will likely be stalled. Even in no wind this will likely be the case. The stalled portion will create no thrust, so generally no.

    >> How about we take a closer look at the “mathematics” you are using to calculate that thrust?

    O.K. But how about if we do it without the quotes – since we’re going to get it right.

    >> You are saying Thrust = Power / Velocity where you are using the velocity of the propeller in the moving air.

    Yes, in a lossless system that’s the case.

    >> So what happens when the cart is moving at exactly the speed of the wind?

    In this case a prop that was 100% efficient could deliver any amount of thrust with with 0 power input. This would essentially be identical to jamming a rod between two rigid walls. That rod could produce as much force on those walls as you like without delivering a single joule of work. As you might guess, you will never find a 100% efficient propeller. Moreover, the model ceases to be meaningful when the velocity is exactly 0 – as described by my rod between two walls example.

    >>The velocity of the propeller in the moving air is ZERO! Plug that into your “math” and the propeller thrust is INFINITE!

    See above – and lose the quotes on “math”. Math requires more than equations. It requires that you understand how to apply them.

    >> How about that, infinite thrust! That sucker must really burn rubber when it reaches windspeed, huh?

    In your world I imagine it would. But then in your world you’ve already explained that it will also work in no wind.

    >> And what about when the cart is moving at LESS than windspeed?

    See above.

    >> According to that same “math” the propeller must have negative thrust!

    See “windmill”

    >> The fact is, at windspeed there is no energy available and the wheels cannot generate any power.

    Wrong.

    >> The Power term goes to zero and so does the thrust and the cart if it ever should reach windspeed would start slowing immediately.

    Wrong.

    >> That is what the real math says

    Now you can use the quotes.

    >> As for the real world “evidence” I don’t accept any of it as it was not scientifically gathered, independently analyzed or peer reviewed.

    It was in fact scientifically gathered, independently analyzed, and peer reviewed (though not in the typical sense that you probably imagine for a published article). What do you think the NALSA observers were there for?

    >> Let’s face it; this is an Internet claim only and that just reinforces the crack pot flavor of it all.

    To you it’s only an internet claim because you *insist* on allowing it to only exist on the internet in your world. In my world it exists in real life. I can touch it and experiment with it. It yields real results that can be witnessed and documented. As you may recall we invited you to witness our tests – you chose not to. Had you done so, you would have been able to touch it and witness it as well. You may still choose to deny the results, but you could no longer claim it exists only as an internet claim.

    As it stands now, you can build one from the videos I’ve posted for about $40. You can then reproduce our results. But I must warn you – if you do so you will have destroyed your best claim – namely that this only exists on the internet.

  136. Rick Cavallaro says:

    … incidentally, “crackpot” is one word. It’s actually pretty important to spell-check your insults. I hope the reasons are obvious.

  137. Neil says:

    This first paragraph (a disclaimer) is encoded in Rot-13, just skip ahead and read what I have to say. Or decode this paragraph first if you wish.

    Gur sbyybjvat grkg vf vagraqrq nf cher fnepnfz. V’ir jevggra guvf sbe pbzrqvp rssrpg. V ubcr lbh trg n tbbq ynhtu. Vg vf vagraqrq gb or n cnebql bs gur qbhogref naq synzr-onvgref gebyyvat gur vagrearg. Abguvat V jevgr orybj unf nal gehgu gb vg. Gurer vf ab fhpu guvat nf pragevshtny raretl be pragevcrgny raretl. V ncbybtvmr sbe gur crefbany nggnpxf. V unir gur terngrfg erfcrpg sbe gur Guva-nve grnz. Rirelguvat Guva-nve naq gur QQGJSJ crbcyr unir qbar vf gehr naq rnfvyl rkcynvarq. Pbagenel gb gur fhowrpg yvar, pbggba oboovaf unir orra naq pbagvahr gb or gur bar bs gur orfg cnenqvtzf sbe gur haqrefgnaqvat bs qverpgyl qbjajvaq snfgre guna jvaq znpuvarf. Ybhvf Oerana jnf n tbbq naq ubarfg ratvarre jvgu n terng vairagvba juvpu jnf fhpprffshyyl hfrq sbe Oevgvfu uneobe qrsrafr.

    I feel it is time to weigh in on all this talk about Impossible Motion Devices (also known as IMD). It is time to expose the BUFC IMD hoax for what it is. I have proof that this hoax goes back at least 100 years, I suspect it could be even older. From here on, everything I say will be 100% factual and easily checked. At the core of this hoax is this fixation on cotton bobbins. I not sure why, however all propeller IMD hoaxers want to talk about cotton bobbins. Maybe this is just a diversion, to confuse the real tricks being played with more complicated IMDs.

    Note: I’m not falling into the trap of calling IMDs perpetual motion devices (even though they are). IMD proponents do not to claim perpetual motion, so I will not fall into this trap for arguments sake.

    The hoax begins (AFAIK) with the Brennan Torpedo invented by Louis Brennan. Louis likes to talk about cotton bobbins (look it up on Wikipedia if you don’t believe me). This torpedo was claimed to be powered by a wire (slowly) pulled out the back which spins the propeller, causing a much faster forward motion. However, pulling backwards on the cable causes an equal force in the opposite direction and no forward motion. The funny thing here is this is the exact opposite (travel direction) vs. the cotton bobbin hoax. So I can’t even see the connection here.

    I don’t deny the Brennan Torpedo moved forward. This forward motion can be explained 100% with centrifugal energy (free energy if you will) which remains in the spinning propeller. After the wire is pulled, this centrifugal energy (like a spinning top) pulls the torpedo forward. For this IMD fraud, Brennan took £110,000 from the British Navy. The torpedo was soon scrapped for bad performance in a scandal over the high price for this worthless junk. I’m not making this up, look it up, read the wiki.

    For the second time around, this IMD hoax wouldn’t be so easy to pull off. The BUFC cart looks at first to be a simple centrifugal energy device , where the wind spins up the blades (with weights in the tips, but I’ll cover this later). It then bursts forward faster than the wind speed as the prop pitch is increased for more air bite. However this is not the case. BUFC is a hoax, but don’t fall for this simplistic explanation.

    The seemingly magic propeller acceleration, just after the cart enters dead air, as clearly shown in the BUFC videos, is a cheat. (No wind, no power). The key here is centripetal energy. Note that I said centripetal not the similar sounding word centrifugal. If you’re reading this and you don’t know the difference between centrifugal energy and centripetal energy you’re an idiot. That was a bit rude of me. I’ll break it down so all can understand.

    The most common example of centripetal energy is the playground merry go-round. You spin it as fast as possible, with everyone on the edges. Then when everyone pulls themselves to the center (a force applied over a distance, your adding energy here). The merry go round spins much faster. Don’t believe me, do the math! Now assume you’re a military/aerospace engineer with access to very heavy DU (if you don’t know that this is, you’re an idiot). The rest is easy to figure out. You simply load the tips of the hollow tubes of the propeller with DU attached to cables (the same cable that controls prop pitch).

    Note: The DU in the blade tips explains why the cart blade start up so slowly, this stuff is heavy, takes a lot of momentum to get it going. This is just further proof I’ve solved this hoax.

    The cart starts slowly, then just as the air goes dead (and the cart shouldn’t go faster), the BUFC driver pulls the DU to the center of the blade, the prop spins like crazy, and the cart shoots forward. HA! I’ve exposed the hoax!

    Oh, I forgot to talk about cotton bobbins. This is just part of the hoax. I haven’t figured out the exact connection, but it is just another trick. Maybe it’s just a straw man argument, designed to confuse the reader/observer of the IMD hoax. There are many youtube.com cotton bobbin videos. All are fakes promoted by the IMD propeller hoaxers.

    1. windmasteruk says:

      The absolutely best link to show without doubt that it works is here: http://sifter.org/~simon/journal/20101107.h.html

      Those that haven’t seen it already should take a look!

      Also the Brennan Torpedo idea does work.
      I know because I have done a practical test.
      A propeller in water (and anything attached to it) can be made to move away from the pull powering it. I found that it was a matter of choosing the correct gearing.

      1. Neil says:

        I know, read the disclaimer.

    2. Alsee says:

      Not funny.

      1. Neil says:

        Ouch, I guess I shouldn’t post sarcasm. Sorry you didn’t like it. Did you read the disclaimer?

        Best Regards,
        Neil

      2. Neil says:

        @Alsee

        Regarding your posts to this forum, I’d like to complement you for setting the gold standard in polite and persuasive rhetorical argument. I’ve been impressed with every post of yours that I’ve read.

        It is in this context that I take your feedback to heart. I should know better than to post stuff like this. I intentionally made every rhetorical/technical mistake I could think of, thinking this would be a funny.

        In the context of good rhetorical style, here is a link to;

        Boston Globe; Ideas Section; Sunday November 14th, 2010
        I hate to tell you
        Phrases That Announce ‘I’m Lying’
        By Erin McKean founder of wordnick.com erin@wordnik.com
        http://www.boston.com/bostonglobe/ideas/articles/2010/11/14/i_hate_to_tell_you/

        Best Regards,
        Neil

        1. Alsee says:

          I didn’t mean my “Not funny” to come across so harshly. It’s easy to read too much into a two word post :) Part of the reason for the minimalist post was that I sort of considered it a “vote”. I knew I was in a frustrated mood before reading it, and I was specifically considering that others might have found it funny.

          Under different circumstances I might write that sort of parody myself, but this situation just strikes me as really hard to parody. It doesn’t fit. It’s not like we’re dealing with clueless irrational Young Earthers. Anyone interested in arguing the physics of this machine is smarter and more rational than the average public. This machine has probably four or more layers that are extremely misleading, with extremely reasonable mistakes that can be made at each layer. Each of those mistaken conclusions (like the appearance of a perpetual motion power loop) is normally a legitimate discussion-ending show stopper. They don’t believe it works and defiantly demand things like a peer reviewed science paper on it, but I haven’t really been hearing outright conspiracy theories or elaborate hoax accusations. Considering how much this *looks* like a free-energy-device it is perhaps surprising how well their objections tend to implicitly allow for honest gross-error on our side.

          I think good parody needs to nail the exact errors being made. The errors they are making are not “clownish” errors. I think they would be really hard to parody properly.

          P.S. Having all replies on one page like this makes for a very awkward format and nasty page loads. That was probably part of the reason for a two-word reply. I think it’s probably why I groaned a bit at your otherwise unobjectionable post. And now I’m clogging the page up more with this post. ::grumble grumble grumble:: (directed at the universe in general)

  138. Rick Cavallaro says:

    Sarcasm can be a little tricky to do on a forum, but I thought it was funny.

  139. TCH says:

    Here is a copy of my comment posted on the Wired site responding to Adam Fisher’s article (March 2011).

    Thanks to Adam Fisher for a thought provoking article. I’m confident it’s possible to build a kind of DDWCFTTW, but along with Charles Platt, I’m still skeptical about the ability of the Rick Cavallaro’s design to do the job.

    Why am I skeptical? When tacking at an angle to the wind a yacht is able to extract energy from the wind at any speed, providing the forward velocity of the craft remains at a suitable angle to the wind. In Cavallaro’s thought experiment involving the cylindrical planet, the yacht always maintains that angle. So far so good. When transformed into a propellor-driven design, that relationship disappears once the craft on which the propellor is mounted achieves wind velocity becaus the sail (the propellor blade) is no longer positioned at an angle to the wind – at that point there is no wind relative to the craft. Confirmation that no energy can be extracted from the wind in such a situation can perhaps be reached by the following thought experiment: picture standing the vehicle in a quiet windless room with a flat smooth floor and see how long it takes before it starts moving forward.

    So why did I think that a DDWCFTTW was possible? Connect the propellor to a flywheel. Stand the vehicle on a runway with the transmission disconnected from the wheels. Let the propellor and the flywheel spin until there’s lots of energy stored. Engage the transmission and just as with a carbon-fuelled car or a motor-boat, the stored energy can be used to outrun the wind – until the stored energy runs out. So a purely wind-powered vehicle can certainly go DDWCFTTW but only in the possibly trivial sense of using stored energy derived from the wind to do it.

    The Cavallaro craft could well have done something similar. At speeds lower than the wind speed the sail-prop was extracting energy and building up momentum. Was the energy stored in the rotating blade sufficient to continue to accelerate the craft? The question lurking in my mind about Cavallaro’s success is whether he has genuinely found a way of extracting energy from wind when the relative velocity between the blade and the wind is zero; or whether the energy stored as rotational momentum in the propeller would have been sufficient to explain the results.

    1. Keith says:

      THC – I think the biggest misunderstanding here (made by Mr Platt and yourself) is the assumption that the wind is turning the prop and that this drives the wheels to push the cart forward. This isn’t exactly the case. If you watch the videos carefully, you will notice that the prop is turning against the wind as the vehicle begins to move forward. This is a critical difference and one that changes everything.

      When the vehicle is at rest, the prop acts like a large sail, providing drag for the vehicle to be pushed forwards. However, the prop is linked to the wheels, and so it begins to turn AGAINST the wind as the cart is pushed forward. The first make article got this wrong and assumed that the prop was being driven by the wind, and then that the prop was driving the wheels and propelling the cart along with the wind. Mr Platt and yourself are correct that this configuration would never achieve faster than wind velocities, as your argument makes clear.

      So what happens when the prop turns against the wind? At first, not a whole lot, as is evident from the very slow acceleration of the full scale vehicle. The wind is directly providing almost all of the thrust via drag created by the vehicle (and large cross-sectional area of the prop). However, as the vehicle (and prop that is driven by the wheels) begin to increase in speed, the prop starts to create a useful amount of thrust (against the wind!). This thrust can only increase as the vehicle’s speed increases, and conversely, the wind pushing against the prop thrust is decreasing (because the vehicle is increasing in speed).

      At this point, one might look at this relationship and think “but doesn’t that thrust from the prop cost something – can’t get it for free right?” Well, it is true that energy isn’t free, but speed and energy are not the same thing here. The energy going into this system is the kinetic energy of the wind (moving air mass) relative to the ground. The gearing involved in coupling the prop to the wheels (and therefore the ground) allows for these vehicles to be a kind of mobile windmill, extracting energy from the wind (relative to the ground) continuously and resulting in increasing vehicle speeds. The result is a continuous force on the vehicle that pushes the vehicle beyond the wind’s speed relative to the ground. Prop inefficiency, friction throughout the mechanical coupling, and drag forces (continuously increasing relative to airspeed) will limit the vehicles velocity. Also, the gearing of the prop relative to the wheels, as well as the prop pitch directly relate to what multiple of wind speed a given vehicle is theoretically limited to. The result is unexpected, but pretty fascinating I think!

    2. Anonymous says:

      TCH: “Here is a copy of my comment posted on the Wired site responding to Adam Fisher’s article (March 2011).”

      Here the copy of my response to you:

      TCH: “In Cavallaro’s thought experiment involving the cylindrical planet, the yacht always maintains that angle. So far so good. When transformed into a propellor-driven design, that relationship disappears once the craft on which the propellor is mounted achieves wind velocity”

      Wrong. Each point on the airfoil moves along a helix of constant pitch, while the true wind is parallel to the helix-axis. So the angle between the airfoil velocity vector and true wind vector is independent of the vehicles speed. Just like in the case of a sail boat holding course during a long tack.

      TCH: “Connect the propellor to a flywheel. (…) The Cavallaro craft could well have done something similar.”

      No it couldn’t. The vehicle was inspected by independent judges from the North American Land Sailing Association before the record runs.

      TCH: “Was the energy stored in the rotating blade sufficient to continue to accelerate the craft?”

      Irrelevant, because the rotation of the blades accelerates as well, when the vehicle accelerates. So they are not releasing any stored energy that could be used for propulsion.

      TCH: “The question lurking in my mind about Cavallaro’s success is whether he has genuinely found a way of extracting energy from wind when the relative velocity between the blade and the wind is zero”

      No, he didn’t. But that is irrelevant, because the relative velocity between the rotating airfoil and the air is never zero (regardless of vehicle speed).

  140. Steve Sanders says:

    I just learned about this the other day from the Wired article, and what a great story. Armed with just classical physics and aerodynamics, this remarkable concept and its proof were carried out without real exotic technology or new scientific breakthroughs. Just a great idea flying in the face of intuition and many haters on the Internet, who now sound like real flat-earthers.
    I haven’t studied all the discussions and posts and squabbling, but one question I still have is about the notion of a transmission. Though the Blackbird has none, I see from construction photos that it has a variable pitch propeller. I know the concept models did not, so it must not be crucial, but my questions are: wouldn’t the Blackbird have gone slower with a fixed pitch prop? And could it be considered a type of transmission?

  141. andrewm says:

    Steve, As I understand it, Blackbird does not have a variable pitch propeller. A variable pitch propeller would help the cart accelerate faster, as the pitch could be optimized to produce maximum thrust through a range of apparent wind speeds (and changing apparent wind angle relative to prop foil). It would not, however, change the ultimate speed of the cart (assuming they got the fixed pitch right).

    To prove this, think of an aircraft. If your aircraft has a variable pitch prop, it can be set for low angle of attack to accelerate the plane from a stop. Acceleration will be good. As the speed of the plane increases, the apparent wind speed from the front of the plane increases, effectively reducing the load on the prop. The prop pitch is changed to increase thrust, resulting in more apparent wind and the cycle repeats until the power of the engine coupled with prop at optimum angle reaches equilibrium with drag.

    Now imagine that same plane with a fixed prop. The prop is fixed to the same angle as the final top speed equilibrium reached above. This time, when the plane starts the prop is not optimized for low speed angles of attack. Still, the thrust is positive, so the plane will accelerate, but slower. Eventually, it will reach the same stasis of power and drag–same top speed, but will take longer to achieve it.

    Blackbird has a fixed pitch. I assume that they did the math, and set pitch to be optimized for the apparent wind angles they were expecting.

  142. Anonymous says:

    I was intrigued by the previous article and (if I remember correctly) a reference on the mythbusters site/forum somewhere. Now that this article is out I had to think through the problem, but I’m pretty sure I see how it works now. http://blog.jshaver.net/2011/04/down-wind-faster-than-the-wind-machine/

  143. louis says:

    Now there are many Coach Outlet which provide cheaper price for the consumer. Coach outlet Store .Now Coach are available in our Coach factory outlet ,come here to choose your favorite coach handbags and Coach bags from Coach Outlet Online coupon .We hope you have a happy shopping in our Coach Factory Outlet store! The Coach Factory Outlets,are more than 100 kinds of products.

  144. Coach Outlet says:

    Choose our Coach outlet store, buy your preferred Coach outlet products,Coach outlet jewellery are now really famous and stylish amid several style jewellery,which is really popular to womens.As the leading corporation all through the world, we offer the best current goods. What’s more, we do long time business in the eBay. In addition, the exclusive suppliers of Coach factory outlet we have also contribute to our low price strategy, so we can supply the best Coach outlet online of products, best price and best service. Look forward to doing good business with all my friends in the world. We do all we can to provide you with the best quality of Coach outlet products with lowest price.
    We are doing site maintenance to build you a better Blue fly. Thank you for your patience on Coach outlet online store !

  145. seton schiraga says:

    I must admit, I was a skeptic.  seeing the video is proof enough for me as far as the possibility of DDWFTW.  that said, I still don’t understand the mechanics here. intuitively, it seems like the cart is extracting more energy from the wind than it losing for frictional forces, thus allowing faster than wind travel.  I reaslize that is not the case, but that is where my intuition is pointing me.  saying that the cart’s forward motion powers the turbine, which pushes the cart forward reads like perpetual motion.

    can someone with an engineering background please publish some vector diagrams?  All this text-based explaining does me no good- I am a mechanical engineer, not an English teacher!   

  146. Katherine Silly says:

    Christian Louboutin  
    80% off sale Discounts
    Clean up the warehouse, discount promotions,
    the latest 2011 super cheap high heeled shoes
    http://www.highheels-outlets.com

  147. Anonymous says:

    If you are mechanical engineer, why don’t don’t you draw a free body diagram yourself, instead of saying where you intuition points to. Here is the math anyway:
    http://www.boatdesign.net/forums/attachments/propulsion/28167d1231128492-ddwfttw-directly-downwind-faster-than-wind-ddw2.pdfhttp://orbit.dtu.dk/getResource?recordId=241183&objectId=2&versionId=1

  148. how to run faster says:

    You may be wondering how you can give your athletes the tools they need to achieve their goals, but without having to commit every moment of free time to learning ‘speed training’. As you continue reading, you’ll be interested to see the solution you’ve been looking for is at your fingertips. And you’ll wonder how much faster your athletes could be if you had stumbled upon these simple speed training strategies earlier in your coaching career!

  149. david pratt says:

    we regularly go several times wind speed in iceboats…

  150. an exercise routine says:

    Wonderful web site. Lots of useful info here.
    I’m sending it to several friends ans also sharing in delicious. And of course, thanks in your sweat!

  151. Basil Caprese says:

    This article, and the arguments posed herein, are absolutely hilarious. “Skepticism is a primary tool of science. We’d be hypocrites if we never directed a skeptical eye towards Scientific Skepticism itself. Denied imperfections and errors are free to grow without limit, and Skepticism is not immune to this problem. Unbridled gullibility can destroy science, but unbridled disbelief is no less a threat because it brings both a tolerance for bias and ridicule as well as the supression of untested new ideas. Better to take a middle road between total closed-mindedness and total gullibility. Practice pragmatism, pursue humility, and maintain a clear, honest, and continuing view of ourselves and the less noble of our own behaviors.” -Bill Beaty

  152. Rick Cavallaro says:

    Now you can have the Blackbird for your very own. It’s not much of a commuter, but one heck of a conversation piece. Look for “Blackbird wind” on Ebay.

  153. john says:

    First I’d like to congratulate The Blackbird team once more on their accomplishments, both DDWFTTW and directly upwind. I’d also like to congratulate them on their efforts to educate the public on the matter.

    Mr. Platt’s attitude is sad, especially for a journalist or scientist. But unfortunately, it seems to be human nature to lose objectivity once insults have started to fly. And when money is involved, that is all the more reason to remain stubbornly ignorant.

    Secondly, I’d like to make a correction. But before that correction, because of the controversy, I think it might be safer to point out that showing a “proof” to be invalid does not mean the conclusion of the proof is false. for instance, if my “proof” that 2+2=4 is: “1+0=2 QED”; that proof is obvious and total bunk. None the less, “2+2=4” is still in fact true.

    Following are the relevant statements.

    Rick, in reply to spleen on Nov. 6 2010 @9:40PM writes:

    …for several reasons we opted not to let the prop drive the wheels (we used ratchets to prevent the possibility). This was partly due to the design of our drive-train, and partly to prove to NALSA that we could not make any use of stored momentum or energy to accelerate.

    And JB (ThinAirDesigns), in reply to edgertro on Nov. 5 2010 @11:35AM writes:

    This ratchet had to be included and functioning (confirmed by NALSA observers) to meet the NALSA rules that no stored momentum from the propeller could be used to drive the vehicle.

    While I have no independent knowledge about the statements themselves, I assume them to be true. However, they directly and clearly imply that the authors and NALSA believe that the ratchet(s) prevent(s) stored energy (JB confines it to stored propeller energy) from being used to propel the vehicle. The fact is the ratchets do no such thing. In fact they make the use of stored propeller energy a little more feasible, not less. It only prevents energy stored in the propeller from being transferred to the wheels, but it can still use that energy itself; just like the rotor of a helicopter in auto-rotation.

    But, let me make it clear that it is obvious that they did not use the ratchet that way, and they did NOT use stored energy. I think anyone who can pass a first year classical (Newtonian) mechanics class for science majors should be able to determine for her or himself that classical mechanics has always claimed that the Blackbird design would go DDWFTTW just as Rick and JB describe.

    Now I’d like to make an assumption explicit, that there are no hidden energy storage devices (such as batteries, springs or movable masses – including but not limited to masses sliding radially on rotating components). I’m sure that the NALSA observers thoroughly checked for any such possibility both before and after the runs they witnessed.

    Given the above assumption, the only possible sources for stored energy would be the visible moving masses, i.e. the wheels, the rotor (propeller), and what links them. But the rotational velocity of the wheels is directly proportional to the velocity of the vehicle, so their energies must increase and decrease together. So the wheels cannot give any energy to the vehicle and therefore cannot be used to store any energy.

    Without any ratchet, the rotor is directly linked to the wheels, and the same argument applies to it (and the linking components). So, without the ratchet, there is no possibility of storing energy. So, the fact that they had runs without any ratchet is proof that they didn’t use stored energy.

    With ratchets that prevent the rotor from turning the wheels, the rotor is free to spin as fast as it wants (just not slower than the wheels would turn it). But to do this, it needs the wind to spin it; and to go in the right direction, the pitch needs to be reversed so it can act as a turbine. So, the fact that some of their runs were with a fixed pitch propeller is also proof that they did not use stored energy.

    Once the energy is stored in the rotor, the Pitch can be reversed again, and it can again be the propeller it was designed to be. This will slow the propeller down while speeding up the vehicle and wheels. At some point the stored energy will be used up, and the ratchets will catch. At that point the vehicle will act as it actually did during the runs and continue accelerating till reaching its peak velocity around 3X wind speed.

    One last point is that the propeller was clearly not designed to store energy. It was designed with a small moment of inertia, and the blades had a significant fixed twist designed for a significant forward pitch. Both of these features are lousy for storing energy, but ideal (or close to it) for the intended purpose (which is to continuously use the wind’s energy and distribute it partly to the forward energy of the vehicle, and partly to the rotational energies of the wheels and propeller).

    The conclusion is that the Blackbird team and NALSA members all seem to have misunderstood the effect the ratchets would have on the ability to store and use energy, but the accomplishments and records are still valid.

    Now, if Rick and JB can admit to their oversight in this matter, maybe that will give Mr. Platt the example he needs to admit to his. To start it off I’ll admit to having made the same mistake as NALSA members and the Blackbird team. It took me months to realize the mistake, and I think it was even months after being reminded of auto-rotation. Of course that isn’t my only mistake, but I don’t really see the point for a litany of mistakes.

    1. Rick Cavallaro says:

      The ratchets do as advertised for the assumed mode of energy storage. I think it was clear to all that we could not effectively store energy operating that propeller as a turbine – and even less so as we approached wind speed. You’re suggesting a mode of operation that was not anticipated by either NALSA or the participants, and it can very easily be shown that the Blackbird could not have approached the performance that was measured in that mode.

      In either mode, one has to consider that the propeller weighs only a tiny fraction of the cart and driver’s weight. If we managed to spin the prop up to an RPM of imminent failure and converted that energy to speed with 100% efficiency, I doubt we could achieve more than 1 or 2 mph faster than wind speed.

      You can call that a mistake if you like – but I don’t think it was necessary to anticipate a mode that could clearly not produce the performance that we set out to produce.

      1. john says:

        Frankly, I’m disappointed in your reply. It certainly does not provide Mr. Platt with a good example.

        I totally agree that it is Highly unlikely that your Blackbird could reach 10% above wind speed using the energy storage method I describe. But I already made it clear that you didn’t use that method. I also made it clear that the Blackbird was obviously not designed to use that method. And when describing the method I clearly stated that once the stored energy was used up it would then be operating in the mode you use (obviously not using stored energy as that is already used up), and would “continue accelerating”. Which clearly means that your method definitely produced a better performance. So it’s not clear to me why you feel the need to repeat it.

        Besides the fact that I already pointed it out, I also don’t think your argument that it could clearly not produce the performance you achieved is all that relevant here (in your discussions with NALSA it could be). For the most part you are presumably trying to convince doubters that you achieved what you claim, and in their minds YOUR method “could clearly not produce the performance that you set out to produce”. Yet you still think they should consider it.

        But my point and your mistake is that you call the ratchets “proof” that you don’t store energy in the rotor. As I’ve pointed out, with a fixed gear transmission, and a propeller that cannot vary its moment of inertia, and NO ratchet, it is impossible to store energy in the propeller. So what mode of energy storage were you assuming that the ratchets would eliminate?

        As I see it, the ratchets remove a simple proof you would have in their absence, leaving you with a much more complex proof (which you outline in your second paragraph) that does not involve the ratchets but rather the design of the propeller. Anyone who can be convinced by this complex proof isn’t likely to need a proof at all, as any such person should be able to understand your design in the first place. But in no way do I dispute that complex proof. If you’d used that instead of the ratchets as your proof, we wouldn’t be having this discussion.

        Of course, it is possible I’m wrong, and there is a mode of energy storage I didn’t anticipate that the ratchets do prevent. If so, explain that mode. Nothing you’ve said so far addresses that in any way.

        1. Rick Cavallaro says:

          Quite frankly – I don’t remember. I think you’re right that a no-ratchet system could not store energy as the prop would have to slow down to use the stored energy, and that would slow the wheels, thereby slowing the cart. I do remember this much (I think)… we originally wanted a ratchet on the prop shaft simply so that braking would not require us to stop the prop in addition to the vehicle itself. Even that is silly when you consider the amount of energy the prop can store. Perhaps I could re-read the submission report and recall the argument for a ratchet as regards stored energy.

          1. John French says:

            Rick — It’s good to see you have recently posted in here. I only recently learned about Blackbird and it is truly facinating. My background includes having done a lot of design work aimed at improving fan performance for applications handling fluid streams with low to moderate specific speed. I could not resist stringing together some calculations involving Blackbird’s performance. I am sending this reply as a guess that it is email notifications that get your attention of new posting in here … and I hope you will view with interest my recent posting. Drop me an email and I’ll send you that Excel file and some additional information about the fan enhancing work I’ve done.
            — John French

        2. ThinAirDesigns says:

          John, I apologize for the time in response, but I don’t regularly check this website for comments years after the fact.

          You use a quote of mine to make a point of yours and it doesn’t really make your point at all:

          “This ratchet had to be included and functioning (confirmed by NALSA observers) to meet the NALSA rules that no stored momentum from the propeller could be used to drive the vehicle.”

          You act through ignorance (not a slam — just a fact of you not being there) as if the ratchet was the only design criteria required to ensure no use of stored energy. Simpy not true and not stated as such in my above quote.

          Perhaps the most relevant to your concerns is this: The adjustment range on the variable pitch prop was intentionally set (and demonstrated to NALSA observers) such that below windspeed, ‘turbine mode’ was unreachable.

          We worked on this adjustment during our tests at the New Jerusalem airstrip to get the pitch as neutral as possible on the low end (to facilitate start-up) while not crossing the line and spinning up. We had a rather scary (for me) incident during this process where we *did* get into turbine mode (with the Blackbird parked) and due to biased mass balancing in the hub portion of the controls, Rick was unable to regain pitch control of the blades with it spinning so fast. We had to turn the vehicle sideways the wind to stop the spinning. The RPMs got way to high for my liking. After that, we were very careful to make full negative stop still in the propeller regime, even separate from the needs of NALSA.

          A combination of ratchet and pitch range *together* ensured that there was no mode of stored energy usage available (to say nothing about the accurate points you make regarding the viability us such if a ‘cheat’ had been enabled to allow pitch range or ratchet to be bypassed.).

          Now as to Platt’s silly concerns, all he need do is consider the results of the Ivanpah NALSA trial where there was no prop shaft ratchet installed and a fixed pitch hub in place and still mutiples of windspeed achieved.

          JB

          1. john says:

            No need to apologize for the delay, as I did not expect a quick response (in fact I was surprised by Rick Cavallaro’s almost immediate response). However, I am just as disappointed in your reply as I was in Mr. Cavallaro’s initial reply.

            You both claim I’m wrong, and that the ratchet was actually instrumental in your proof that you couldn’t have used stored energy, but you don’t give a single argument to support that. Instead you point to rotor characteristics that prove you couldn’t have used stored energy to achieve your goal. Those arguments in no way contradict ANYTHING I said. Quite the contrary, they support 3 points I made, namely: that you did NOT use stored energy; that you clearly had no intention of using stored energy; and that with the Blackbird you could not have achieved your goal by using stored energy.

            Given the assumptions I outlined in my first note (unless there is a mode of energy storage that eludes me), I gave unequivocal proof you could absolutely not use any stored energy on some of your runs (including but not limited to all runs that LACKED the ratchets). That also proves that you didn’t. And I believe that I gave as solid a proof as is possible about an intention, that you never had the intention of using stored energy. Your arguments (using information about the rotor of which I was ignorant) reinforce my points by providing more direct proof that you couldn’t use stored energy on ANY or your runs.

            So, on the one hand I’m glad for this new information and these arguments that support and reinforce my position; on the other hand I’m aghast that both of you could consider that these arguments contradict my point. Both of you have demonstrated a well above average ability to reason, so I can only (well, not quite “only”, but…) surmise that you didn’t deem it necessary to really read my comment. Mr. Cavallaro demonstrates a slightly better understanding of my points so probably put a little more effort into his reading, but you must have just skimmed my comment and written your reply based on the point you falsely assumed I would be trying to make.

            If memory serves, I believe I’ve seen instances where you have complained that people attacking comments of yours didn’t really read your comments. You can’t expect a courtesy from others unless you are also willing to provide it.

            So, once again, please try to address the actual issue. If the ratchets do in some way provide an element of proof, explain how. If you can’t explain how, I would urge you to consider the possibility that you aren’t infallible.

            In his second reply Mr. Cavallaro made a small step in the right direction. I hope you can continue to progress in that direction.

  154. John French says:

    I put together an Excel spreadsheet to evaluate a combined aerodynamic drag, rolling resistance, and fan performance analysis of the Blackbird car. I’d like to share this with those interested in seeing some quantifying analysis. This work is aimed at contributing to the understanding of the basic physics and demonstrating what I believe to be a simple yet powerful solution to the zero wind speed with push start performance expectation.

    Send me an email and I’ll send you the file. Or better yet … is there a way to post files in here?

    1. John French says:

      Just realized clicking my name or that icon does not provide email address …
      jtfrench911@att.net

    2. Rick Cavallaro says:

      John French,
      I’m quite confident that the Blackbird will come to rest when operated at 0 wind-speed. I base this on the analysis and theory upon which it was designed and built, as well as experience in the real world. I’m also pretty confident that no system can be made to output more energy than it uses by harnessing the relative wind over the device. If you are able to build an operational prototype that you think demonstrates this principle I’d be very interested to see it in operation.

  155. Rick Cavallaro says:

    >> You both claim I’m wrong,

    Let’s be fair (and honest) John. Go back and re-read my 2nd response.

    1. john says:

      Sorry, my bad, I didn’t make it sufficiently clear that I was only referring to your original answer where you did in fact imply I was wrong (by saying the ratchets do as advertised).

      I also pointed out that you seemed to have understood my point a little better than JB. At that point as well I was only referring to your first reply. In your second reply, you do seem to have understood. So, If you’re not comfortable being lumped in with JB, I’m sorry I did so.

      I also said that you had made a small step in the right direction with your second reply. Again, I apologize: I should not have used the word “small”. It was a definite step in the right direction.

  156. John French says:

    Whether the vehicle is pointed downwind, upwind, or push started in zero wind … a proper force balance determines if the vehicle maintains a certain speed, accelerates, or decelerates. The cart on the treadmill demonstrates a forward thrust and its doing so does not make it some sort of greater than unity device. The fan’s power is simply greater than the opposing forces that are mostly rolling resistance in the absence of wind plus vehicle weight times x sin(angle).

    Rolling resistance is a function of the normal force and by inclining the treadmill the rolling resistance decreases. cos(10 degrees) = 0.9848 … so only 1.52% decrease in rolling resistance; however, the car’s height remains fixed and it is not actually “going” uphill. A person running on the treadmill “is” going uphill. The person is advancing/climbing. The car is developing force and doing work … work = force x distance … and the distance travelled is the belt’s distance per unit time (i.e. 10 mph). But there is zero “potential” energy being accumulated because of the fixed height. Analysis of the sloped treadmill performance requires a more thorough experiment. If one were to remove the propeller and begin with a level treadmill at zero belt speed … as the belt speed is increased at what incline does the rolling resistance offset the vehicle’s tendency to roll down hill? Weighing the vehicle and performing such and experiment would establish a rolling resistance value with a high degree of confidence. There are also “coast down” methods for this determination.

    In the presence of wind, the aerodynamic drag on the vehicle increases with the square of the velocity of wind over the vehicle. Downwind at the speed of the wind equates to zero aerodynamic drag. Downwind at 2.8 x windspeed is an aerodynamic wind speed of 1.8. Upwind at 2 x windspeed is an aerodynamic wind speed of 3. The ratio of aerodynamic force is 3^2 / 1.8^2 = 2.78 times more aerodynamic resistance upwind than downwind at each respective speed achieved.

    Performance predictions depend upon having a fan performance curve, knowing the vehicle’s drag coefficient, the vehicle’s cross sectional area, the vehicle + driver weight, and the rolling resistance coefficient (That can be as high as 0.06 on a salt flat. Steel wheels over rail is only about 0.0017 .) Sufficient experimental data can make the determination of all of the above inputs very precise; however, with only two data points one can still perform a fairly decent analysis by making high quality assumptions. One such assumption is to equate fan performance to that of similar type fans with a known performance curve. Guessing at the cross sectional area is not a “super critical” choice because the analysis essentially solves for the Cd x A value (Drag Coefficient x Cross Sectional Area).

    Crunching through the numbers reveals a push started Blackbird in zero wind should just begin to accelerate on its own at near 18 mph and cease to accelerate at about 38 mph.

    Aerodynamic and Rolling Resistance formula can be found here:
    http://en.wikipedia.org/wiki/Drag_equation
    http://en.wikipedia.org/wiki/Rolling_resistance

    A good fan curve can be found near the bottom of this article:
    http://www.wind-power-program.com/large_turbines.htm

    Note: The calculations are a bit tricky. Easiest to use SI units. The normal force should be converted to Newton. Work is in joule and power in Watts. By using SI units and meters per second as the velocity … the work and power quantities are the same magnitude.

    Of course, my offer to send anyone interested the detailed calculations within an Excel spreadsheet still stands. It will just cost you an email and my address is found in a previous posting.

    Notes: Towing Blackbird may be problematic because of the disruptive draft from the towing vehicle. Also, observing Blackbird on a windy day is definately problematic because a cross wind plays major havoc on the fan’s performance. It must be a calm day for best observations and pushing seems better than towing to get things rolling.

    Propellers are notoriously poor performers at low speed. A blade’s ability to perform is reduced by the turbulence of other blades competing for the same air. Axial flow devices need a rather high velocity in order to have a “clean” source of undisturbed air into which to bite. Observe a plane at full throttle barely creeping off the line and not starting to develop any g-force until typically well over 50 mph.

    Once the car is outrunning the treadmill belt and has its forward progress constrained by a string or other fixed object … there is actually work shaft being imparted into the belt. A combination of three events are possible:
    1) The treadmill belt must accelerate.
    2) The power used to drive the treadmill belt must be reduced.
    3) Drive wheel slippage must increase.
    It is doubtful the treadmill’s power consumption change would be “measurable”. It is also accurate to state that the slower moving drive wheels constrain the propeller from accelerating. A very large and very smooth surface with a remote control would certainly be fun. Further data gathering at increased angles with some number crunching of the results would generate a good quality fan curve. The belt speed vs angle where the string is neutral is the data of most value. The realized top speed on a calm day would provide a high quality drag coefficient value.

    1. Rick Cavallaro says:

      “Crunching through the numbers reveals a push started Blackbird in zero wind should just begin to accelerate on its own at near 18 mph and cease to accelerate at about 38 mph”

      I’m quite certain that both a proper analysis as well as experimental results indicate that the vehicle will not accelerate on its own on a level surface with zero wind. If towed to any speed at all it will roll to a stop in such conditions.

      “The cart on the treadmill demonstrates a forward thrust and its doing so does not make it some sort of greater than unity device.”

      Correct. We do our best to make it quite clear that this is not an over-unity device. Nothing about the Blackbird or its performance challenges any of the well established laws of physics.

  157. johnf911 says:

    Rick — At what speeds was Blackbird pushed or towed on a calm day to observe its behavior?

    The analysis of data you have provided establishes a fan curve, vehicle aerodynamic drag, and vehicle rolling resistance.

    The question >> Is there any speed at which the fan’s output is greater than the combined aerodynamic drag and rolling resistance?

    Both your upwind and downwind experiments demonstrate the answer is: “Yes, there are such speeds.” You have proven Blackbird accelerates on its own from a dead stop with the assistance of wind energy to initiate its movement.

    The analysis shows on a calm day that a push start to at any speed less than 18 mph will indeed cause it to coast down to zero. Below 18 mph the force balance is negative and there is deceleration.

    The analysis also shows that if pushed above 38 mph it will then coast down to 38 mph.

    The treadmill tests with a fan placed in front of the cart did indeed demonstrate acceleration? There was mention that such conditions were videotaped?

    If a push start replaces the push start from the treadmill, and the vehicle’s speed replaces the speed from the fan that was placed in front of the cart … the only remaining criteria is at what speed does acceleration occur for Blackbird vs the little cart. The physics are the same. The ratio of fan power to vehicle drag characteristics (aerodynamic drag and rolling resistance) are obviously very different.

    Blackbird appears to have less power to drag ratio than the tiny cart. The structural requirements and carrying a driver force this compromise … as the cart is certainly a minimalist design. There is consistency between the observed cart accelerating at 10 mph and the analysis showing Blackbird needs at least 18 mph.

    There are boats that are wind turbine powered that accelerate directly into the wind. Blackbird duplicates this behavior. No huge surprise, but certainly enjoyable to see accomplished.

    The reduced land vehicle drag as compared to a boat allows Blackbird to outrun the wind. This is a “homerun” type of demonstration.

    In my opinion, the “grand slam” demonstration is self-propelling itself on a calm day with a push start.

    I encourage you to scrutinize the analysis I have sent you and let’s correspond using email instead of this public forum? Of course I am willing to talk here … but this is so much more awkward than a simple phone call. At least in here Blackbird continues to live and a 3 year old thread is amazing!! I don’t know who now owns Blackbird and if it is structurally sound for push start tests. It would be amazing to demonstrate on a hard surfaced (less rolling resistance) circle track like the brickyard; however, any test has some risks and a fan failure could certainly endanger the driver.

    1. Rick Cavallaro says:

      “In my opinion, the “grand slam” demonstration is self-propelling itself on a calm day with a push start.”

      I agree that would be the “grand slam” demonstration as it would prove some of the most well established laws of physics wrong. But neither the Blackbird nor the small treadmill cart is capable of that – and it would be an understatement to say that I’m skeptical of any vehicle doing that.

    2. john says:

      It sounds like you put a fair amount of effort into your calculations, unfortunately they are based on a faulty premise. When you list the opposing forces to your “fan”, you are omitting the most important one: the force of the ground on the wheel. I’m not talking about rolling resistance here, but the force on the wheel that provides the torque that gets transferred through the transmission to turn the propeller.

      The cart is designed to use the energy in the shear plane that exists between the air and ground when there is wind. By leveraging one against the other, speed can be gained at the expense of force. To achieve 3 times wind speed, they had to sacrifice at least 2/3 (I suspect closer to 3/4) of the propeller’s thrust to the ground to provide the torque to turn the propeller.

      However, if you add that force into your analysis, you would still probably conclude that the Blackbird can self propel (at appropriate speeds) with no wind. That is because your analysis seems to be ignoring the effect of the rotor’s rotational velocity relative to its linear velocity through the air. This is actually quite important as it switches from Propeller to turbine when the cart’s linear velocity relative to the air becomes greater than the rotational velocity times the pitch. When this happens, the directions of the forces get reversed. Reversing the forces means they are putting the breaks on the cart which gets added to the breaking of the resistive forces.

      The rotational velocity of the rotor times its pitch divided by the ground velocity of the cart is a constant determined by the fixed gear transmission. For simplicity, I’ll call this constant R.

      For the DDWFTTW cart, the wheel turns the propeller, so through the transmission, the smaller force of the ground on the wheel must overcome the larger force of the air on the propeller. Since power is at best constant through the transmission, this increase in force must be accompanied by a decrease in velocity. So the DDWFTTW cart must have 0<R<1.

      With no wind, the ground velocity is equal to the air velocity. So if the cart is pulled (or pushed) with no wind present, R<1 means the air velocity is greater than the rotational velocity times the pitch and the rotor will act as a turbine, and the force will be a breaking force. When resistive forces (air drag, rolling resistance, etc.) are added, they will only add to that breaking.

      A similar analysis with any value of R will show that with no wind the cart will always be in a breaking mode (except for R=1 when there are only resistive forces).

      A few added notes:

      The Blackbird on the salt flats and the cart on the treadmill are essentially the same system. The only difference is that in one case the Earth (the reference frame from which we tend to view things) is the Blackbirds ground frame, and in the other the Earth is the same as the air frame (different from the cart's ground which is the treadmill). An analysis of the cart in the treadmill reference frame is identical to analysing the Blackbird in the Earth frame, and an analysis of the Blackbird in the air reference frame is identical to analysing the cart on a treadmill in the Earth frame.

      A push (or pull) start in no wind cannot replace the wind or the treadmill. The carts are continuously taking energy from the system (the shear interface between ground and air). They are not simply getting a push start. They depend on a continuous supply of energy, and without wind they don't have it.

      An over unity device (or greater than unity device) refers to its energy usage. If it ends up with more energy than it started with (if its net power balance is positive) then the final energy divided by the initial energy is "over unity" (greater than 1), and it is an over unity device. If these carts could self accelerate in no wind, they would in fact be over unity devices. At the start neither the air nor the ground has energy, and at the end the air would have some energy, and the cart would have more than it started with.

      With wind, the air has energy and the propeller is slowing the air down, thus removing energy from the air and transferring it to the cart. It continues to do so past wind speed and only stops when the cart reaches wind speed divided by (1-R). Since 0<R<1, that is greater than wind speed. For the Blackbird, 2/3<R<1, and I would guess R is close to 3/4. Using that guess, the net thrust would be downwind until the cart reached 4 times wind speed, but would not be able to overcome resistive forces past 3 times wind speed for the conditions they had.

      1. Rick Cavallaro says:

        Correct on all counts. The parameter you call R we call VSR (for Vehicle Speed Ratio). And it is in fact around 0.75 for the Blackbird, but can be varied significantly via the variable pitch propeller. That being said, we used a VSR right around 0.75 when our prop pitch was fixed, and used that same value for the design pitch of the prop. Increasing or decreasing the pitch leads to off-design performance which results in a less efficient propeller (at least at the design point).

  158. prezzi ugg in italia says:

    Jackets are fantastic way to maintain your sense of fashion alive. With stylish jackets in winters, you cannot break the stamp obtaining remarkable individual. In this season, you have great choice of selecting that brand which is ideal in producing different forms of jackets. Why don’t you consider Moncler The fabric. These jackets are having beautiful colors like pink white, grayscale blue that provide prominent look versus your appearance. When you seek for winter stuff, you often see same style of jackets everywhere, which featuring jeans. Let yourself release from most of these same style, when you are able to opt to buy a different step.As the majority of trends, we have not had a person to look carefully at it, when true became outdated bathroom and kitchen. But something what make us strange is that this moncler outlet jacket has never out in the people’s attention. Since the emergence of years, enables not disappeared from public’s sight and always be the preferred winter covers.These jackets are if you can most sophisticated of every one of the kind company. You might be able decide various types and models, and are able inside area of option. cheap moncler jackets for males, women and young youngsters are available. Famous . the mark something for the whole family would Familie.Ihre a contemporary and fashionable family hotel, where an individual go to any occasion despite the fact that wearing these jackets.Moncler clothing is the react for of the questions coming the female thought in terms of outerwear. Moncler clothing is a which adapts to your busy outlook on life. Moncler merchandise is evolving with huge pace, so women nowadays have a multitude of outfit jackets and options to decide on. moncler jackets like tracksuits look awesome and add to your persona during winter.Even early start could be more dry, because ought to be ready to make all facets. “He believed that the main lies not only in Europe show on time, but as well as to attract more international visitors than New york. However, the coming of moncler jackets break through women rules and list in get list.What’s more, you can match moncler jackets Women, that grants you the opportunity indulge their Moncler regarding luxury, comfort and fashion, The Moncler jacket is constructed from shiny water repellent polyamide fabric and quilted with 100% goose down filling class A, that makes this product extremely light and warm, thus you can gain more warm, Meeting Fashion and style. And now, moncler outlet are popular around the world, Appear being fashionable and comfortable, moncler has to be your best answer.

Comments are closed.

Discuss this article with the rest of the community on our Discord server!
Tagged

Mark Frauenfelder is the founding Editor-in-Chief of Make: magazine, and the founder of the popular Boing Boing blog.

View more articles by Mark Frauenfelder

ADVERTISEMENT

Maker Faire Bay Area 2023 - Mare Island, CA

Escape to an island of imagination + innovation as Maker Faire Bay Area returns for its 15th iteration!

Buy Tickets today! SAVE 15% and lock-in your preferred date(s).

FEEDBACK